Sei sulla pagina 1di 243



  

 

   


  

 

   


 
 


 
      
             
  
 
   
 !  
  
"        #     
$  !%  !      &  $  
'      '    ($    
'   # %  % )   % *   %
'   $  '     
+      " %        & 
'  !#     
 $,
 ( $


   
  -     . 
                  
  
           
  !  
"-           ( 
  %                
    
        .    
     %  
  %   %   %    $ 
      $ $ -    
      -            
    
         - -

// $$$   

0  


 
     

1& )*2/ +)


  
 345%66777&  %  ) -
8 
9  

*   &


    
 

/- :#   # * 


/- :4;7<1& )*2/ +)
"=   "    &  4;7<
ADVANCED ENGINEERING MATHEMATICS

BY

Dr. Mohamed Ismail Mohamed Hessein


Professor Of Mathematics
Faculty Of Engineering (Shoubra)
Benha University

2017
CONTENTS

Chapter Page
CONTENTS i
1 Linear Dependence and Independence 1
2 Second-order linear differential equations 21
3 Fourier series 54
4 Fourier integrals 100
5 Laplace transforms 111
6 Special functions 149
7 Partial differential equations 177
8 Solution of differential equations in series 204
References 229
Advanced Engineering Mathematics Chapter 1- Linear Dependence and Independence

Chapter 1
Linear Dependence and Independence
Let V be a vector space over a eld F. The following denes the notion of
linear dependence and independence of vectors over F. (One usually suppresses
mentioning F when the eld is understood.) This concept plays an essential role in the
theory of linear algebra and in mathematics in general.
Definition 1.1: We say that the vectors v1, v2, ... , vm in V are linearly dependent if
there exist scalars a1, a2, .. . , am in F, not all of them 0, such that
a1v1 + a2v2 + amvm = 0
Otherwise, we say that the vectors are linearly independent.
The above denition may be restated as follows. Consider the vector equation
a1v1 + a2v2 + amvm = 0 (*)
where the as are unknown scalars. This equation always has the zero
solution a1 = 0, a2 = 0, .. . , am = 0 . Suppose this is the only solution, that is,
suppose we can show: a1v1 + a2v2 + amvm = 0 implies a1 = 0, a2 = 0, .. . , am = 0.
Then the vectors v1, v2, ... , vm are linearly independent, On the other hand, suppose
the equation (*) has a nonzero solution, then the vectors are linearly dependent.
A set S = {v1, v2, ... , vm} of vectors in V is linearly dependent or independent
according to whether the vectors v1, v2, ... , vm are linearly dependent or independent.
An innite set S of vectors is linearly dependent or independent according to whether
there do or do not exist vectors v1, v2, ... , vm in S that are linearly dependent.

Warning: The set S = {v1, v2, ... , vm} above represents a list or, in other words, a
nite sequence of vectors where the vectors are ordered and repetition is permitted.
The following remarks follow directly from the above denition.

Remark 1: Suppose 0 is one of the vectors v1, v2, ... , vm say v1 = 0. Then the vectors
must be linearly dependent, because we have the following linear combination where
the coefficient of v1 0:
1v1 + 0 v2++ 0vm = 1. 0 + 0 ++ 0 = 0
Remark 2: Suppose v is a nonzero vector. Then v, by itself, is linearly independent,
because k v = 0, v 0 implies k = 0.
Remark 3: Suppose two of the vectors v1, v2, ... , vm are equal or one is a scalar
multiple of the other, say v1 = k v2. Then the vectors must be linearly dependent,
because we have the following linear combination where the coefficient of v1 0:
v1 k v2 + 0 v3 + + 0 vm = 0
Remark 4: Two vectors v1 and v2 are linearly dependent if and only if one of them is
a multiple of the other.
Remark 5: If the set {v1, v2, ... , vm} is linearly independent, then any rearrangement


of the vectors vi1 , vi 2 ,, vi m is also linearly independent.
Remark 6: If a set S of vectors is linearly independent, then any subset of S is linearly
independent. Alternatively, if S contains a linearly dependent subset, then S is
linearly dependent.
1
Advanced Engineering Mathematics Chapter 1- Linear Dependence and Independence

Example 1.1:
a) Let u = (0, 1, 1, 1), v = (2, 1, 3, 1), w = ( 5, 4, 9, 4). Then u, v, w are linearly
dependent, because 3u + 5v 2w = 3(0, 1, 1, 1) + 5 (2, 1, 3, 1) 2 ( 5, 4, 9, 4)=
= ( 0+1010 , 3+58, 3+1518, 3+58) = ( 0, 0, 0, 0)

b) We show that the vectors u = (1, 2, 3, 3), v = (2, 5, 7, 6), w = ( 1, 3, 5, 3).


are linearly independent. We form the vector equation x u + y v + z w = 0, where x,
y, z are unknown scalars. This yields
1 2 1 0 x + 2 y + z = 0 x +2 y + z = 0 x +2 y + z = 0
2 5 3 0 2x +5 y + 3z = 0 y+ z=0 y+z=0
x y z 3x +7 y + 5z = 0 y + 2z = 0 2z = 0
3 7 5 0 3x +6 y + 3z = 0 0+ 0=0

3 6 3 0

Back-substitution yields x = 0, y = 0, z = 0. We have shown that


x u + y v + z w = 0 implies x = 0, y = 0, z = 0
Accordingly u, v, w are linearly independent.

c) Let V be the vector space of functions from R into R. We show that the functions
t 2
f (t) = sin t, g(t) = e , h(t) = t are linearly independent. We form the vector (function)
equation a f + b g + c h = 0, where a, b, c are unknown scalars. This function
t 2
equation means that, for every value of t, a sin t + b e + c t = 0.
Thus, in this equation, we choose appropriate values of t to easily get
a = 0, b = 0, c = 0. For example,
(i) Substitute t = 0 to obtain a (0) + b (1) + c (0) = 0 or b = 0.
2
(ii) Substitute t = to obtain a (0) + 0 (e ) + c ( ) = 0 or c = 0.
/2 2
(iii) Substitute t =/2 to obtain a (1) + b (e ) + 0 ( /4) = 0 or a = 0.
We have shown a f + b g + c h = 0 implies a = 0, b = 0 , c = 0 .
Accordingly, f, g, h are linearly independent.
1.1. Linear Dependence and Linear Combinations:
The notions of linear dependence and linear combinations are closely related.
Specifically, for more than one vector, we show that the vectors v1, v2, ... , vm are
linearly dependent if and only if one of them is a linear combination of the others.
Suppose, say, vi is a linear combination of the others,
v i a1v1 a i 1v i 1 a i 1v i 1 a m v m
Then by adding vi to both sides, we obtain
a1v1 a i 1v i 1 v i a i 1v i 1 a m v m 0
where the coefficient of vi is not 0. Hence, the vectors are linearly dependent.
Conversely, suppose the vectors are linearly dependent, say,
b1v1 b jv j b m v m 0, where b j1 0
Then we can solve for vj obtaining
v j b 1j b1v1 b 1j b j1v j1 b 1j bi 1vi 1 b 1j b m v m .

2
Advanced Engineering Mathematics Chapter 1- Linear Dependence and Independence

and so vj is a linear combination of the other vectors.


We now state a slightly stronger statement than the one above. This result has many
important consequences.

Lemma 1: Suppose two or more nonzero vectors v1, v2, ... , vm are linearly dependent.
Then one of the vectors is a linear combination of the preceding vectors, that is, there
exists k > 1 such that v k c1v1 c 2 v 2 c k 1v k 1 .
1.2. Basis and Dimension

First we state two equivalent ways to dene a basis of a vector space V.

Definition 1.2: A set S = {u1, u2, ... , un} of vectors is a basis of V if it has
the following two properties:
(1) S is linearly independent. (2) S spans V.

Definition 1.3: A set S = {u1, u2, ... , un} of vectors is a basis of V if every
v V can be written uniquely as a linear combination of the
basis vectors.
The following is a fundamental result in linear algebra.

Theorem 1.1: Let V be a vector space such that one basis has m elements and
another basis has n elements. Then m = n.
A vector space V is said to be of nite dimension n or n dimensional, written
dim V = n.
if V has a basis with n elements. Theorem 2 tells us that all bases of V have the
same number of elements, so this denition is well dened.
The vector space {0} is dened to have dimension 0.
Suppose a vector space V does not have a nite basis. Then V is said to be of
innite dimension or to be innite-dimensional.
The above fundamental Theorem 1 is a consequence of the following
replacement lemma .
Lemma 1.2:
Suppose {v1, v2, ... , vn} spans V, and suppose {w1, w2, ... , wm} is linearly
independent. Then m n, and V is spanned by a set of the form
{v1, v2, ... , vn, w1, w2, ... , wm }
Thus, in particular, n + 1 or more vectors in V are linearly dependent.
Observe in the above lemma that we have replaced m of the vectors in the
spanning set of V by the m independent vectors and still retained a spanning set.
Examples of Bases
This subsection presents important examples of bases of some of the main vector
spaces appearing in this text.
3
Advanced Engineering Mathematics Chapter 1- Linear Dependence and Independence

n n
a) Vector space R : Consider the following n vectors in R :
e1 = (1, 0, 0, 0, ... , 0, 0), e2 = (0, 1, 0, 0, .. . , 0, 0), , en = (0, 0, 0, 0, .. . , 0, 1)
These vectors are linearly independent. Furthermore, any vector u = (a1, a2, ... , an) in
Rn can be written as a linear combination of the above vectors. Specically,
u = a1e1 + a2e2 + + anen.
n n
Accordingly, the vectors form a basis of R called the usual or standard basis of R .
n n
Thus (as one might expect), R has dimension n. In particular, any other basis of R
has n elements.
b) Vector space M = Mr,s of all r s matrices: The following six matrices form a
basis of the vector space M2,3 of all 2 x 3 matrices over R:
1 0 0 0 1 0 0 0 1 0 0 0 0 0 0 0 0 0
0 0 0 , 0 0 0 , 0 0 0 , 1 0 0 , 0 1 0 , 0 0 1 .

More generally, in the vector space M = Mr,s of all r s matrices, let Eij be the
matrix with ij-entry 1 and 0s elsewhere. Then all such matrices form a basis of Mr,s
called the usual or standard basis of Mr,s. Accordingly, dim Mr,s = r s.
c) Vector space Pn(t) of all polynomials of degree n:
2 3 n
The set S = {1, t, t , t , .. . , t } of n+1 polynomials is a basis of Pn(t). Specically,
any polynomial f (t) of degree n can be expressed as a linear combination of these
powers of t, and one can show that these polynomials are linearly independent.
Therefore, dim Pn(t) = n + 1.
d) Vector space P(t) of all polynomials:
Consider any nite set S = {f1(t), f2(t), ... , fm(t)} of polynomials in P(t), and let m
denote the largest of the degrees of the polynomials. Then any polynomial g(t) of
degree exceeding m cannot be expressed as a linear combination of the elements of S.
Thus, S cannot be a basis of P(t). This means that the dimension of P(t) is innite. We
2 3
note that the innite set S'= {1, t, t , t , }, consisting of all the powers of t, spans
P(t) and is linearly independent. Accordingly, S' is an innite basis of P(t).

1.3. Theorems on Bases


The following three theorems will be used frequently.

Theorem 1.2: Let V be a vector space of nite dimension n. Then:


i) Any n + 1 or more vectors in V are linearly dependent.
ii) Any linearly independent set S = {u1, u2, ... , un} with n elements is a basis of V.
iii) Any spanning set T = {v1, v2, ... , vn} of V with n elements is a basis of V.

Theorem 1.3: Suppose S spans a vector space V. Then:


(i) Any maximum number of linearly independent vectors in S form a basis of V.
(ii) Suppose one deletes from S every vector that is a linear combination of
preceding vectors in S. Then the remaining vectors form a basis of V.

4
Advanced Engineering Mathematics Chapter 1- Linear Dependence and Independence

Theorem 1.4: Let V be a vector space of finite dimension and let S = {u1, u2, ... , un}
be a set of linearly independent vectors in V. Then S is part of a basis of V; that is, S
may be extended to a basis of V.
Example 1.2:
4
a) The following four vectors in R form a matrix in echelon form:
(1, 1, 1, 1), (0, 1, 1, 1), (0, 0, 1, 1), (0, 0, 0, 1)
1 0 0 0
0 1 0 0

0 0 1 0

0 0 0 1
4
Thus, the vectors are linearly independent, and, because dim R = 4, the four vectors
4
form a basis of R .
(1, 2, 3, 4) = 1(1, 1, 1, 1) + 1(0, 1, 1, 1) + 1(0, 0, 1, 1) +1 (0, 0, 0, 1)
(3, 6, 5, 3) = 3(1, 1, 1, 1) + 2(0, 1, 1, 1) + 0(0, 0, 1, 1) 8 (0, 0, 0, 1)
b) The following n + 1 polynomials in Pn(t) are of increasing degree:
2 n
1, t 1, (t 1) , ... , (t 1)
Therefore, no polynomial is a linear combination of preceding polynomials, hence,
the polynomials are linear independent. Furthermore, they form a basis of Pn(t),
because dim Pn(t) = n + 1.
3
c) Consider any four vectors in R , say (1, 1, 1), (1, 1, 0), (1, 0, 0), (3, 5, 7)
By Theorem 2(i), the four vectors must be linearly dependent, because they come
3
from the three-dimensional vector space R .
1.4. Dimension and Subspaces:
The following theorem gives the basic relationship between the dimension of a vector
space and the dimension of a subspace.
Theorem 1.5: Let W be a subspace of an n-dimensional vector space V.
Then dim W n. In particular, if dim W = n, then W = V.
Proof:
Because V is of dimension n, any n + 1 or more vectors are linearly dependent.
Furthermore, because a basis of W consists of linearly independent vectors, it cannot
contain more than n elements. Accordingly, dim Wn.
In particular, if {w1, w2, ... , wn} is a basis of W, then, because it is an independent

5
Advanced Engineering Mathematics Chapter 1- Linear Dependence and Independence

set with n elements, it is also a basis of V. Thus, W = V when dim W = n.


3 3
Example 1.3: Let W be a subspace of the real space R . Note that dim R = 3.
Theorem 2 tells us that the dimension of W can only be 0, 1, 2, or 3. The following
cases apply:
If dim W = 0, then W = {0}, a point.
If dim W = 1, then W is a line through the origin 0.
If dim W = 2, then W is a plane through the origin 0.
3
If dim W = 3, then W is the entire space R .
1.5. Linear Independent Functions
Two functions f(x) and g(x) are said to be linearly independent over an interval x
if the equation a f(x) + b g(x) = 0 (*)
is only true for all x in the interval if a = b = 0. The functions are said to be linearly
dependent if (*) is true for some non vanishing constants a and b.
When the functions are linearly dependent, provided a 0, equation (*) can be written
b a
f (x) g(x) , with a corresponding result g(x) f (x) , if b 0,
a b
showing that in each case the linear dependence of the functions means they are
proportional. We have established the following simple test.
Test for linear independence of f(x) and g(x) over x
The two functions f(x) and g(x) will be linearly independent over x if they are
not proportional over the interval; otherwise, they will be linearly dependent.
Example 1.4:
Apply the test for linear independence to the following pairs of functions.
(a) ex and e2x are linearly independent for all x because e2x/ex = ex is defined for
all x and ex is not a constant.
(b) n x2 and n x3 are linearly dependent for x > 0, because n x2 = 2 n x and
n x3 = 3 n x, so n x2/n x3 = 2/3 is a constant, and the logarithmic function is
defined for x > 0.
(c) sinh 2x and sinh x cosh x are linearly dependent for all x because sinh 2x = 2 sinh x
cosh x.
The notion of the linear independence of functions is of special significance when the
functions are solutions of homogeneous differential equations. This is because it will be
seen later that all particular solutions of such differential equations can be represented in
the form of suitable linear combinations of as many linearly independent solutions as the
equation allows. In fact, the number of linearly independent solutions is equal to the
order of the differential equation, so the second order differential equation (1) has two
linearly independent solutions.
So, if f(x) and g(x) are linearly independent solutions of (1), and a and b are general
solution arbitrary constants, the general solution of (1) from which all particular
solutions can be obtained can be written y(x) = a f(x) + b g(x).
1.6. Linear Independence and the Wronskian
Recall from linear algebra that two vectors v and w are called linearly dependent if there
are nonzero constants a and b with a v + b w = 0
6
Advanced Engineering Mathematics Chapter 1- Linear Dependence and Independence

We can think of differentiable functions f(x) and g(x) as being vectors in the vector
space of differentiable functions. The analogous definition is
Let f(x) and g(x) be differentiable functions. Then they are called linearly dependent if
there are nonzero constants a and b with a f(x) + b g(x) = 0 for all x. Otherwise they are
called linearly independent.
Example 1.5:
The functions f(t) = 2sin2 t and g(t) = 1 cos2 t are linearly dependent since
(1)(2sin2 t)+(2)(1cos2 t)= 0

Example 1.6:
The functions f(t) = t and g(t) = t2 are linearly independent since otherwise there would
be nonzero constants a and b such that a t + b t2 = 0 for all t. First let t = 1. Then a + b =
0 . Now let t = 2. Then 2 a + 4 b = 0.
This is a system of 2 equations and two unknowns. The determinant of the
corresponding matrix is
42=2
Since the determinant is nonzero, the only solution is the trivial solution. That is a = b =
0, The two functions are linearly independent. In this example, we arbitrarily selected
two values for t. It turns out that there is a systematic way to check for linear
dependence. The following theorem states this way.
Theorem 1.6:
Let f and g be differentiable on [,]. If Wronskian W(f, g)(to) is nonzero for some to
in [, ] then f and g are linearly independent on [, ]. If f and g are linearly dependent
then the Wronskian is zero for all t in [, ].
Proof
If a f(t) + b g(t) = 0. Then we can take derivatives of both sides to get
a f'(t) + b g'(t) = 0
This is a system of two equations with two unknowns. The determinant of the
f (t) g(t)
corresponding matrix is the Wronskian. W(f, g)(t) =
f '(t) g '(t) .
Hence, if the Wronskian is nonzero at some to, only the trivial solution exists. Hence
they are linearly independent.

Example 1.7:
Show that the functions f(t) = t and g(t) = e2t are linearly independent.
Solution:
We compute the Wronskian f '(t) = 1, g '(t) = 2e2t
f(t) g(t) t e 2t
2t 2t 2t
The Wronskian is =
f '(t) g'(t) 2 t = (t)(2e ) (e )(1) = (e )(2t 1).
1 2e
Now plug in 0 to get W(f, g)(0) = 1.
which is nonzero. We can conclude that f and g are linearly independent.

7
Advanced Engineering Mathematics Chapter 1- Linear Dependence and Independence

Example 1.8:
1. The vectors e1 = (1 ,0 ,0, , 0), e2 = (0 ,1 ,0, , 0), e3 = (0 ,0 ,1, , 0),,
en = (0 ,0 ,0, , 1) in Rn are linearly independent.
To show this, consider any scalars a1, a2,ak such that a1 e1+ a2e2 + a3e3+ ... + am em =
(a1, a2, a3,,an ) = (0 ,0 ,0, , 0)
The last equation implies that a1= a2 = a3 == ak = 0.
The vector space Rn is spanned by the unit n-vectors
e1 = (1, 0, 0, 0, . . . , 0), e2 = (0, 1, 0, 0, . . . , 0), . . . , en = (0, 0, 0, 0, . . . , 1).
These vectors can be put in identity matrix in (echelon) form.
1 0 0 0 e1
0 1 0 0 e
2
A = = e1 e2 en 1 en

0 0 1 0 e n 1
0 0 0 1 en
2. The vectors u = (1, 2, 3), v = (4, 5, 6), and w = (7, 8, 9), in R3 are linearly dependent,
since u2v+ w = 0.
How did we come up with this? Start with the equation in the definition of linear
dependence au +bv +c w = (0, 0, 0), and solve this system for possible values of a, b , c.
2. We can write this system in matrix form as
1 4 7 a 0
2 5 8 b 0

3 6 9 c 0
The coefficient matrix A is simply the matrix with columns u, v, w which we can write
as A = [u, v, w]. If we reduce A to row echelon form we get
1 4 7 1 4 7 1 4 7
0 6 0 6 0

3
3 1 2

0 6 12 0 0 0 0 0 0
We have found that rank A = 2 is not maximal, and so the system is singular and has a
nonzero solution (many in fact). i.e. a c = 0 , b + 2c = 0 a = b/2 = c.
If we take c = 1 a = 1 , b = 2 , c = 1.
If we take c = 2 a = 2, b = 4 and so on. u 2v + w = 0 .
Theorem 1.7: Let v1, v2, v3, ..., vm Rn be a set of vectors and let A = (v1v2v3...vm)
Then the following statements are equivalent.
1. v1, v2, v3, ..., vm are linearly independent.
2. v1, v2, v3, ..., vm span Rn.
3. v1, v2, v3, ..., vm form a basis of Rn.
4. A 0.
5. rank A = n.

8
Advanced Engineering Mathematics Chapter 1- Linear Dependence and Independence

The fact that any basis of Rn contains exactly n vectors tells us that there are n
independent directions in Rn. This is why we say that Rn is n-dimensional.
Definition 1.4: The number of vectors in any basis of a subspace V is called the
dimension of V.
This definition only makes sense if any basis of V has the same number of vectors.
Fortunately, this can be shown to be true.

Theorem 1.8: Let S = {v1, v2, v3, ..., vm} be a set of m vectors in Rm and let A be the
square matrix of vectors of S. Then S is linearly independent if and only if det A 0.

1) An example of a linear combination of vectors in R5 is provided by the vector


sum (m = 3, n = 5) ; A = 2x + y + 3z, B = x + 2y z
where x = (1, 2, 3, 0, 4), y = (2, 1, 4, 1,3), and z = (6, 0, 2, 2,1). The vector
in R5 formed by this linear combination is
A = 2x+ y+3z = 2(1, 2, 3, 0, 4) + (2, 1, 4, 1,3) + 3(6, 0, 2, 2,1) = (22, 5, 16, 7, 2).

B = x + 2y2 z = (1, 2, 3, 0, 4) + 2(2, 1, 4, 1,3) (6, 0, 2, 2,1) = (1, 4, 9, 0, 1).

2) Let u = (1, 4, 2, 3) , v = (7, 10, 4, 1) , w = (2, 1, 5, 4), These are dependent


vectors since a (1, 4, 2, 3) +b (7, 10, 4, 1) + c (2, 1, 5, 4) = (0, 0, 0, 0)
i.e. 3 (1, 4, 2, 3) +1 (7, 10, 4, 1) + 2 (2, 1, 5, 4)
a = 3 , b = 1 , c = 2
We can calculate the values of a, b, and c by solving the system of linear equations:
a + 7b 2c = 0, 4 a +10 b + c = 0, 2 a 4 b + 5 c = 0, 3 a b 4 c = 0.
3) An example of linear dependence in R4 is provided by the vectors
x =(1, 0, 2, 5), y = (2, 1, 2, 1), z = (3, 2, 1, 0), and t = (1,1,1, 7), because
t = 2x 3y + z.
2(1, 0, 2, 5) 3 (2, 1, 2, 1)+(3, 2, 1, 0) = (1,1,1, 7).

As example of linear independence in R4 is provided by the vectors


x1 =(1, 0, 2, 5), x2 = (2, 1, 2, 1), x3 = (3, 2, 1, 0), and x4 = (1,1,1, 8),
a(1, 0, 2, 5) +b (2, 1, 2, 1) + c(3, 2, 1, 0) + d(1,1,1, 8) = (0, 0, 0, 0).
with a = b = c = d = 0.
Example 1.9: The vectors (1,1,0) , (1,0,1), (0,1,1) are independent in R3 .
Since a (1,1,0) + b (1,0,1) + c (0,1,1) = (0, 0, 0)
This is equivalent to the following set of linear equations: a + b = 0 , a + c = 0,
b + c = 0.
You can verify that the solution is a = b = c = 0. Hence, the vectors are independent.

Example 1.10: We know that the set of vectors {(1, 2) , (n 3 , n 2) . ( e4 , e5)}in R2


is dependent without doing any computation. Any set of three (or more) vectors in R2 is
dependent.

Example 1.11: R[x] is a vector space over the reals. The set 1 , x , x2 , x3 , is
independent. For if ao + a1 x + a2 x2 + + an xn = 0, it follows that ai = 0 for all i.

9
Advanced Engineering Mathematics Chapter 1- Linear Dependence and Independence

Example 1.12: Determine whether the set of vectors {(1, 0, 1, 2), (1, 2, 2, 1), (0, 1, 2, 1)}
is Linear dependence or Linear Independence in.

In order to determine if the three vectors in u, v , w R4, are linearly dependent or


independent, where u = (1, 4, 2, 3) , v = (7, 10, 4, 1) , w = (2, 1, 5, 4) form the
matrix,
1 4 2 3 u 1 4 2 3 u
7 10 4 1 v
0 18 18 22 v 7u
2 1 5 4 w 0 9 9 1 w 2u
1 4 2 3 u 1 4 2 3 u
0 9 9 1 w 2u 0 9 9 1 w 2u

0 18 18 22 v 7 u 0 0 0 20 v 3u 2 w

1 7 2 1 7 2
4 10 1 0 18 9 4

2 4 5 0 18 9 2

3 1 4 0 22 2 3
1 7 2 1 7 2
0 2 1 ( 4)/(-9) 0 1 0.5 ( 4)/( 18)

0 0 0 +2 0 0 4.5 (11 17 9)/18

0 11 1 ( 3) / (2) 0 0 0 2
It is clear that these vectors are linearly dependent and 0 = 2 +
= 2 + or = 2 or 2 =
= (4, 10, 1) = 2 (1, 7, 2) + (2, 4, 5) = (2, 14, 4) + (2, 4, 5)

Example 1.13: The vectors (2, 3) and (10, 15) are dependent in R2 . To show this, we
have to find numbers a and b, not both 0, such that a (2, 3) + b(10, 15) = (0, 0).
There are many pairs of numbers that work. For example,
5 (2, 3) + 1 (10, 15) = (0, 0).
Likewise, 10 (2, 3) + 2 ( 10, 15) = (0, 0).
More generally, vectors (a , b) and (c, d) in R2 are dependent if and only if they are
multiples of one another.
Example 1.14: The set of vectors {(4, 1, 2) , (3, 3, 0), (0, 1, 1)} is a dependent set in R3.
For 3(4, 1, 2) + 1(3, 3, 0) + 4 (0, l , 1) = (0, 0, 0).

10
Advanced Engineering Mathematics Chapter 1- Linear Dependence and Independence

(One of the things we'll discuss shortly is how you find numbers like 3, 1, and 4 which
give a linear combination which equals 0 .)

Example 1.15: Is S = {(1, 0, 1), (2, 1,1), (5,1, 0), (2, 1, 1)} linearly dependent or
linearly independent?
Solution: We have a set of n = 4 vectors in R3 (i.e., Rm with m = 3). Since n = 4 > m = 3
(i.e., n > m), then S is linearly dependent.

Example 1.16: Is S = {(1, 1, 1), (1, 0, 1), (0, 2, 0)} linearly dependent or linearly
independent?
1 1 0
Solution: We form the matrix A = 1 0 2 .

1 1 0
1 1
Then expanding along column 3 gives et A = 2(1)2+3 0.
1 1
Since det A = 0, S is linearly dependent.

Example 1.17:
Show that the following vectors are linearly independent,
u = (1, 2, 0, 2), v = (1, 1, 1, 0) , w = (2, 0, 1, 3)
Find a fourth vector x so that the set {u, v, w, x} is a basis for R4 ?
Solution:
To find the one vector that we need to complete a basis (the space has dimension four),
we could start with the matrix
1 1 2 a
2 1 0 b

0 1 1 c

2 0 3 d
Row reduce this and in the end select a, b, c, d so that the reduced form has independent
columns (four pivots).
Apply row-reduction suggested above and obtained
1 1 2 a 1 1 2 a 1 1 2 a
2 1 0 b 0 1 4 b 2a 0 1 4 b 2a

0 1 1 c 0 1 1 c 0 0 3 c 2a b

2 0 3 d 0 2 1 d 2a 0 0 7 d 2a 2b
1 1 2
0 1 4
It is easy to see that the rank of the matrix is 3 ,
0 0 3

0 0 7

11
Advanced Engineering Mathematics Chapter 1- Linear Dependence and Independence

which means that the vectors u, v, w are linearly independent. complete the reduction,
we get
1 1 2 a 1 1 2 a
0 1 4 b 2a 0 1 4 b 2a
(1)
0 0 21 7c 14a 7b 0 0 21 7c 14a 7b

0 0 21 3d 6a 6b 0 0 0 8a b 7c 3d
Taking a = 1 and b = c = d = 0 in (1) will give us a matrix with independent columns
(we want 8a +b + 7c + 3d = 8 0). So we can take (1, 0, 0, 0) to be the forth vector
needed to complete a basis. We can see also that
1 1 2 1 1 1 2 0 1 1 2 0 1 1 2 0
2 1 0 0 2 1 0 1 2 1 0 0 2 1 0 0
8 , 1, 7, 3.
0 1 1 0 0 1 1 0 0 1 1 1 0 1 1 0
2 0 3 0 2 0 3 0 2 0 3 0 2 0 3 1

Example 1.18: Determine whether {u, v, w} is linearly independent, where


u = (1, 1, 2, 1), v = (0, 2, 1, 1), and w = (3, 1, 2, 0).
Solution: Using the definitions, we wish to know whether a u + b v + c w = 0 has only
the trivial solution. Suppose that a u + b v + c w = 0 .
Then a(1, 1, 2, 1) + b(0, 2, 1, 1) + c(3, 1, 2, 0) = (0, 0, 0, 0)
(a, a , 2a, a) + (0, 2b, b, b) + (3c, c, 2c, 0) = (0, 0, 0, 0)
(a + 3c, a + 2b + c, 2a + b + 2c, a + b) = (0, 0, 0, 0)
Equating corresponding components, we have
a+ 3c = 0
a + 2b + c = 0
2a + b + 2c = 0
a+ b =0
1 0 3 0 1 0 0 0
1 2 1 0 0 1 0 0
Forming the augmented matrix, we get
2 1 2 0 0 0 1 0

1 1 0 0 0 0 0 0
We see that the system has the unique solution a = b = c = 0. Thus, the homogeneous
system has only the trivial solution. Since there is no solution in which any of the
constant values is non-zero, we see that the set {u, v, w} is linearly independent.

Example 1.19: Given the set S = {u, v, w, x, y}, with u = (1, 2, 3), v = (1, 1, 2),
w = (0, 1, 1), x = (2, 3, 1), and y = (1, 4, 1). Show that S is linearly dependent and
express (at least) one of the vectors as a linear combination of some of the others.
Solution: We construct the matrix as in the test for linear independence, and transform it
to echelon form:

12
Advanced Engineering Mathematics Chapter 1- Linear Dependence and Independence

1 1 0 2 1 1 0 1 0 2
2 1 1 3 4 0 1 1 0 3

3 2 1 1 1 0 0 0 1 1
Notice that column 3 of the row reduced matrix, (1,1, 0), is not an elementary column.
Notice also that in this matrix, column 3 is equal to one times column 1 plus (1) times
column 2, i.e., (1,1, 0) = 1(1, 0, 0)+(1)(0, 1, 0).
We see that the entries in the non-elementary column 3 are the coefficients of
dependence with which this vector can be expressed as a linear combination of the
vectors preceding it in the original matrix.
What the theorem is telling us is that the third column of the original matrix is the
same linear combination of the vectors which were transformed into these elementary
vectors. That is, just as it is true in the last matrix that column 3 is equal to 1 times the
first column plus 1 times the second column (using columns of the last matrix), in the
original matrix we also have C3 = C1 C2. We can, of course, verify this:
u v = (1, 2, 3) (1, 1, 2) = (1 1, 2 1, 3 2) = (0, 1, 1) = w
Thus, we see that w is dependent on u and v, and the entries in column 3 of the last
matrix give the coefficients of dependency. And since we can express w as a linear
combination of u and v, we see that the set S is linearly dependent.
Look also at column 5 of the matrix. This column is not an elementary column, so
the vector y can be written as a linear combination of some of the other vectors. Which
ones? We look for the vectors (i.e., columns) containing leading 1s in the last matrix,
because these are elementary columns. We see that row 1s leading 1 is in column 1,
row 2s leading 1 is in column 2 and row 3s leading 1 is in column 4.
In the last matrix we have:
(2,3, 1) = 2(1, 0, 0) 3(0, 1, 0) + 1(0, 0, 1)
so that column 5 = 2 (column 1) + (3) (column) 2 + 1 (column 4).
This tells us that this relationship also holds true in the original matrix. That is, we have
y = 2u 3v + x. (Again, we can easily confirm that this is true.) Thus, we see that y is
dependent on u, v and x, and that the entries in column 5 of the last matrix give us the
coefficients of dependence, i.e., tell us how to write y as a linear combination of these
vectors.

Example 1.20: consider the functions x2, x, and 1. Take the Wronskian:
x2 x 1
W = 2x 1 0 2
2 0 0
Note that W is always non-zero, so these functions are independent everywhere.
Consider, however, x2 and x:
x2 x
W= x 2 2x 2 x 2
2x 1
Here W = 0 only when x = 0. Therefore x2 and x are independent except at x = 0.
Consider 2x2 + 3, x2, and 1:

13
Advanced Engineering Mathematics Chapter 1- Linear Dependence and Independence

2x 2 3 x 2 1
W = 4x 2x 0 8x 8x 0
4 2 0
Here W is always zero, so these functions are always dependent. This is intuitively
obvious, of course, since 2x2 + 3 = 2(x2) + 3(1)

Example 1.21: Consider the functions u = sin x, v = cos x, and w = sin x + cos x .
Then f(x) = 3 sin x + 5 cos x = 3 u + 5 v, unique when expressed with linearly
independent functions,
but f(x) = 3 sin x + 5 cos x = 5 w 2 u = 3 w + 2 v not unique when expressed with
linearly dependent functions.
Example 1.22:
Let u = sin x , v = cos x and w = sin x + cos x from vector space V. Find which of the
following can be represented in terms of u and v, and w.
a) f(x) = 3 sin x + 5 cos x = 3 u + 5 v = 5 w 2 u = 3 w + 2 v not unique when
expressed with linearly dependent functions.
b) cos2x can't expressed by u, v and w only.
c) sin2 x can't expressed by u, v and w only.
d) sin 2x can't expressed by u, v and w only.
e) cos 3x can't expressed by u, v and w only.
f) 5 can't expressed by u, v and w only.
Example 1.23:
Let u = cos2 x and v = sin2 x form a basis for a vector space V. Find which of the
following can be represented in terms of u and v, and so lie in V.
(a) 2. (b) sin 2x. (c) 0.
(d) cos 2x. (e) 2 + 3x. (f) 3 4 cos 2x.
Solution:
a) 2 = 2 cos2 x + 2 sin2 x = 2(u + v ),
b) sin 2x V
c) 0 = cos2 x cos2 x = sin2 x sin2 x = u u = v v
d) cos 2x = cos2 x sin2 x = u v
e) 2 + 3x V
f) 3 4 cos 2x = 3(cos2 x + sin2 x) 4 cos2 x + 4sin2 x = 3( u + v) -4(u v) = u + 7v.
Example 1.24: Let u = (2, 0) , v = (3, 5),
and w = (8, 6).
Explain (without doing any computations) how you know that the set of vectors
{u, v, w } is linearly dependent. Then write a linear dependence relation for this
set. (This latter part does require computation.)
We know that the set of vectors {u, v, w} in R2 is dependent without doing any
computation. Any set of three (or more) vectors in R2 is dependent.
a (2, 0) + b (3, 5) + c(8, 6) = (0, 0)
2a 3b + 8c = 0 , 5b +6c = 0

14
Advanced Engineering Mathematics Chapter 1- Linear Dependence and Independence

2 3 8 a 0
0 5 6 b 0

This means that w can be expressed as a linear combination of {u, v} or v as a linear
combination of {u, w} as follows
2 0 u 1 0 u/2 1 0 u/2

3 5 v 0 5 v 3u / 2 0 5 v 3u / 2
8 6 w 0 6 w 4u 0 0 5w 29u 6v
This system will be consistent if 29u + 6 v 5w = 0
i.e. 5 w = 29 u + 6 v , 29 u = 5 w 6v and 6v = 5 w 29 u.
Another Solution: Let u = (2, 0) , v = (3, 5), and w = (8, 6).
w = (8, 6) = a (2, 0) + b (3, 5) = (2a 3b , 5b ) solving we get 5b = 6 , 2a 3b = 8
a = 5.8, b = 1.2 w = (8, 6) = 5.8 u + 1.2 v 10 w = 58 u + 12 v

Example 1.25: Let u = (0, 0, 2) , v = (0, 5, 8)


and w = (3, 4, 1)
1. Determine whether the set {u, v, w} is linearly independent or linearly dependent.
2. If the set {u, v, w} is linearly dependent, then write a linear dependence relation for
0 0 3
this set. = 0 5 4 30 , then the set is linearly independent.
2 8 1
Example 1.26: Let u = (0, 0, 2) , v = (3, 4, 7) and
w = (3, 4, 1).
1. Determine whether the set {u, v, w} is linearly independent or linearly dependent.
2. If the set {u, v, w} is linearly dependent, then write a linear dependence relation for
this set.
0 3 3
= 0 4 4 0 ,
2 7 1
then the set is linearly dependent.
Let u = (0, 0, 2) , v = (3, 4, 7) and w = (3, 4, 1)
v = (3, 4, 7) = a (0, 0, 2) + b (3, 4, 1) = (3b , 4b, 2a+b )
solving we get 3b = 3, 2a+ b = 7, a = 3, b = 1 v = (3, 4, 7) = 3 u + w.
Example 1.27: Let u = (1, 4, 0) , v = (1, 2, 3) , w = (1, 4 ,12) , h = (1, 4, 3)
Explain why the set of vectors S= {u, v, w, h } is linearly dependent and find one of
them expressed as a linear combination of the other vectors.
We have a set of n = 4 vectors in R3 (i.e., Rm with m = 3). Since n = 4 > m = 3 (i.e., n >
m), then S is linearly dependent. This means that w can be expressed as a linear
combination of {u, v, h} or h as a linear combination of {u, v, w} as follows:

15
Advanced Engineering Mathematics Chapter 1- Linear Dependence and Independence

1 4 0 u 1 4 0 u 1 4 0 u
1 2 3 v 0 2 3 v u 0 2 3 vu
=
1 4 12 w 0 8 12 w u 0 0 0 w u 4(v u)

1 4 3 h 0 0 3 hu 0 0 3 hu
1 4 0 u
0 2 3 vu

0 0 0 w 4v 3u

0 0 3 hu
w + 4v 3u = 0 or w = 3u 4v ,
or 3u = w + 4v or 4v = 3u w
w = 3(1, 4, 0) 4(1, 2, 3) = (1, 4, 12)
1.7: Linear independence of basis functions:
Let V be the vector space of all functions of a real variable t. Then the functions et and
e2t in V are linearly independent.
Proof:
Suppose a and b are two real numbers such that a et + b e2t = 0, for all values of t.
We need to show that a = 0 and b = 0. In order to do this, we divide through by et (which
is never zero) and subtract to obtain b et = a.
In other words, the function b et must be independent of t, which only occurs when b =
0. It follows that a is also zero.
Examples 1.28: consider the functions x2, x, and 1. Take the Wronskian:
x2 x 1
W = 2x 1 0 2
2 0 0
Note that W is always non-zero, so these functions are independent everywhere.
Consider, however, x2 and x:
x2 x
W= x 2 2x 2 x 2
2x 1
Here W = 0 only when x = 0. Therefore x2 and x are independent except at x = 0.
2x 2 3 x 2 1
Consider 2x2 + 3, x2, and 1: W = 4x 2x 0 8x 8x 0
4 2 0
Here W is always zero, so these functions are always dependent. This is intuitively
obvious, of course, since 2x2 + 3 = 2(x2) + 3(1)

Note:
Linear dependence or independence is a property of a set of vectors, not of an individual
vector. That is, it has to do with the relationship between the vectors in the set.
16
Advanced Engineering Mathematics Chapter 1- Linear Dependence and Independence

1 0 1
In the matrix The column vectors i = (1, 0) , j = (0, 1), f = (1, 1),
0 1 1
are linearly dependent while the row vectors u = (1, 0, 1) and v = (0, 1, 1) are
linearly independent.
Also note: If S is a set of linearly dependent vectors, then at least one of the vectors in S
can be written as a linear combination of the others. This does not necessarily mean that
each vector in the set can be written as a linear combination of the others. For instance,
Although S = {(1, 0), (3, 0), (0, 1)} is a set of linearly dependent vectors in R2, since
(3, 0) =3 (1, 0) + 0 (0, 1). However, (0, 1) cannot be written as a linear combination of
(1, 0) and (3, 0), but G = {(1, 3, 0) , (0, 0, 1)} has two linearly independent vectors in R3.

Some remarks:
m
Theorem 1.9: Let S = {v1, v2, v3, ..., vm} be a set of m vectors in R and let A be the
square matrix of vectors of S. Then S is linearly independent if and only if det A 0.

5
1) An example of a linear combination of vectors in R is provided by the vector
sum (m = 3, n = 5) ; A = 2x + y + 3z, B = x + 2y z
where x = (1, 2, 3, 0, 4), y = (2, 1, 4, 1,3), and z = (6, 0, 2, 2,1). The vector
5
in R formed by this linear combination is
A = 2x+ y+3z = 2(1, 2, 3, 0, 4) + (2, 1, 4, 1,3) + 3(6, 0, 2, 2,1) = (22, 5, 16, 7, 2).
B = x + 2y2 z = (1, 2, 3, 0, 4) + 2(2, 1, 4, 1,3) (6, 0, 2, 2,1) = (1, 4, 9, 0, 1).

2) Let u = (1, 4, 2, 3) , v = (7, 10, 4, 1) , w = (2, 1, 5, 4), These are dependent


vectors since a (1, 4, 2, 3) +b (7, 10, 4, 1) + c (2, 1, 5, 4) = (0, 0, 0, 0)
i.e. 3 (1, 4, 2, 3) +1 (7, 10, 4, 1) + 2 (2, 1, 5, 4)
a = 3 , b = 1 , c = 2
We can calculate the values of a, b, and c by solving the system of linear equations:
a + 7b 2c = 0, 4 a +10 b + c = 0, 2 a 4 b + 5 c = 0, 3 a b 4 c = 0.
4
3) An example of linear dependence in R is provided by the vectors
x =(1, 0, 2, 5), y = (2, 1, 2, 1), z = (3, 2, 1, 0), and t = (1,1,1, 7),
because t = 2x 3y + z.
2(1, 0, 2, 5) 3 (2, 1, 2, 1) + (3, 2, 1, 0) = (1,1,1, 7).
4
As example of linear independence in R is provided by the vectors
x1 =(1, 0, 2, 5), x2 = (2, 1, 2, 1), x3 = (3, 2, 1, 0), and x4 = (1,1,1, 8),
a(1, 0, 2, 5) +b (2, 1, 2, 1) + c(3, 2, 1, 0) + d(1,1,1, 8) = (0, 0, 0, 0).
with a = b = c = d = 0.
3
Example 1.29: The vectors (1,1,0) , (1,0,1), (0,1,1) are independent in R .
Since a (1,1,0) + b (1,0,1) + c (0,1,1) = (0, 0, 0)
This is equivalent to the following set of linear equations:
a + b = 0 , a + c = 0, b + c = 0.
You can verify that the solution is a = b = c = 0. Hence, the vectors are independent.

17
Advanced Engineering Mathematics Chapter 1- Linear Dependence and Independence

4 5 2
Example 1.30: We know that the set of vectors {(1, 2) , (n 3 , n 2) . ( e , e )}in R is
2
dependent without doing any computation. Any set of three (or more) vectors in R is
dependent.
2 3
Example 1.31: R[x] is a vector space over the reals. The set 1 , x , x , x , is
2 n
independent. For if ao + a1 x + a2 x + + an x = 0, it follows that ai = 0 for all i.

Example 1.32: Determine whether the set of vectors {(1, 0, 1, 2), (1, 2, 2,1), (0, 1, 2,1)}
4
is Linear dependence or Linear Independence in R .
Solution:
In order to determine if the three vectors in u, v , w R4, are linearly dependent or
independent, where u = (1, 4, 2, 3) , v = (7, 10, 4, 1) , w = (2, 1, 5, 4) form the
matrix,
1 4 2 3 u 1 4 2 3 u
7 10 4 1 v 0 18 18 22 v 7u

2 1 5 4 w 0 9 9 1 w 2u
1 4 2 3 u 1 4 2 3 u
0 9 9 1 w 2u 0 9 9 1 w 2u

0 18 18 22 v 7u 0 0 0 20 v 3u 2w
It is clear that these vectors are linearly independent.(Since the last term in the third row
is not zero 20 0.
3
In order to determine if the Four vectors in , , , R ,
= (1, 7, 2) , = (4, 10, 1) , = (2, 4, 5) and = (3, 1, 4) are linearly dependent
3 m
or independent. We have a set of n = 4 vectors in R (i.e., R with m = 3). Since n = 4 >
m = 3 (i.e., n > m), then S is linearly dependent. In order to find one of
them expressed as a linear combination of the other vectors, form the matrix
1 7 2 1 7 2
4 10 1 0 18 9 4

2 4 5 0 18 9 2

3 1 4 0 22 2 3
1 7 2 1 7 2
0 2 1 ( 4)/(-9) 0 1 0.5 ( 4)/( 18)

0 0 0 +2 0 0 4.5 (11 17 9)/18

0 11 1 ( 3) / (2) 0 0 0 2
It is clear that these vectors are linearly dependent and 0 = 2 +
= 2 + or = 2 or 2 =
= (4, 10, 1) = 2 (1, 7, 2) + (2, 4, 5) = (2, 14, 4) + (2, 4, 5)
In order to determine if the three vectors in R4, u = (1, 4, 2, 3) ,
v = (7, 10, 4, 1) , w = (2, 1, 5, 4) are linearly dependent or independent, form the
matrix
18
Advanced Engineering Mathematics Chapter 1- Linear Dependence and Independence

1 4 2 3 u 1 4 2 3 u
7 10 4 1 v 0 9 9 2 w 2u

2 1 5 4 w 0 0 0 16 v 3u 2w
It is clear that these vectors are linearly independent.
Now, let us see the same required for the column vectors of the above matrix.
Example 1.33: Determine whether the set of vectors u = (1, 0, 1, 2), v = (0, 1, 2, 1) and
w = (1, 2, 2, 1) are linearly independent in R4 .
1 0 1 2 u 1 0 1 2 u 1 0 1 2 u
0 1 2 1 v 0 1 2 1
v 0 1 2 1 v

1 2 2 1 w 0 2 1 1 w u 0 0 3 3 w u v
It is clear that these vectors are linearly independent in R4. But the vectors
A = (1, 0, 1) , B = (0, 1, 2), C = (1, 2, 3) , D = (2, 1, 1) in R3 are linearly dependent
since n = 4 > m = 3. And
1 0 1 A 1 0 1 A 1 0 1 A
0 1 2 B 0 1 2 B 0 1 2 B

1 2 2 C 0 2 1 C A 0 0 3 C A 2B

2 1 1 D 0 1 1 D 2A 0 0 0 D A C B
Then D A C + B = 0 D = A + C B and B = A + C D A and
C = D A + B.
The question is whether you can find a, b , c R, not all 0, such that
a u + b v + c w = a (1, 0, 1, 2) + b (1, 2, 2, 1) +c (0, 1, 2, 1) = (0, 0, 0, 0)
This amounts to solving the system
1 1 0 0 1 1 0 0
0 2 1 a 0
0 2 1 0
b The augmented
1 2 2 0 1 2 2 0
c
2 1 1 0
2 1 1 0
However, we can see that row operations will never change the fourth column, so we
can omit it. So the row reduction is
1 1 0 1 0 1
0 2 1 0 1 2

1 2 2 0 0 0

2 1 1 0 0 0
Remembering that there is a fourth all zero column that we haven't writing, this says
a + c = 0 , b + 2c = 0. The parameterized solution is 2a = 2c = b = 2t
a = t, b = 2t, c = t.
a = 2t, b = t, c = t.
So, for example, we can get a nonzero solution by setting c = t = 1. Then a = 1 and
b = 2 .
In the row reduced echelon matrix, you can see that we didn't get a copy of the identity
matrix. If this had happened,
19
Advanced Engineering Mathematics Chapter 1- Linear Dependence and Independence

1 0 0
0 1 0

0 0 1

0 0 0
says a = 0, b = 0, and c = 0, and the vectors would have been independent.
You can just do the algorithm if you wish, but it's always better to understand where it's
coming from.
3 m
We have a set of n = 4 vectors in R (i.e., R with m = 3). Since n = 4 > m = 3
(i.e., n > m), then S is linearly dependent.

20
Advanced Engineering Mathematics Chapter 2- Second-Order Linear D.E.

Chapter 2
SECOND-ORDER LINEAR DIFFERENTIAL EQUATIONS
2.1 A second-order linear differential equation is one of the form
d2y dy
p(x) q(x)y f (x) ..........................................................................(1)
dx 2 dx
or, in alternative notation, y" + p(x)y' + q(x) y = f(x)
If f(x) is identically 0, then (1) reduces to
d2y dy
p(x)
q(x)y 0 ..................................................(2)
dx 2 dx
which is called the second-order linear homogeneous differential equation.
In order to discuss the solutions to a second-order linear homogeneous
differential equation, it will be useful to introduce some terminology. Two functions f
and g are said to be linearly dependent if one is a constant multiple of the other. If
neither is a constant multiple of the other, then they are called linearly independent.
Thus, g(x) = 3 sin x and h(x) = 5 sin x are linearly dependent, but
2
g(x) = x and h(x) = x are linearly independent. The following theorem is central to
the study of second-order linear homogeneous differential equations.
dy d2y
Theorem 2.1 Consider the homogeneous equation p(x)
q(x)y 0
dx 2 dx
where the functions p(x) and q(x) are continuous on a common open interval. Then
there exist linearly independent solutions u(x) and v(x) to (2) on that interval.
Furthermore, given any such pair of linearly independent solutions u(x) and v(x), a
general solution of (2) on the interval is given by y(x) = A u(x) + B v(x)............ (3)
That is, every solution of (2) on the interval can be obtained from (3) by choosing
appropriate values of the constants A and B; conversely, (3) is a solution of (2) for
all choices of A and B.
Second-order linear homogeneous differential equations with Constant coefficients
We will restrict our attention to second-order linear homogeneous equations of the
d2y dy
form p q y 0 ........................................................................................(4)
2 dx
dx
where p and q are constants. Since the constant functions p(x) = p and q(x) = q are
continuous on ( , ), it follows from Theorem 2.1 that to determine a general
solution to (4) we need only find two linearly independent solutions u(x) and v(x) on
( , ) .
The general solution will then be given by y(x) = A u(x) + B v(x), where A and B are
arbitrary constants.
mx
We will start by looking for solutions to (4) of the form y = e . This is
motivated by the fact that the first and second derivatives of this function are
multiples of y, suggesting that a solution of (4) might result by choosing m
appropriately. To find such an m, we substitute
mx mx 2 mx
y = e , y' = me , y" = m e .................................................(5)
21
Advanced Engineering Mathematics Chapter 2- Second-Order Linear D.E.
2 mx
into (4) to obtain (m + p m + q) e = 0 ................................................................. (6)
2
which is satisfied if and only if m + p m + q = 0 ................................................... (7)
mx
since e 0 for every x.
Equation (7), which is called the auxiliary equation for (4), can be obtained from (4)
d2y dy 2 1 0
by replacing by m , by m (= m ), and y by 1 (= m ). The solutions, m1
dx 2 dx
and m2, of the auxiliary equation can be obtained by factoring or by the quadratic
formula. These solutions are
p p 2 4q p p 2 4q
m1 , m2 ............................(8)
2 2
2
Depending on whether p 4q is positive, zero, or negative, these roots will be
distinct and real, equal and real, or complex conjugates. We will consider each of
these cases separately.
Distinct Real Roots
x x
If and are distinct real roots, then (4) has the two solutions y1 = e , y2 = e
x x
Neither of the functions e and e is a constant multiple of the other, so the general
x x
solution of (4) in this case is y = A e + B e ......................................................(9)

Example 2.1: Find the general solution of y" + y' 6 y = 0


2
Solution: The auxiliary equation is m + m 6 = 0 or equivalently (m 2)(m +3) = 0
so its roots are = 2, = 3. Thus, from (9) the general solution of the differential
2x 3x
equation is y = A e + B e , where A and B are arbitrary constants.
Equal Real Roots
If m1 and m2 are equal real roots, say m1 = m2 (= ), then the auxiliary equation
mx
yields only one solution of (4): u(x) = e
mx
However, substitution of the function v(x) = x e into the differential equation
shows that it is also a solution. The functions v(x) and u(x) are linearly independent
because v(x)/u(x) = x is not a constant, so in this case a basis for the solution space of
mx mx
(1) is formed by the functions e and x e , with the corresponding general
mx
solution y = A u(x) + B v(x) = e [ A + B x ]........................................................(10)
Example 2.2: Find the general solution of y" 4 y' + 4 y = 0
2 2
Solution: The auxiliary equation is m 4m + 4 = 0 or equivalently (m 2) = 0
so m = 2 is the only root. Thus, the general solution of the differential equation
2x
is y = e ( A + B x).
Complex Roots
If the auxiliary equation has complex roots m1 = + i and m2 = i, then both
x x
u(x) = e cos x and v(x) = e sin x are linearly independent solutions of (4) and
x
y = e (A cos x + B sin x)..................................................................................(12)
is the general solution.

22
Advanced Engineering Mathematics Chapter 2- Second-Order Linear D.E.

Example 2.3: Find the general solution of y" + y' + y = 0


2
Solution: The auxiliary equation is m + m + 1 = 0 has roots
1 1 4 1 3 1 1 4 1 3
m1 = i , m2 = i
2 2 2 2 2 2
Thus, the general solution of the differential equation is
x2 3 3
y= e A cos 2 x Bsin 2 x .

2.2 Linear Equations with Constant Coefficients
We have already remarked that the problem of finding the general solution of the
complete equation reduces to that of finding n linearly independent particular
solutions of the reduced equation, and any (preferably the simplest possible)
particular solution of the complete equation. This can be done in only a very few
cases. However, if the coefficients c1, c2,, cn, are constants, a complete theory and
several methods of solution are available. Because of this fact, and also because
linear equations with constant coefficients are of great practical importance as well as
theoretical interest we shall devote most of this section to their consideration.
Case I : Roots, Real and Different. If and are the roots:
Let us consider the equation with constant coefficients
y" ( + ) y' + y = 0
(D2 ( + )D + )y = 0
(D )(D )y = 0
(D )[(D )y] = 0
Let y = e mx , then m2 ( + ) m + = 0 (m )(m ) = 0,
If , then we have two different solutions ex and ex , the general solution
y = A ex + B ex.
Case II : Roots, Real and Equal.
If = , then we have y" 2 y' + 2 y = 0
(D2 2 D + 2 )y = 0
(D )2 y = 0, we have ex as solution , the second solution u ex, where u is a
function of x, to determine it,
applying (D )2 u ex to it and equate to 0.
Since D u ex = a ex u + ex u' (D ) ex u = ex u',
then (D ) ex u' = ex u + ex u" ex u = ex u" ,
and (D )2 ex u = ex u" = 0, u" = 0 u' = A , u = A + B x.
Then y = (A + B x) ex. Also from the fact that (D )2 ex u = ex u",
we can write (D )2 ex u = ex D2 u
In general F(D ) ex u = (D )n ex u = ex Dn u = ex F(D) u
Case III : Roots Imaginary. It must appear in conjugate pairs
i.e. if the roots are i, then the solution will be
y = C1 e( + i)x + C2 e( i)x = ex [C1 eix + C2 eix] =
= ex [C1{cos x + i sin x} + C2 {cos x i sin x}] =
= ex [(C1 + C2)cos x + i (C1 C2) sin x] = ex [A cos x + B sin x].

23
Advanced Engineering Mathematics Chapter 2- Second-Order Linear D.E.

Summary
d2y dy
Equation p(x) q(x)y 0
dx 2 dx
2
auxiliary equation m + p m + q = 0

Case General Solution


Distinct real roots , of the auxiliary equation y = A ex + B ex,
Equal real roots = (= ) of the auxiliary equation y = ex (A + B x),
Complex roots +i, i of the auxiliary equation y = ex[A cos x+ B sin x].

Ordinary Differential Equations - Second-Order Equations


The simplest of the second-order equations is y" + p y' + q y = f(x) (p, q real), with
the initial conditions y(xo) = yo , y'1(xo) = y'o or the boundary conditions y(xo) = yo ,
y(x1) = y1 . The general solution of the equation is given as follows. y(x) = yh(x) + yC
where yH(x), is the solution of Homogenous equation yp the particular solution.
2.3 The operator D and the fundamental laws of algebra.
When a particular integral is not obvious by inspection, it is convenient to employ
d
certain methods involving the operator D, which stands for . This operator is also
dx
useful in establishing the form of the complementary function when the auxiliary
equation has equal roots.
2 d2 3 d3
D will be used for , D for , and so on.
dx 2 dx 3
d2y dy 2
The expression 2 5 2y may then be written 2 D y + 5 D y + 2y,
2 dx
dx
2
or (2 D + 5 D + 2)y.
We shall even write this in the factorized form (2D + l)(D +2)y, factorizing the
expression in D as if it were an ordinary algebraic quantity. Is this justifiable ?
The operations performed in ordinary algebra are based upon three laws:
I. The Distributive Law: m (a +b) = m a +m b;
II. The Commutative Law: a b = b a;
m n m+n
III. The Index Law: a . a =a .
Now D satisfies the first and third of these laws, for
m n m+n
D (u + v) = D u + D v, and D . D u = D u (m and n positive integers).
As for the second law, D(c u) =c (D u) is true if c is a constant, but not if c is a
variable.
m n n m
Also D (D u) =D (D u) (m and n positive integers).
Thus D satisfies the fundamental laws of algebra except in that it is not
commutative with variables. In what follows we shall write
n n1
F(D) ao D + a1 D + ... + anl D + an ,
24
Advanced Engineering Mathematics Chapter 2- Second-Order Linear D.E.

where the a's are constants and n is a a positive integer. We are justified in factorizing
this or performing any other operations depending on the fundamental laws of
algebra. For an example of how the commutative law for operators ceases to hold
when negative powers of D occur, see Ex. (iii).
x x
1. F(D) e =e F(a):
x x 2 ax 2 x 3 x 3 x
Since D e = e , D e = e , D e = e , and so on,
x n n1 x
F(D) e = (ao D + a1D + + an1 D + an) e =
n n1 x x
= (ao + a1 + + an1 + an) e = e F().
x x
2. F(D){ e V} =e F(D + ) V, where V is any function of x.
th
By Leibniz's theorem for the n differential coefficient of a product,
n x n x n1 x n2 x 2
D {e V} = (D e ) V + n(D e )(D V) + n(n l)(D e )(D V) + ... +
x n
+ e (D V) =
n x n1 x n2 x 2 x n
= e V + n e D V + n(n1) e D V + + e D V =
x n n1 n2 2 n
= e ( + n D + n(n1) D + +D ) V =
x n
= e (D + ) V.
n1 x x n1
Similarly D {e V} = e (D + ) V and so on.
Therefore
x n n1 x
F(D) {e V} = ( ao D + a1 D + ... + anl D + an ) {e V} =
x n n1 x
= e { ao (D +) + a1 (D +) + ... + anl (D +) + an } V = e F(D + ) V.
2 2
3. F(D ) cos x = F( ) cos x:
2 2 4 2 2
Since D cos x = cos x, D cos x = ( ) cos x , and so on,
2 n 2n2
F (D ) cos x = ( ao D2 + a1 D + ... + anl D2 + an ) cos x
n 2n2
= ( ao D2 + a1 D + ... + anl D2 + an ) cos x =
2 n 2 n1 2 2
= { ao ( ) + a1 ( ) + ... + anl ( ) + an } cos x = F( ) cos x.
2 2
Similarly F ( D ) sin x = F ( ) sin x.

2.4. Complementary Function when the auxiliary equation has equal roots.
When the auxiliary equation has equal roots and , it may be written
2 2
m 2 m + = 0.
d2y
dy
The original differential equation will then be 2 y 0,
2
2 dx
dx
2 2 2
i.e. (D 2 D + ) y = 0, (D ) y =0 (9)
x
We have already found that y =A e is one solution. To find a more general one put
x
y = e V, where V is a function of x.
2 x x 2 x 2
(D ) {e V} = e (D + ) V = e D V.
2 x
Thus equation (9) becomes D V = 0, i.e. V = A + B x, so that y = e (A + B x).
p p
Similarly the equation ( D ) y = 0, reduces to D V = 0, giving
2 p1
V = (A1 + A2 x + A3 x + + Ap1 x ), and
25
Advanced Engineering Mathematics Chapter 2- Second-Order Linear D.E.
x 2 p1
y = e (A1 + A2 x + A3 x + + Ap1 x ).
When there are several repeated roots, as in
p q r
(D ) (D ) (D ) y = 0, (10)
we note that as the operators are commutative we may rewrite the equation in the
q r p
form (D ) (D ) {(D ) y} = 0,
which is therefore satisfied by any solution of the simpler equation
p
(D ) y = 0 (11)
Similarly equation (10) is satisfied by any solution of
q
(D ) y = 0 . (12)
r
or of (D ) y = 0 . (13)
The general solution of (10) is the sum of the general solutions of (11), (12), and (13),
containing together (p + q + r) arbitrary constants.
4 2 2 2
Example 2.4 Solve (D 8 D + 16) y = 0, i.e. (D 4) y = 0.
2 2
The auxiliary equation is (m 4) = 0, m = 2 (twice) or 2 (twice).
2x 2x
Thus by the rule the solution is y= (A+ B x) e + (E + F x) e .
2 2 2 2
Example 2.5: Solve (D + 1) y = 0. The auxiliary equation is (m + 1) =0,
ix ix
m = i (twice) and i (twice). Thus y = (A + B x) e + (E + F x) e ,
or better y = (P +Q x) cos x + (R + S x) sin x.
Examples for solution:
4 3 2 6 4 2
(1) (D +2D +D ) y = 0. (2) (D + 3D + 3D + 1) y = 0,
4 3 2 5 3 2
(3) (D 2D + 2D 2D + l) y = 0. (4) (4D 3D D ) y = 0.
2 2
(5) Show that: F (D )(P cosh ax + Q sinh ax) = F (a )(P cosh ax + Q sinh ax).
4n ax 4n ax
(6) Show that (D a) (e sin p x) = p e : sin p x.
x
2.5. Symbolical methods of finding the Particular Integral when f(x) = e .
The following methods are a development of the idea of treating the operator D as if
it were an ordinary algebraic quantity. We shall proceed tentatively, at first
performing any operations that seem plausible, and then, when a result has been
obtained in this manner, verifying it by direct differentiation. We shall use the
1
notation f (x) to denote a particular integral of the equation F(D)y f (x).
F(D)
x
(i) If f(x) = e , F() 0
x x 1 x
F(D) e =e F() suggests that, as long as F() 0 , e may be a value of
F()
1 x 1 x ex F()
e . This suggestion is easily verified, for F(D) e = e x ,
F(D) F() F()
(ii) If F() = 0, (D ) must be a factor of F(D). Suppose that F(D) = (D )p g(D),
where g() 0. Then the result, F(D){ ex V} =ex F(D + ) V,
suggests that the following may be true, if V is 1,

26
Advanced Engineering Mathematics Chapter 2- Second-Order Linear D.E.

1 x 1 x 1 ex .1 ex 1 e x x p
e = e = = .1 = ,
F(D) (D )p g(D) (D ) p g() g() Dp g() p!
1
adopting the very natural suggestion that is the operator inverse to D, that is the
D
1
operator that integrates with respect to x, while integrates p times. Again the
p
D
result obtained in this tentative manner is easily verified, for
ex x p ex x p x x p
p pe
F(D) = (D ) g(D) = g(D) (D ) =
g( ) p! g( ) p! g( ) p!
e x p e x x
p x
= g(D) D g(D) .1 e
g( ) p! g( )

In working numerical examples it will not be necessary to repeat the verification of
our tentative methods.
2 2x 2 2x
Ex. (i): (D + 3) y = 50 e , Ex. (ii): (D 2) y = 50 e .
1 2x 50e2x
i) The particular integral is .50e 2e2x
2 2
(D 3) (2 3)
2x 3x
Adding the complementary function, we get y = 2e + (A + B x) e

1
ii) If we substitute 2 for D in .50e2x , we get infinity.
(D 2)2
But using the other method,
1 1 1
.50e2x 50e2x .1 50e 2x . x 2 25x 2e 2x .
(D 2)2 D2 2
2 2x 2x
Adding the complementary function, we get y = 25 x e + (A + B x) e

Examples for solution: Solve


2 3x 2 2 2 ax
(1) (D + 6 D + 25) y = l04 e , (2) (D + 2 D + + ) y = e ,
2 3x 3 x x
(3) (D 9) y = 54 e , (4) (D D) y = e + e ,
2 2 3 2 2x
(5) (D ) y = a sinh x, (6) (D + 4D + 4D) y = 8e .
Answers:
x
3x 3x (2) y = e (A cos x + B sin x) +
(1) y=2e +e (A cos 4x+ B sin 4x). ax 2 2
+ e /{(a + ) + }.
3x 3x x x
(3) y = (A + 9x) e + B e . (4) y = A + (B + x)e + (C + x)e .
(5) y = (A + ax/2) cosh x+ B sinh x, (6) y = A + (B + C x 2x2) e2x.

27
Advanced Engineering Mathematics Chapter 2- Second-Order Linear D.E.

2.6. Particular Integral when f (x) = cos ax.


2 2
g (D ) cos ax = g( a ) cos ax.
2 2
This suggests that we may obtain the particular integral by writing a for D
wherever it occurs.
2
Example 2.6: Solve (D + 3D + 2) y = cos 2x.
1 1 1
.cos 2x .cos 2x .cos 2x
D2 3D 2 4 3D 2 3D 2
2 1 3D 2
To get D in the denominator, try the effect of writing ,
3D 2 9D2 4
suggested by the usual method of dealing with surds.
3D 2 1
This gives cos 2x (3Dcos 2x 2cos 2x) =
9D2 4 40
1 1
= (6sin 2x 2cos 2x) (3sin 2x 2cos 2x) .
40 20
3 2
Example 2.7: Solve (D + 6 D + 11D + 6) y = 2 sin 3x.
1 1 1
2sin 3x 2 sin 3x sin 3x
3 2 9D 54 11D 6 D 24
D 6D 11D 6
D 24 D 24 D 24 1
sin 3x sin 3x sin 3x = (D 24)sin 3x =
D 2 576 9 576 585 585
1 1
= [ 3 cos 3x + 24 sin 3x] = [cos 3x + 8 sin 3x].
585 195
We may now show, by direct differentiation, that the results obtained are correct.
Examples for solution: Solve
2
(1) (D + 1) y = 10 sin 2x. (2) (D 5D + 6) y = 100 sin 4x.
(3) (D2+ 8D + 25) y = 48 cos x16 sin x . (4) (D2 +2D+401) y = sin 20x+40 cos 20x.
Answers:
(1) y =2 sin 2x 4 cos 2x + A ex. (2) y = 4 cos 4x 2 sin 4x + A e2x + B e3x.
(3) y = 2 cos x+e4x (A cos 3x + B sin 3x). (4) y = sin 20x+ex (A cos 20x+ B sin 20x).
m
2.7. Particular integral when f(x) = x , where m is a positive integer.
1
In this case the tentative method is to expand in a series of ascending powers
F(D)
of D.
1
Example 2.8: x 2 = 1 (1 1 D 2 )1 x 2 =
4 4
D2 4
1 (1 1 D 2 1 D 4 )x 2 1 (x 2 1 ).
4 4 16 4 2
Hence, adding the complementary function, the solution suggested for
(D + 4) y = x , is y = 1 (x 2 1 ) A cos 2x Bsin 2x.
2 2
4 2

28
Advanced Engineering Mathematics Chapter 2- Second-Order Linear D.E.

1 1 1 1 3
Example 2.9: x3 = x = by partial fractions,
D2 4D 3 2 1 D 3 D

= 1 (1 D D 2 D3 D4 ) 1 (1 1 D 1 D2 1 D3 1 D 4 ) x 3 =
2 3 3 9 27 81
3 9 27 81 243
4

= 1 4 D 13 D2 40 D3 121 D x 3 1 x 3 4 x 2 26 x 80 .
3 3 9 27
Adding the complementary function, the solution suggested for
2 3
(D 4D + 3) y = x
is y 1 x 3 4 x 2 26 x 80 Ae x Be3x .
3 3 9 27
Example 2.10:
1 1 1 2 1 1 2 1
96x 4 96 x 96 (x ), from Ex. (i),
D2 (D2 4) D2 D2 4 D2 4 2

1
96 ( 1 x 4 1 x 2 ) 2 x 4 6 x 2 .
4 12 4
2 2 2
Hence the solution of D ( D + 4) y = 96 x should be
4 2
y = 2x 6x + A cos 2x + B sin 2x + E + F x.
Alternative method .
1 1 96 1
.96x 2 . (1 1 D 2 1 D4 )x 2 =
D2 (D 2 4) D2 4 4 16

= (24D2 6 3 D2 )x 2 2x 4 6x 2 + 3.
2
This gives an extra term 3, which is, however, included in the complementary
function ( in constant E).
1
Note that if D u denotes the simplest form of the integral of u, without any arbitrary
l 1 l
constant, D (D. l) = D . 0 = 0, while D (D . l) = D. x = 1,
l l
so that D (D . l) D (D. l) .
m m n m m n
Similarly D (D . x ) D (D . x ), if m is greater than n.
So when negative powers of D are concerned, the laws of algebra are not
always obeyed. This explains why the two different methods adopted in Example
2.10: give different results.

Examples for solution: Solve


3
(1) (D + l) y=x . (2) (D2 + 2D) y = 24x.
2 4 3 2
(3) (D 6 D + 9) y = 54x + 18. (4) (D 6D + 9 D )y = 54x + l8.
2 2 3 2 2
(5) (D D 2) y = 44 76x 48x . (6) (D D 2D) y = 44 76x 48x .

Answers:
3 2 x 2 2x
(1) y = x 3x + 6x 6 + A e . (2) y = 6x 6x + A + Be .
3x 3 2 3x
(3) y = 6x + 6 + (A + B x) e . (4) y = x + 3x + Ex + F + (A + B x)e .
2 x 2x 3 2 x 2x
(5) y = 24x + 14x 5 + A e + B e . (6) y= 8x + 7x 5x + Ae + Be + C

29
Advanced Engineering Mathematics Chapter 2- Second-Order Linear D.E.

2.8. Particular integrals in other simple cases:


We shall now give some typical examples of the evaluation of particular integrals
in simple cases which have not been dealt with in the preceding articles. The work is
tentative, as before. For the sake of brevity, the verification is omitted, as it is very
similar to the verifications already given.
2
Example 2.11: Solve (D + 4) y = sin 2x.
1 2 2
We cannot evaluate sin 2x by writing 2 for D , for this gives zero in the
D2 4
denominator.
2ix
But i sin 2x is the imaginary part of e , and
1 1
e2ix e2ix .1,
D2 4 (D 2i) 2 4
2ix 1 2ix 1 D 1 2ix 1
D D2
e .1 e (1 ) .1 e (1 2 2 (1)
D(D 4i) 4iD 4i 4iD 4i 4 i
1 x
= e2ix .1 = e2ix = 1 ix( cos 2x + i sin 2x);
4iD 4i 4
hence, picking out the imaginary part,
1
sin 2x = 1 x cos 2x.
2 4
D 4
Adding the complementary function, we get y = A cos 2x + B sin 2x 1 x cos 2x.
4
2 3 2x
Example 2.12: Solve (D 5D + 6) y = x e .
1 1 1 3 2x 1 1 3
.x 3e2x = .x e = e2x .x =
2 2 D 3 D D 1 D
(D 5D 6)
1

= e2x 1 D D 2 D3 D4 x 3 = e2x 1 x 4 x 3 3x 2 6x 6 .
D 4
Adding the complementary function, we get

y = Ae e2x 1 x 4 x 3 3x 2 6x B .
3x
4
2x 2x
including the term 6e in B e .
2 3x
Example 2.13: Solve (D 6 D + 13) y = 8 e sin 2x.
1 1
.8e3x sin 2x = 8e3x .sin 2x =
2 2
(D 6D 13) {(D 3) 6(D 3) 13}
1 1
= 8e3x .sin 2x = 8e3x sin 2x = 8e3x ( 1 x cos 2x)
2 2 4
{D 6D 9 6D 18 13} D 4
3x
= 2 x e cos 2x. (see Ex. (i))
Adding the complementary function, we get
3x
y = e (A cos 2x + B sin 2x 2 x cos 2x).

30
Advanced Engineering Mathematics Chapter 2- Second-Order Linear D.E.

These methods are sufficient to evaluate nearly all the particular integrals that
the student is likely to meet. All other cases may be dealt with on the lines indicated
in (33) and (34) of the miscellaneous examples at the end of this chapter.
Examples for solution:
Solve
2 2x
(l) (D +1) y = 4cosx. (2) (D l) y = (x + 3) e .
3 3 x 2 x
(3) (D 3D 2) y = 540 x e . (4) (D + 2 D +2)y = 2e sin x .
2 2 5 x
(5) (D + 1) y = 24 x cos x. (6) (D D) y = l2e + 8 sin x 2x .
2 3x 3x
(7) (D 6D + 25) y = 2 e cos 4x + 8e (l 2x) sin 4x.
Answers:
x 2x
(1) y = A cos x + (B + 2x) sin x. 2) y =A e + (x + 2) e .
(3) y = Ae2x + (B + Cx 20 x2 20 x3 15x4 9x5)ex. (4) y = {A sin x + (B x) cos x}ex.
(5) y = (A + B x x3) cos x + (E + F x + 3 x2) sin x. (6) y = A + (B + 3x)ex + C ex +x2 + E cos x +(F + 2x) sin x.
2 3x
(7) y = {A sin 4x + (B x + x ) cos 4x}e .

2.9. The Homogeneous Linear Equation:


This is the name given to the form
n n n1 n1
(ao x D + a1 x D + ... + anl D + an ) y = f (x).
t
It reduces to the type considered before if we put x = e .
3 2
3d y 2d y dy
Example 2.14: Solve x 3x x 24x 2 .
3 2 dx
dx dx
t d
Put x = e , t = og x, denote to D
dt
2
dy dy dt 1 dy dy dy 2d y
. = or x Dy x D(D 1)y .
dx dt dx x dt dx dt dx 2
3 d3 y
Similarly x D(D 1)(D 2)y
3
dx
t
D(D1)(D2)y +3D(D1)y + Dy = 24 e2
2t
(D3 3D2 + 2D + 3D2 3D + D) y = 24 e
2t
D3 y = 24 e
3
Auxiliary equation is m = 0 m = 0 , three times (trios)
2
or C.F. = A + B t + C t .
1 2t 24 2t
P.I. = 24 e e 3e2t ,
3 8
D
2 2t 2 2
giving y = A + B t + C t + 3e = A+ B og x+ C (og x) +3x

31
Advanced Engineering Mathematics Chapter 2- Second-Order Linear D.E.

d3 y d2y dy
Example 2.15: Solve x 3 3x 2 x x 3 og x.
dx 3 dx 2 dx
d
Put x = et, t = og x, denote to D
dt
2
dy dy dt 1 dy dy dy 2d y
. = or x Dy x D(D 1)y .
dx dt dx x dt dx dt dx 2
3
3d y
Similarly x D(D 1)(D 2)y
3
dx
D(D1)(D2)y +3D(D1)y + Dy = t e3t
(D 3D + 2D + 3D 3D + D) y = t e3t
3 2 2

D y = t e3t
3

Auxiliary equation is m3 = 0 m = 0 , three times (trios)


or C.F. = A + B t + C t2.
1 1 1 D e3t
P.I. = t e3t e3t t = e3t (1 ) 3 t = (1 D)t =
D3 (D 3)3 27 3 27
e3t x3
= (t 1) (og x 1) .
27 27
Complete solution is
e3t 2 2 x3
y=A+Bt+Ct + (t 1) = A+ B og x+ C (og x) + (og x 1) .
27 27
d2y dy
Example 2.16: Solve x 2 x y x.
dx 2 dx
Solution:
2
2d y dy
x x y x. (1)
2 dx
dx
d
Put x = et, t = og x, denote to D
dt
2
dy dy dt 1 dy dy dy 2d y
. = or x Dy x D(D 1)y .
dx dt dx x dt dx dt 2
dx
3
3d y
Similarly x D(D 1)(D 2)y .
3
dx
The equation (l) becomes
t t t
D(D1)y + Dy + y = e (D2 D + D + 1) y = e (D2 + 1) y = e
Auxiliary equation is m2 + 1 = 0 m = i
or C.F. = (A cos t + B sin t) .

32
Advanced Engineering Mathematics Chapter 2- Second-Order Linear D.E.

1 1 t 1 t
P.I. = et e e .
2 1 1 2
D 1
1
The complete solution is y = (A cos t + B sin t) + e t =
2
x
y = A cos (og x) + B sin (og x) + =
2

Another method is indicated in (28)-(30) of the miscellaneous examples at the


end of this chapter.
The equation
n n n1 n1
ao( + x) D y + a1 ( + x) D y + ... + anl( + x) D y+ an y = f (x).
can be reduced to the homogeneous linear form by putting z = + x, giving
dy dy dz dy
Dy=
dx dz dx dz

Examples for solution: Solve,


2 2
2d y dy 2d y dy
(1) x 2x 2y 4x 3 (2) x 9x 25y 50
2 dx 2 dx
dx dx
3
3d y d2y dy
(3) x 3x 2 x 8y 65 cos (og x)
dx 3 dx 2 dx
4 3 2
4d y 3d y 2d y dy
(4) x 2x x x y og x.
dx 4 dx 3
dx 2 dx
2
2d y dy
(5) (1 2x) 6(1 2x) 16y 8(1 2x) 2
dx 2 dx
2
2d y dy
(6) (1 x) (1 x) y 4cos og(1 x).
2 dx
dx

Answers:
2 3
(1) y = Ax + Bx + 2x .
4 4
(2) y = 2 + A x cos (3 og x) + B x sin (3 og x).
2
(3) y = 8 cos (og x) sin (og x) + A x + B x cos ( 3 og x ).
2 3
(4) y = 4 + og x + A x + B x og x + C x (og x) + D x (og x)
2 2
(5) y = (l +2x) [{og (1 + 2x)} +A og (1 + 2x) + B].
(6) y=A cos {og (1 + x) a}+2 ] og (1 + x) sin og (1 + x).

2.10. Simultaneous linear equations with constant coefficients:


The method will be illustrated by an example. We have two dependent
variables, y and z, and one independent variable x.

33
Advanced Engineering Mathematics Chapter 2- Second-Order Linear D.E.

d
D stands for , as before.
dx
x
Consider (5D + 4) y (2D + 1) z = e , (1)
x
( D + 8) y 3z =5 e (2)
Eliminate z, as in simultaneous linear equations of elementary algebra. To do this we
multiply equation (1) by 3 and operate on equation (2) by (2D + 1 )
Subtracting the results, we get
x x
{3 (5D + 4) (2D + l)(D +8)} y = 3e (2D + 1) 5 e ,
2 x 2 x
i.e. ( 2D 2D + 4) y = 8 e , or (D + D 2) y= 4 e .
x x 2x
Solving this in the usual way, we get y = 2e + A e + B e .
The easiest way to get z in this particular example is to use equation (2), which does
not involve any differential coefficients of z. Substituting for y in (2), we get
x x 2x x
l4 e + 9 A e + 6 B e 3z = 5 e ,
x x 2x
so that z = 3 e + 3A e + 2B e .
However, when the equations do not permit of such a simple method of finding
z, we may eliminate y.
In our case this gives
x x
{ (D + 8)(2D + 1) + 3(5D +4)} y = (D +8) e (5D +4)5 e ,
2 x x x 2x
i.e. ( 2D 2D + 4) z = 12 e , giving z = 3 e + E e + F e .
To find the relation between the four constants A, B, E, and F, substitute in either of
the original equations, say (2). This gives
x x 2x x x 2x x
(D + 8) (2 e + A e + B e ) 3 (3 e + E e + F e ) = 5 e ,
x 2x
i.e. (9A 3E) e + (6B 3F) e = 0, whence 9A 3E = 6B 3F = 0
i.e. E = 3A and F = 2B,
x x 2x x x 2x
so z = 3 e + E e + F e = 3 e + 3A e + 2B e , as before.

Examples for solution: Solve,


(1) D y z = 0 , (D l) y (D + l)z = 0.
(2) (D 17) y + (2D 8) z = 0, (13D 53) y 2z = 0 .
2 2 2 2
(3) (2D D + 9) y (D + D + 3) z = 0, (2D + D +7) y (D D + 5) z = 0.
x x
(4) (D + l)y = z + e , (D + l) z = y + e
2 2
(5) (D + 5)y 4z = 36 cos 7x, y + D z = 99 cos 7x .
x
(6) (2D + l) y + (D + 32)z = 91e +147 sin 2x + 135 cos 2x,
x
y (D 8) z = 29 e + 47 sin 2x + 23 cos 2x.
Answers:
(1) y = A cos (x ); z = A sin (x ).
5x 3x 5x 3x
(2) y = A e + B e ; z = 6 A e 7 B e
x x
(3) y = A e + B cos (2x ); z = 2 A e B cos (2x ).
x 2x x 2x
(4) y = e +A + Be ; z = e + A Be .
(5) y = A cos (x ) + 4 B cos (2x ) + cos 7x;
z = A cos (x ) + B cos (2x ) 2 cos 7x.

34
Advanced Engineering Mathematics Chapter 2- Second-Order Linear D.E.
3x 4x x
(6) y = 5Ae 4B e + 2e + cos 2x sin 2x ;
3x 4x x
z = A e + B e + 3e + 4 cos 2x + 5 sin 2x.
MISCELLANEOUS EXAMPLES ON CHAPTER 2. Solve:
3 3x
(1) (D 1) y = 16 e
2 5x/2
(2) (4D + 12D + 9) y = l44x e .
4 3 2 2x
(3) (D + 6D + 11D + 6D) y = 20e sin x.
3 x
(4) (D D2 + 4D 4) y = 68 e sin 2x.
4 2 3x
(5) (D 6D 8D 3) y = 256 (x + l) e .
4 2
(6) (D 8D 9) y =50 sinh 2x.
4 2
(7) (D 2D + 1) y = 40 cosh x.
2 2 2x
(8) (D 2) y = 8(x + e + sin2x).
2 2 2x
(9) (D 2) y = 8 x e sin2x .
2 2 3
(10) (D + 1) y = 3 cos x+2 sin x .
4 2
(11) (D + 10D + 9) y= 96 sin 2x cos x .
n x
(12) (D n) y = n , where n is a positive integer .
d2y 1 dy 12og x
(l3) .
dx 2 x dx x2
d2y 2 dy d3y 6y
(l4) 10. (15) .
dx 2 x dx dx 3 x 3
d2y dy
(16) (x 1) 2 (x 1) (2x 3)(2x 4) .
dx 2 dx
d2x dx d2y dy
(l7) 4 4x y, 4 4y 25x 16e t .
dt 2 dt dt 2 dt
dx dy dz
(18) 2y ; 2z. ; 2x ;
dt dt dt
dx dy
(19) t y 0; t x 0 .
dt dt
2 2
2d y dy 2d x dx
(20) t t 2x 0 , t t 2y 0 .
dt 2 dt dt 2 dt
(21) Prove that if V is a function of x and F(D) has its usual meaning,
n n n1
(i) D [x V] = x D V + n D V;
(ii) F(D) [x V] = x F(D) V + F'(D) V;
1 1 F'(D)
(iii) x V x V V.
F(D) F(D) [F(D)]2
n n nl n nr (r)
(iv) g(D) [x V] = x g(D) V + n x g'(D) V + + Cr x g (D) V+to (n + 1)
1
terms, where g(D) stands for .
F(D)
2x
(22) Obtain the Particular Integrals of (i) (D 1) y = xe ,
35
Advanced Engineering Mathematics Chapter 2- Second-Order Linear D.E.
2
(ii) (D + 1) y = x cos x, by using the results (iii) and (iv) of the last example.
Answers:
2 x 3x 3 3x/2
(1) y = (A+ B x+ C x ) e + 2e . (2) y = (A + B x+ 6 x ) e .
3x 2x x 2x
(3) y = A e + B e + C e + E + 2e (sin x 2 cos x).
x x
(4) y =Ae + B cos (2x ) 2e (4 sin 2x + cos 2x).
2 x 2 3x
(5) y = (A + B x + C x ) e + (E + x + 2x )e .
(6) y = A sin (x ) + B sinh (3x ) 2 sinh 2x.
2
(7) y = (A + B x + 5 x ) cosh x+ (E + F x) sinh x.
2 2 2x
(8) y = 3 + 4 x + 2 x + (A + B x + 4x ) e + cos 2x.
2 2x
(9) y = (A+ B x+ 3 sin 2x 4x cos 2x 2x sin 2x) e .
(10) y = A cos (x ) + 3 1 cos 2x 3 x cos x + 1 sin 3x.
2 2 4 16
(11) y =A cos (x ) + B cos (3x ) 3x cos x + x cos 3x.
2 n1 nx x n
(12) y = (Ao + A1 x +A2 x + + An1 x ) e + n /(og n n) .
3 1 2
(13) y =A + B og x + 2(og x) . (14) y =A + B x + 5 x .
8
3
(15) y = A x + B cos ( 2 og x ).
2 2
(16) y = A + B og (x + 1) +{og (x + 1)} + x + 8 x.
3t 3t t
(17) x = Ae + Be + E cos t + F sin t e ;
3t 3t t
y = Ae + 25e + (3E 4F) cos t + (3F + 4E) sin t e .
2t t
(18) x = A e + B e cos ( 3 t ) ;
2t t
y = A e + B e cos ( 3 t + 2/3) ;
2t t
z = A e + B e cos ( 3 t + 4/3)
1 1
(19) x = A t + B t ; y = A t + B t .
1
(20) x = A t cos (og t ) + B t cos (og t ) ;
1
y = A t sin (og t ) B t sin (og t ).
2x 2 2
(22) (i) (x l) e ; (ii) (x 2x + l) sin x + (x l) cos x.

2.11. Rules to find particular integral :


(i) 1 ex 1 ex ,f () 0.
f (D) f ()
If f() = 0 , the above rule fails, then we apply
If f() = 0 , then 1 e x x 1 e x , f() 0, f '() 0.
f (D) f '( )
If f() = 0 , f '() = 0 the above rule fails, then we apply
If f() = 0 , f '() = 0, then 1 e x x 2 1 e x ,
f (D) f ''( )

1 x n f (D) 1 , Expand [f(D)]1 and then operate.


(ii)
f (D)

36
Advanced Engineering Mathematics Chapter 2- Second-Order Linear D.E.

(iii) 1 sin x 1 sin x,


2
f (D ) f ( 2 )
1 cos x 1 cos x,
2
f (D ) f ( 2 )
sin x, f( 2 ) 0, f '( 2 ) 0.
2 1 sin x x 1
If f( ) = 0 , then 2 2
f (D ) f '( )
cos x, f( 2 ) 0, f '( 2 ) 0.
2 1 cos x x 1
If f( ) = 0 , then 2 2
f (D ) f '( )

(iv) 1 ex .(x) ex 1 (x).


f (D) f (D )

1 (x) e x ex (x)dx.
(v)
D

3 3 2 2 2
Example 2.17: Solve (x D + 3x D + x D) y = 24 x
t dx t
Put x = e , =e =x,
dt
d dt d 1 d
so that D = = = ;
dx dx dt x dt
2 1 d 1 d 1 d 1 d d2
D =D = 2 D 2 2 ;
x dt x dt x dt x dt dt

3
1 d d2 2 d d2 1 d d2
D = D = 3 2 + 2 D 2 =
x 2 dt dt 2 x dt dt x dt dt

2 d d 2 1 d 2 d3 1 d d 2 d3
= + = 2 3 2 3 ;
x 3 dt dt 2 x 3 dt 2 dt 3 x 3 dt dt dt
d3 y 2t
thus the given differential equation reduces to = 24e ,
dt 3
2 2t 2 2
giving y = A + B t + C t + 3e = A+ B og x+ C (og x) +3x

Solution of the differential equation, consisting of two parts i.e. ( A u + B v +


W). The first part is the solution of the differential equation whose R.H.S. is zero.
A u + B v is known as complementary function C.F.. Second parts W is free from
any arbitrary constant and is known as particular integral.
Complete Solution = Complementary Function + Particular Integral
or y = C.F. + P.I.

37
Advanced Engineering Mathematics Chapter 2- Second-Order Linear D.E.

2.12. Method for finding the complementary function.


d2y
dy
Let the given equation is P
Qy F(x)
2 dx
dx
(l) In finding the complementary function, R.H.S. of the given equation is replaced by
d2y dy
zero. i.e. P Qy 0 .
dx 2 dx
mx d2y dy
(2) Let y = A e be the complementary function of P Qy 0 (1)
dx 2 dx
dy d2y mx 2
Putting the values of y, and in (1) , then A e ( m + P m + Q ) = 0
dx 2
dx
2
or m + P m + Q = 0. It is called Auxiliary equation.
(3) Solve the auxiliary equation, there are three cases:
Case I : Roots, Real and Different. If m and n are the roots, then the C.F. is
mx nx
y=Ae +Be .
Case II : Roots, Real and Equal. If both roots are m = n, then C.F. is
mx
y = (A + B x)e .
Case III : Roots Imaginary. If the roots are i, then the solution will be
( + i)x (i)x x
y=Ae +Be or y = e [A' cos x + B' sin x],
where A' = (A + B) and B' = i(A B).
d2y
dy
Example 2.18: Solve 8
15y 0
2 dx
dx
2
Solution: {D 8 D + l5}y = 0,
2
Auxiliary equation is m 8 m + 15 = 0 or m = 3, 5
3x 5x
C.F. = A e + B e x,
d2y
dy
Example 2.19: Solve 6
9y 0
dx 2 dx
2
Solution: Given equation can be written as : (D 6D + 9) y = 0
2 2
A.E. is m 6 m + 9 = 0 or (m 3) = 0 or m = 3, 3
3x
Hence the required solution is y ={A + B x) e .
d2y
dy
Example 2.20: Solve 4
5y 0
dx 2 dx
2
Solution: Here the auxiliary equation is m + 4m + 5 = 0
2x
Its root are 2 i. The complementary function is e (A cos x + B sin x).
d2y dy
Example 2.2: Solve 6 9y 5e3x
dx 2 dx
Solution:
2 3x
Auxiliary equation is m + 6 m + 9 = 0 or m = 3 twice C.F. = (A + B x) e

38
Advanced Engineering Mathematics Chapter 2- Second-Order Linear D.E.

1 e3x e3x
P.I. = 5.e3x = 5 .= 5 .
D 2 6D 9 (3)2 6(3) 9 36
3x e3x
The complete solution is y = (A + B x) e +5 .
36
d2y dy
Example 2.22: Solve 2a a 2 y eax .
dx 2 dx
Solution:
2 ax
Auxiliary equation is m 2a m + a2 = 0 or m = a twice C.F. = (A + B x) e
1 1
P.I. = eax = eax Since the denominator equal zero,
2 2 2
D 2aD a (D a)
1
differentiating the denominator P.I. = x eax Since the denominator equal
2(D a)
2
zero, differentiating the denominator, again differentiating P.I. = x eax .
2
2
Complete solution is y = (A + B x) e + x eax .
ax
2
d2y dy
Example 2.23: Solve 7
12y x
dx 2 dx
2
Solution: {D 7 D + l2}y = x,
2
Auxiliary equation is m 7 m + 12 = 0 or m = 3, 4
3x 4x
C.F. = A e + B e x,
1 1 1 1 2 1
P.I. = .x = .x = 7
1 D D x=
D 2 7D 12 12 1 7 D D2 12 12 12
12 12
1 1
1 7 D D x (x 7 ) .
2
=
12 12 12 12 12
3x 4x 1
Complete solution is y = A e + B e x + (x 7 ) .
12 12
2
Example 2.24: Solve (D + 4) y = sin 3x.
2
Solution: (D + 4) y = sin 3x
2
Auxiliary equation is m + 4 = 0 or m = 2i C.F. = A cos 2x + B sin 2x
1 sin 3x 1
P. I. = sin 3x = = sin 3x
D2 4 (32 ) 4 5
1
Complete solution is y = A cos 2x + B sin 2x + sin 3x
5
d2y dy
Example 2.25: Solve y cos 2x
dx 2 dx

39
Advanced Engineering Mathematics Chapter 2- Second-Order Linear D.E.

2 1 3 1 i 3
Solution: Auxiliary equation is m + m + 1 = 0 m =
2 2
x
or C.F. = e A cos 3 x Bcos 3 x
2
2 2
1 1 1 D3
P. I. = cos 2 x = cos 2 x = cos 2 x = cos 2 x =
2 2 D 3 2
D D 1 (2 ) D 1 D 9
D3 D3
= cos 2 x = cos 2 x = 1 (D 3)cos 2x = 1 (2sin 2 x 3cos 2x)
2 2 13 13
D 9 (2 ) 9
x
Complete solution is y = e A cos 3 x Bcos 3 x + 1 (2sin 2 x 3cos 2x) .
2
2 2 13
2
Example 2.26: Solve (D + 4) y = cos 2x.
2
Solution: (D + 4) y = cos 2x
2
Auxiliary equation is m + 4 = 0 or m = 2i
C.F. = A cos 2x + B sin 2x
1 1
P. I. = cos 2x = x cos 2x = x ( 1 sin 2x) = x sin 2x
2 2D 2 2 4
D 4
Complete solution is y = A cos 2x + B sin 2x + x sin 2x .
4
d2y dy
Example 2.27: Solve 2 5y e2x sin x
dx 2 dx
Solution:
2 2 16
Auxiliary equation is m 2 m + 5 = 0 m= 1 2i
2
or C.F. = e x A cos 2x Bcos 2x
1 1
P. I. = e2x sinx = e2x sinx
D2 2D 5 (D 2)2 2(D 2) 5
1 1 1
= e2x sinx = e2x sinx = e2x sinx =
D2 4D 4 2D 4 5 D2 2D 5 1 2D 5
1 1 D2
= e2x sinx = 1 e2x sinx = 1 e2x sinx =
2D 4 2 D2 2 2
D 4
D2
= 1 e2x sinx = 1 e2x (D 2)sinx = 1 e (2 sin x cos x).
2x
2 1 4 10 10
Complete solution is y = e x A cos 2x Bcos 2x + 1 e (2 sin x cos x) .
2x
10
2
d y dy
Example 2.28: Solve 4 3y 2xe3x 3e x cos x
dx 2 dx
Solution:
2
Auxiliary equation is m 4 m + 3 = 0 m = 1 , 3

40
Advanced Engineering Mathematics Chapter 2- Second-Order Linear D.E.
x 3x
or C.F. = A e + B e .
1 1
P. I. = 2 xe3x + 2e x cos 2 x =
D 2 4D 3 D2 4D 3
1 1
= 2e3x .x + 3e x .cos x
2 2
(D 3) 4(D 3) 3 (D 1) 4(D 1) 3
1 1
= 2e3x .x + 3e x .cos x =
D2 6D 9 4D 12 3 D2 2D 1 4D 4 3
1 1
= 2e3x .x + 3e x .cos x =
D2 2D D2 2D
1 1
= 2e3x .x + 3e x .cos x =
2D(1 D / 2) 4 2D
1 D 1
= e3x (1 )1.x 3 e x .cos x =
D 2 2 2D
3x 1 D D2 2D
=e (1 ).x 3 e x .cos x =
2
D 2 4 4 D2
1 1 D 2D
= e3x ( ).x 3 e x .cos x =
D 2 4 2 44
2
3x x x 1
= e ( ) 3 e x (2cos 2x 2sin 2x) .
2 2 4 16
The complete solution is
2
x 3x 3x x x 1 3 x
y=Ae +Be + e ( ) e (cos 2x sin 2x)
2 2 4 8
2
3x x x
) 3 e x (cos 2x sin 2x)
x 3x
y=Ae +Ce + e (
2 2 8
e3x 3x
The term has been omitted from the P.I. , since B e is present in the C.F.
4
d2y dy
Example 2.29: Solve 6 13y 8e3x sin 4x 2 x.
dx 2 dx
Solution:
2
Its auxiliary equation is m 6 m + 13 = 0 m = 3 2i
3x
or C.F. = e (A cos 2x + B sin 2x).
1 8e3x sin 4x 2 x =
P. I. =
D 6D 13
2
1 8e3x sin 4x + 1 e x og2
=
D2 6D 13 D2 6D 13
Replacing D by D+3 in first part and by og 2 in second part we get

41
Advanced Engineering Mathematics Chapter 2- Second-Order Linear D.E.

1 1
P. I. = 8e3x sin 4x + e x og2 =
2 2
(D 3) 6(D 3) 13 (og2) 6(og2) 13
1 1
= 8e3x sin 4x + e x og2
D 2 6D 9 6D 18 13 (og2)2 6(og2) 13
1 1
= 8e3x sin 4x + e x og2 =
D2 4 (og2)2 6(og2) 13
1 1
= 8e3x sin 4x + e x og2
16 4 (og2) 2 6(og2) 13
2 2x
= e3x sin 4x + .
3 2
(og2) 6(og2) 13
The complete solution is
3x 2 3x 2x
y = e (A cos 2x + B sin 2x) e sin 4x + .
3 (og2) 2 6(og2) 13
d2y dy
Example 2.30: Solve 2 y x sin x.
dx 2 dx
Solution:
2
Auxiliary equation is m 2 m + 1 = 0 m = 1 twice
x
or C.F. = (A + B x) e .
1
P. I. =
2
x sin x =
D 2D 1
1
= Imaginary part of x cos x isin x =
D2 2D 1
1
= Imaginary part of x.eix =
2
D 2D 1
1
= Imaginary part of eix x
(D i)2 2(D i) 1
1
= Imaginary part of eix x=
D2 2iD 1 2D 2i 1
1
= Imaginary part of eix x
2
D 2(1 i)D 2i
1
ix 1 1 2
= Imaginary part of e 1 (1 i)D D .x =
2i 2i
i
= Imaginary part of (cos x isin x) 1 (1 i)D .x
2
1
= Imaginary part of (i cos x sin x) x 1 i = 1 x cos x 1 cos x 1 sin x
2 2 2 2

42
Advanced Engineering Mathematics Chapter 2- Second-Order Linear D.E.

The complete solution is


x
y = (A + B x) e + 1 x cos x 1 cos x 1 sin x .
2 2 2
2
d y dy
Example 2.31: Solve 2 y xe x sin x.
dx 2 dx
Solution:
2
Auxiliary equation is m 2 m + 1 = 0 m = 1 twice
x
or C.F. = (A + B x) e .
1 xe x sin x = 1 xe x sin x = e x 1 x sin x =
P. I. =
D2 2D 1 (D 1)2 D2
1
= e x x cos sin x Note that 1/D =
D
x x
= e [ x sin x x cos x cos x] = e [ x sin x + 2 cos x].
x x
The complete solution is y = (A + B x) e e [ x sin x + 2 cos x].
2.13 General method of finding the particular integral of any function (x).
1
P.I. = (x) = y
Da
1
or (D a) (x) = (D a) y (x) = (D a) y or (x) = D y ay.
Da
dy
i.e. ay (x) which is the linear differential equation.
dx
adx a dx
Its solution is y e = e
.(x)dx
= eax .(x)dx y = e eax .(x)dx
ax ax
ye
1
(x) = e eax .(x)dx
ax
Then y =
Da
d2y
Example 2.32: Solve 9y sec3x.
dx 2
Solution:
2
Auxiliary equation is m + 9 = 0 m = 3i
or C.F. = (A cos 3x + B sin 3x) .
1 1 1 1 1
P. I. = sec3x = sec3x = sec3x =
D2 9 (D 3i)(D 3i) 6i D 3i D 3i
1 1 1 1
= sec3x sec3x
6i D 3i 6i D 3i
Now
1 cos3x isin 3x
sec3x e3ix e 3ix sec3xdx = e3ix dx = e3ix [1 i tan 3x]dx =
D 3i cos3x
= e3ix (x i og cos3x) ,
3

43
Advanced Engineering Mathematics Chapter 2- Second-Order Linear D.E.

Change i to i, we have
1
sec3x = e3ix (x i og cos3x ) , then
D 3i 3

P. I. = 1 [ e3ix (x i og cos3x) e3ix (x i og cos3x ) ] =


6i 3 3
e e3ix 1 e e3ix
3ix 3ix
= x og 3x = x sin 3x + 1 cos 3x .og cos 3x
3 2i 18 2 3 9
Hence complete solution is
y = (A cos 3x + B sin 3x) + x sin 3x + 1 cos 3x .og cos 3x.
3 9
2
d y dy
Example 2.33: Solve x 2 x y x.
dx 2 dx
Solution:
2
2d y dy
x x y x. (1)
2 dx
dx
t d
Put x = e , t = og x, denote to D
dt
2
dy dy dt 1 dy dy dy 2d y
. = or x Dy x D(D 1)y .
dx dt dx x dt dx dt dx 2
3
3d y
Similarly x D(D 1)(D 2)y .
3
dx
The equation (l) becomes
t 2 t 2 t
D(D1)y + Dy + y = e (D D + D + 1) y = e (D + 1) y = e
2
Auxiliary equation is m + 1 = 0 m = i
or C.F. = (A cos t + B sin t) .
1 1 t 1 t
P.I. = et e e .
2 11 2
D 1
1
The complete solution is y = (A cos t + B sin t) + e t =
2
x
y = A cos (og x) + B sin (og x) + =
2
3 2
3d y 2d y dy
Example 2.34: Solve x 3x x x 3 og x.
dx 3 dx 2 dx
t d
Put x = e , t = og x, denote to D
dt
2
dy dy dt 1 dy dy dy 2d y
. = or x Dy x D(D 1)y .
dx dt dx x dt dx dt dx 2

44
Advanced Engineering Mathematics Chapter 2- Second-Order Linear D.E.

3
3d y
Similarly x D(D 1)(D 2)y
3
dx

D(D1)(D2)y +3D(D1)y + Dy = t e3t


(D 3D + 2D + 3D 3D + D) y = t e3t
3 2 2

D y = t e3t
3
3
Auxiliary equation is m = 0 m = 0 , three times (trio)
2
or C.F. = A + B t + C t .
1 3t 3t 1 3t 1
D 3 e3t
P.I. = te e t =e (1 ) t = (1 D)t =
D3 (D 3)3 27 3 27
e3t x3
= (t 1) (og x 1) .
27 27
Complete solution is
e3t 2 2 x
3
y=A+Bt+Ct + (t 1) = A+ B og x+ C (og x) + (og x 1) .
27 27
2
2d y dy
Example 2.35: Solve (1 x) (1 x) y sin 2[ og(1 x)]
2 dx
dx
t d
Put 1 + x = e , t = og (x + 1), denote to (1 x) D ,
dt
2
dy 2d y
(1 x) Dy (1 x) D(D 1)y
dx 2
dx
3
3d y
Similarly (1 x) 3
D(D 1)(D 2)y
dx
Putting these values in the given differential equation we get
2
D(D 1)y + Dy + y = sin 2t, (D D + D + 1)y = sin 2t,
2
(D + 1)y = sin 2t
2
Auxiliary equation is m + 1 = 0 m = i
C.F. = (A cos t + B sin t)
1 1 1
P.I. = sin 2t sin 2t sin 2t
D2 1 4 1 3
1
The complete solution is y = (A cos t + B sin t) sin 2t
3
1
= A cos og (x + 1) + B sin og (x + 1) sin 2og x 1 .
3

45
Advanced Engineering Mathematics Chapter 2- Second-Order Linear D.E.

2.14 The Method of Undetermined Coefficients


A particular solution Y of the nonhomogeneous nth order linear equation with
constant coefficients
n n1
L[y] = (ao D + a1 D + ... + anl D + an ) y = f (x). .................(1)
can be obtained by the method of undetermined coefficients, provided that f(t) is of
an appropriate form. While the method of undetermined coefficients is not as general
as the method of variation of parameters, it is usually much easier to use when
applicable.
Just as for the second order linear equation, when the constant coefficient
m m1
linear differential operator L is applied to a polynomial ao t + a1t ++ am, an
t
exponential function e , a sine function sin t, or a cosine function cos t, the
result is a polynomial, an exponential function, or a linear combination of sine and
cosine functions, respectively. Hence, if f(t) is a sum of polynomials, exponentials,
sines, and cosines, or products of such functions, we can expect that it is possible to
find g (t) by choosing a suitable combination of polynomials, exponentials, and so
forth, multiplied by a number of undetermined constants. The constants are then
determined so that Eq. (1) is satisfied.
The main difference in using this method for higher order equations stems
from the fact that roots of the characteristic polynomial equation may have
multiplicity greater than 2. Consequently, terms proposed for the nonhomogeneous
part of the solution may need to be multiplied by higher powers of t to make them
different from terms in the solution of the corresponding homogeneous equation.
The Method of Undetermined Coefficients
Simple Form of the Method
The method of undetermined coefficients is applicable only if f(x) and all
of its derivatives can be written in terms of the same finite set of linearly
independent functions, which we denote by {y1(x), y2(x),..., yn(x)}. The
method is initiated by assuming a particular solution of the form
yp(x) = A1 y1(x) + A2 y2 (x) + + An yn (x),
where A1, A2,, An denote arbitrary multiplicative constants. These arbitrary
constants are then evaluated by substituting the proposed solution into the given
differential equation and equating the coefficients of like terms.
th
Case 1: f(x) = pn(x), an n degree polynomial in x. Assume a solution
of the form
n n1
yp = (An x + An1 x ++ A1 x + Ao) ...............................................(2)
where Ai ( i = 0,1,2,, n) is a constant to be determined.
ax
Case 2: f(x) = k e where k and a are known constants. Assume a solution of the
form
ax
yp = A e ................................ (3)
where A is a constant to be determined.
Case 3: f(x) = k1 sin x + k2 cos x where k1, k2, and are known constants. Assume
a solution of the form yp = A sin x + B cos x... (4)
where A and B are constants to be determined.
46
Advanced Engineering Mathematics Chapter 2- Second-Order Linear D.E.

Dont Forget
yp = A sin x + B cos x in its entirety is assumed for
f(x) = k1 sin x + k2 cos x even when k1 or k2 is zero, because the derivatives of
sines or cosines involve both sines and cosines.
Generalizations
If f(x) is the product of terms considered in Cases 1 through 3, take yp to
be the product of the corresponding assumed solutions and algebraically
combine arbitrary constants where possible. In particular, if
ax
f(x) = e pn(x) is the product of a polynomial with an exponential, assume
x n n1
yp = e (An x + An1 x ++ A1 x + Ao)........................................ (5)
x
where Ai is as in Case 1. If, instead, f(x) = e pn(x) sin x is the product
x
of a polynomial, exponential, and sine term, or if f(x) = e pn(x) cos x
is the product of a polynomial, exponential, and cosine term, then assume
x n n1
yp = e sin x (An x + An1 x ++ A1 x + Ao) +
x n n1
+ e cos x (Bn x + Bn1 x ++ B1 x + Bo)..................(6)
where Ai and Bi ( i = 0,1,2,, n) are constants which still must be determined.
If f(x) is the sum (or difference) of terms already considered, then we take yp to
be the sum (or difference) of the corresponding assumed solutions and algebraically
combine arbitrary constants where possible.
Modifications
If any term of the assumed solution, disregarding multiplicative constants, is also a
term of yh (the homogeneous solution), then the assumed solution must be modified
m
by multiplying it by x , where m is the smallest positive integer such that the product
m
of x with the assumed solution has no terms in common with yh.
Limitations of the Method
In general, if f(x) is not one of the types of functions considered above, or if the
differential equation does not have constant coefficients, then the following method is
preferred. Variation of parameters is another method for finding a particular solution
th
of the n order linear differential equation
d2y dy
Example 2.36: Solve 6 9y 5e3x (1)
2 dx
dx
Solution:
2 3x
Auxiliary equation is m + 6 m + 9 = 0 or m = 3 twice C.F. = (A + B x) e
3x
To find a particular solution Y (t) of Eq. (1), we start by assuming that Y (x) = C e .
3x
Thus our final assumption is that Y (t) = C e , where C is an undetermined
coefficient. To find the correct value for C, we differentiate Y (x) two times,
substitute for y and its derivatives in Eq. (1), and collect terms in the resulting
3x 3x
equation. In this way we obtain [9C +(6).(3)C + (9)C] e 5 e
5 e3x
i.e. 36 C = 5 C = . P.I. = 5 .
36 36
47
Advanced Engineering Mathematics Chapter 2- Second-Order Linear D.E.

3x e3x
The complete solution is y = (A + B x) e +5 .
36
Example 2.37:
d2y dy
Solve 7 12y x .................................(1)
dx 2 dx
2
Solution: (D 7 D + l2) y = x,
2
Auxiliary equation is m 7 m + 12 = 0 or m = 3, 4
3x 4x
C.F. = A e + B e x,
To find a particular solution Y (t) of Eq. (1), we start by assuming that
Y (x) = + x, where and are undetermined coefficients. To find the correct
value for and , we differentiate Y (x) two times, substitute for y and its derivatives
in Eq. (1), and collect terms in the resulting equation. In this way we obtain
[7 + 12 + 12 x] x must be true for all x, the constant terms must sum to zero
or 12 7 = 0 , 12 = 1. i.e. = 1/12 and 12 = 7 =7/12 = 7/144
Similarly, all of the terms involving the polynomial x must balance, yielding = 7 ,
144
and = 1 .
12
1
Therefore, the particular solution is P.I. = (x 7 ) .
12 12
3x 4x 1
Complete solution is y = A e + B e x + (x 7 ) .
12 12

d3 y d2y dy
Example 2.38: Find the general solution of 3 3 y 4e x ......(1)
dx 3 dx 2 dx
3 2 3
Auxiliary equation is m 3 m + 3m 1= 0 or (m 1) = 0 , m = 1 trio (three times)
x x 2 x
C.F. = A e + B x e + C x e , ...(2)
x
To find a particular solution Y (x) of Eq. (1), we start by assuming that Y (x) = K e .
x x 2 x
However, since e , x e , and x e are all solutions of the homogeneous equation, we
3
must multiply this initial choice by x . Thus our final assumption is that
3 x
Y (x) = Kx e , where K is an undetermined coefficient. To find the correct value for
K, we differentiate Y (t) three times, substitute for y and its derivatives in Eq. (1), and
x x
collect terms in the resulting equation. In this way we obtain 6 K e 4 e . Thus
3 x
K = 2 and the particular solution is Y (x) = 2 x e . (3)
3 3
The general solution of Eq. (1) is the sum of C.F. from Eq. (2) and Y (t) from Eq.
x x 2 x 3 x
(3). y = A e + B x e + C x e + 2 x e ,
3
dy d2y
Example 2.39: Find the general solution of y x sin x (1)
2
dx 2 dx
2 2
Auxiliary equation is m 2 m + 1= 0 or (m 1) = 0 , m = 1 twice
x x
C.F. = A e + B x e ...(2)

48
Advanced Engineering Mathematics Chapter 2- Second-Order Linear D.E.

From the form of the right side of (1), we guess the particular solution
yp(x) = x + + sin x + cos x, (3)
Therefore, y'p(x) = + cos x sin x, ...(4)
and y"p(x) = sin x cos x, ...(5)
y"p(x) 2 y'p(x) + yp(x) = sin x cos x 2( + cos x sin x) +
+ ( x + + sin x + cos x) x + sin x... (6)
sin x cos x 2( + cos x sin x) + ( x + + sin x + cos x) x + sin x
i.e. 2 2 cos x +2 sin x + x x + sin x
coefficient of x : = 1, coefficient of sin x: 2 = 1 =
coefficient of cos x: 2 = 0 = 0 , free term: 2 + = 0 = 2 = 2, then
Since (6) must be true for all x, the constant terms must sum to zero or 2 = 0.
Similarly, all of the terms involving the polynomial x must balance, yielding =1
and = 2 = 2. Turning to the trigonometric terms, the coefficients of sin x and cos x
give 2 = 1 and 2 = 0, respectively.
Therefore, the particular solution is yp(x) = x + 2 + + cos x ....(7)
x x
and the general solution is y = A e + B x e + x + 2 + + cos x
Why does this technique work? The reason lies in the set of functions that we
have allowed to be included in f(x). They enjoy the remarkable property that
derivatives of their sums and products yield sums and products that are also
constants, polynomials, exponentials, sines, and cosines. Because a linear
combination of derivatives such as a y"p b y p' c y p must equal f(x), it seems
reasonable to assume that y p has the same form as f(x). The following examples
show that our conjecture is correct.
Example 2.40: Find the general solution of
d2y dy
4 4y 2e2x 4x 12. ............................................................................ (1)
dx 2 dx
2 2
Auxiliary equation is m 4 m + 4 = 0 or (m 2) = 0 , m = 2 twice
2x 2x
C.F. = A e + B x e , ......(2)
To find a particular solution Y (x) of Eq. (1), we start by assuming that
2 2x
Y (x) = x e + x + , where , and are undetermined coefficients. To find
the correct values for , and we differentiate Y (t) two times, substitute for y and
its derivatives in Eq. (1), and collect terms in the resulting equation. In this way we
obtain
2 2x 2x 2 2x 2
Y (x) = x e + x + , Y' (x) = 2e [x + x] + , Y" (x) = 2e [ 2x + 4 x +1 ]
2x 2 2x 2 2 2x
i.e. 2e [ 2x + 4 x +1 ]4[2e [x + x] + ] +4 [ x e + x + ]
2x
2 e + 4 x 12,
2x 2 2 2 2x
e [4 x 8x +4 x +8 x4x + 2] + 4x + 4 2 e + 4 x 12
2x 2 2 2 2x
e [4 x 8x +4 x +8 x4x + 2] + 4x + 4 2 e + 4 x 12
2x 2x
e [4x + 2] + 4x + 4 2 e + 4 x 12
Then, if we take = 1, = 1, = 3

49
Advanced Engineering Mathematics Chapter 2- Second-Order Linear D.E.
2 2x 2x
The particular solution is Y (x) = x e + x e + x 3 (3)
2 2x
i.e. Y (x) = x e + x 3 (4)
2x 2x
The term x e has been omitted from the P.I. , since B x e is present in the C.F.
Example 2.41: Let us find the particular solution to
d2y dy
2y xe x . .....(1)
2 dx
dx
by the method of undetermined coefficients.
From the form of the right side of (1), we guess the particular solution
x x
Y(x) Yp(x) = e + x e ............................................(2)
x x x x x x x x
Y(x) = e + x e , Y'(x) = e + x e + e , and Y"(x)= e + x e + 2 e .
x x
Substituting into (1), we find that 3 e = x e .....................................................(3)
Clearly we cannot choose a constant such that (2) is satisfied. What went wrong?
To understand why, let us find the homogeneous or complementary solution to (1);
2x x 2
it is yH(x) = A e + B e , Since its auxiliary equation is m + m 2 = 0
(m 1)(m + 2) = 0
x
Therefore, one of the assumed particular solutions, e , is also a homogeneous
solution and cannot possibly give a nonzero left side when substituted into the
differential equation. Consequently, it would appear that the method of undetermined
coefficients does not work when one of the terms on the right side is also a
homogeneous solution. Before we give up, let us recall that we had a similar situation
in the case of linear homogeneous second-order ordinary differential equations when
the roots from the auxiliary equation were equal. There we found one of the
homogeneous solution was We eventually found that the second solution was Could
such a solution work here? Let us try.
We begin by modifying (3.4.16) by multiplying it by x. Thus, our new guess for the
x 2 x
particular solution reads yp(x) = x e + x e .
x x 2 x x
Then y'p(x) = x e + e + x e + 2 xe ,
x x 2 x x x
and y"p(x) = x e + 2 e + x e + 4 xe + 2 e
Substituting into (1) gives
y"p y'p 2y p = x e + 2 e + x e + 4 x e + 2 e + x e + e + x e +
x x 2 x x x x x 2 x

x x 2 x x
+ 2 x e 2 x e 2 x e x e .
2 2 2
i.e. x + 2 + x + 4 x + 2 + x + + x + 2 x 2 x 2 x x.
2 2 2
i.e. x + 2 + x + 4 x + 2 + x + + x + 2 x 2 x 2 x x
2
x [ + 2 ] + x[ + 4 + + 2 2 ]+[2 + 2 + ] x,
6 x + 3 + 2 x
i.e. = 1/6 , 3 + 1/3 = 0 = 1/9 and = 1/6
x
Grouping together terms that vary as x e , we find that 6 =1. Similarly, terms that
x
vary as e yield 2 + 3 = 0. Therefore = 1/9 and = 1/6
x 2 x
i.e. yp(x) = 1 x e + 1 x e ,
9 6

50
Advanced Engineering Mathematics Chapter 2- Second-Order Linear D.E.
2x x x 2 x
so that the general solution is y = yH(x) + yp(x) = A e + B e 1 x e + 1 x e .
9 6
In summary, the method of finding particular solutions to higher-order
ordinary differential equations by the method of undetermined coefficients is as
follows:
Step 1: Find the homogeneous solution to the differential equation.
Step 2: Make an initial guess at the particular solution. The form of yp(x) is a
linear combination of all linearly independent functions that are generated by
repeated differentiations of f(x).
Step 3: If any of the terms in yp(x) given in Step 2 duplicate any of the
homogeneous solutions, then that particular term in yp(x) must be multiplied
n
by x , where n is the smallest positive integer that eliminates the duplication.

Example 2.42: Let us apply the method of undetermined coefficients to solve


d2y dy
sin x e3x cos5x. .(1)
dx 2 dx
We begin by first finding the solution to the homogeneous version of (1)
2 2
Auxiliary equation is m + 1 = 0 or (m + 1) = 0 , m = i.
C.F. = A cos x + B sin x , ............(2)
To find the particular solution we examine the right side of (1) or
3x
f(x) = sin x e cos 5x. ................(3)
Taking a few derivatives of f(x), we find that
3x 3x
f '(x) = cos x 3e cos 5x + 5e sin 5x, ..........(4)
3x 3x 3x
f"(x)= sin x 9 e cos 5x + 30 e sin 5x + 25e cos 5x, .........(5)
and so forth. Therefore, our guess at the particular solution is
3x 3x
yp(x) = C x sin x + x cos x + E e cos 5x + F e sin 5x,...........(6)

Why have we chosen x sin x and x cos x rather than sin x and cos x? Because sin x
and cos x are homogeneous solutions to (1), we must multiply them by a power of x.
Since
3x
y"p(x) = 2C cos x C x sin x 2 sin x x cos x + (30 F 16E)e cos 5x
3x
(30 E + 16 F) e sin 5x ,
and
3x 3x
y"p + yp = 2C cos x 2 sin x + (30 F 16E) e cos 5x (30 E + 16 F) e sin 5x
3x
sin x e cos 5x ,.......................................................................(7)
Therefore, 2C = 0, 2 = 1, 30F15E= 1, and 30E + 15F = 0. Solving this system of
equations yields C = 0, = 1 , E = 1 and F = 2 . Thus, the general solution is
2 75 75
3x 3x
y(x) = A cos x + B sin x 1 x cos x + 1 e cos 5x + 2 e sin 5x
2 75 75
Example 2.43: Find a particular solution of the equation

51
Advanced Engineering Mathematics Chapter 2- Second-Order Linear D.E.

d4y d2y
2 y 3sin x 5cos x. .........................................................................(1)
4 2
dx dx
4 2 2 2
Auxiliary equation is m + 2m + 1 = 0 or (m + 1) = 0 , m = i twice.
C.F. = A cos x + B sin x + C x cos x + x sin x , ............(2)
The general solution of the homogeneous equation is given in Relation (2).
yc(x) = A cos x + B sin x + C x cos x + x sin x,
corresponding to the roots r = i , i , i, and i of the characteristic equation. Our
initial assumption for a particular solution is Y (t) = sin x + cos x, but we must
2
multiply this choice by x to make it different from all solutions of the homogeneous
2 2
equation. Thus our final assumption is Yp (t) = x sin x + x cos x.
Next, we differentiate Yp (x) four times, substitute into the differential equation (1),
and collect terms, obtaining finally 8 sin x 8 cos x 3 sin x 5 cos x.
Thus = 3/8 , = 5/8 , and the particular solution of Eq. (1) is
2 2
Yp (x) = 3 x sin x + 5 x cos x. The complete solution is
8 8
2 2
y(x) = A cos x + B sin x + C x cos x + x sin x 3 x sin x + 5 x cos x.
8 8
If f(t) is a sum of several terms, it is often easier in practice to compute
separately the particular solution corresponding to each term in g(t). As for the
second order equation, the particular solution of the complete problem is the sum of
the particular solutions of the individual component problems. This is illustrated in
the following example.
dy d3 y
Example 2.44: Solve x 3cos x e2x . ...............(1)
4
dx 3 dx
3
First we solve the homogeneous equation. The characteristic equation is m 4m = 0,
2x 2x
and the roots are 0, 2; hence C.F. = A + B e + C e ....................................(2)

We can write a particular solution of Eq. (1) as the sum of particular solutions of the
differential equations
d3y dy d3 y dy d3 y dy
4 x 4 3cos x , 4 e2x .
dx 3 dx dx 3 dx dx 3 dx
Our initial choice for a particular solution Y1(x) of the first equation is Ao x + A1, but
since a constant is a solution of the homogeneous equation, we multiply by x. Thus
Y1(x) = x (Ao x + A1).
For the second equation we choose Y2(x) = B cos x + C sin x,
and there is no need to modify this initial choice since cos x and sin x are not
2x
solutions of the homogeneous equation. Finally, for the third equation, since e is a
2x
solution of the homogeneous equation, we assume that Y3(x) = E xe .
52
Advanced Engineering Mathematics Chapter 2- Second-Order Linear D.E.

The constants are determined by substituting into the individual differential


equations; they are Ao= 1 , A1 = 0 , B = 0 , C = 3 , and E = 1 .Hence a particular
8 5 8
1 2 3 1 2x
solution of Eq. (1) is Yp(x) = x sin x + x e .
8 5 8
2x 2x 1 2 3 2x
The complete solution is y = A + B e + C e x sin x + 1 x e .
8 5 8
The amount of algebra required to calculate the coefficients may be quite substantial
for higher order equations, especially if the nonhomogeneous term is even
moderately complicated. A computer algebra system can be extremely helpful in
executing these algebraic calculations.
The method of undetermined coefficients can be used whenever it is possible to
guess the correct form for Y (t). However, this is usually impossible for differential
equations not having constant coefficients, or for nonhomogeneous terms other than
the type described previously.

d2y dy
Example 2.45: Solve 3 2y x 2 1. ....(1)
dx 2 dx
2
Auxiliary equation is m 3m + 2 = 0 or (m 1)(m 2) = 0 , m = 1 ,2.
x 2x
C.F. = A e + B e , .................................................(2)
The general solution of the homogeneous equation is given in Relation (2).
Since the right side of the equation is of order two, we propose for y the quadratic
2
polynomial a x + b x + c, and proceed via substitution in the given equation to
determine the values of the constants a, b and c.
2
yp = a x + b x + c , yp' = 2 a x + b , yp" = 2a .
2 2
Substitution yields 2a 3 (2 a x + b) + 2 (a x + b x + c) x +1.
2 2
x [2 a] + x[ 6a + 2b] + [2a 3b + 2c] x +1
2 2
2a 3 (2 a x + b) + 2 (a x + b x + c) x +1.
2
Equating the coefficients of x yields 2a = 1, 2b 6a = 0 and 2a 3b + 2c = 1.
a = 1 , b = 2 and c = 9
2 3 4
x 2x 2 x 2x 2
y = yh + yp = A e + B e + 1 x + 2 x + 9 = A e + B e + 1 [ 2 x + 6 x + 9].
2 3 4 4

53
Advanced Engineering Mathematics Chapter 3- Fourier Series

CHAPTER 3
FOURIER SERIES
3.1 Periodic Functions:
Let f(x) is defined on the real x-axis. If f(x + T) = f(x), for all x(, ),
T is a real constant, f(x) is said to be periodic with period T. It is clear that:
f(x + n T) = f (x + [n1]T) = = f(x + T) = f(x), where n is a positive integer.
The least value of T > 0 is called the least period or simply the period of f(x). For
example: cos x = cos(x + 2) = = cos(x + 2n), thus the period of cos x is 2.
Also the period of sin x is 2, the period of tan x is and the period of a constant
function is any positive number.
If f(x) has the period T then the function: g(x) = f(a x) has the period T/a. It
is easy to see that g(x + T/a) = f(a{x + T/a}) = f(a x + T) = f(a x) = g(x) .
It is clear that cos 2x has the period , the period of sin 4x is /2.
CT T
If f(x) has the period T, then: f (x)dx = f (x)dx (Prove it?)
C 0
It is apparent that a sum, a difference, a product and a ratio of periodic
functions with the same period T are again periodic functions with period T, (not
necessary the least period). It is clear that tan x = sin x/ cos x , the period of sin x,
cos x is 2, but the period of tan x is .

3.2 Odd and Even Functions:


A function f(x) is said to be odd if: f(x) = f(x) for all x.
5 3 x x
It is easy to see that: x , x 2x, sin x, tan x , (e e ) are odd functions.
A function f(x) is said to be even if : f(x) = f(x) for all x .
2 4 2 x x
Thus x , x + 3 x + 7, cos x , (e + e ) are even functions.

These definitions imply that the graph of an even function is symmetric with
respect to the axis of ordinates and the graph of an odd function is symmetric with

54
Advanced Engineering Mathematics Chapter 3- Fourier Series

respect to the origin. It is clear that a linear combination of an even (odd) functions is
also even (odd) function.
If f(x) is integrable on the interval [a , a], then
a 0 a
f (x)dx = f (x)dx + f (x)dx
a a 0
0 a
Replacing x by x in the first integral, we get: f (x)dx = f ( x)dx .
a 0
a
a a 2 f (x)dx, if f (x) is an even function,
Then f (x)dx = [f (x) f ( x)]dx = 0
a 0
0 if f (x) is an odd function.

In general, any function f(x) can be represented as the sum of even and odd
functions. It is obvious that:
f(x) = [f(x) + f(x)] + [f(x) f(x)] = F(x) + G(x),
where F(x) = [f(x) + f(x)], G(x) = [f(x) f(x)], F(x) even function and
G(x) odd function. Every function f(x), f(x) defined on the interval of the form
[a, a] can be represented as the sum of the corresponding even and odd functions.
Example 3.1:
Graph each of the following functions:
a) f(x) = |x| , x [ , ] and has a period 2.

Since the period is 2, that portion of the graph in x is extended


periodically outside this range. It is clear that |x| = |x|, then f(x) is an even
function.

4 , x ( ,0)
b) f(x) = period 2.
, x (0 , )
4

55
Advanced Engineering Mathematics Chapter 3- Fourier Series

It is clear that, f(x) is not defined at x = 0, , 2 , etc.


These values are the discontinuities of f(x). It is obvious that f(x) is an odd function.

c) f(x) = x , < x , period = 2.

Exercises 3.1
Graph each of the following functions and classify each of them according
as they are even, odd, or neither even nor odd.

cos x 0 x
2 0 < x 3 2) f (x) ,period =2
1) f (x) , period = 6. 0 x 2
2 3 x 0

sin x 0 x
sin x 0 x 4) f (x) , period = 2
3) f (x) , period =2 cos x x 2
0 x 2

0, 0 x<1
5, 0<x<3
5) f (x) 2, 1 x < 2 , period = 3. 6) f (x) , period = 6.
0, 5, 3 < x < 0
2 x<3

7) f(x) = x(5x) , 0 < x < 5, period = 5.

56
Advanced Engineering Mathematics Chapter 3- Fourier Series

3.3 Fourier Series:


Let f(x) be defined in the interval (T, T) and outside of this interval by
f(x + 2T) = f(x), i.e. assume that f(x) has the period 2T. The Fourier expansion
or Fourier series corresponding to f(x) is given by:


S(x) = 1 ao +
2 [ an cos
n 1
n T x + b n
sin n x ],
T (1)

where ao , an , bn , n = l, 2, are called Fourier coefficients and given by :

1 T CT
an =
T T
f (x) cos
n x dx = 1
T
T CT
f (x) cos n x dx,
T (2)

1 T CT
bn =
T T
f (x) sinn x dx = 1
T
T CT
f (x) sin n x dx,
T (3)

where C is any real number, n = 1, 2, 3, .

1 T
To determine ao, using (2) with n = 0, hence ao = f (x) dx
T T

1 1 T
The constant term in (1) is equal to: ao =
2 f (x) dx,
2T T

this is the mean of f(x) over a period. It is clear that bo = 0, since sin 0 = 0.

Example 3.2:
Find the Fourier coefficients corresponding to the function:
0 3 < x < 0
f (x) , period = 6
2 0x3

Solution:

The graph of f(x) is shown below

57
Advanced Engineering Mathematics Chapter 3- Fourier Series

Period = 2T = 6, and T = 3. Then

13 10 13
an = f (x) cos
3 3
n
3
x

dx = (0) cos
3 3
n
3
x
30

dx + (2) cos n x dx =
3
23
2 3

= cos n x dx = [ sin n x ] 30 = 0 for all n = 1, 2, 3, .
30 3 3 n 3

23
ao = dx = 2.
30

Similarly,

23
bn = sin
30
n 3 x dx = 2[ 1 cos n ]/n = =2[ 1 (1) n
]/n =

0 if n is even
=
4 / n if n is odd

i.e. b1 = 4/ , b2 = 0, b3 = 4/3 , b4 = 0, b5 = 4/5, b6 = 0, etc.


Therefore b2 = b4 = b6 = = b2m = 0, b2m+1 = 4/(2m+1) , for all m,

ao = 2, a1 = a2 = a3 = = am = 0.

58
Advanced Engineering Mathematics Chapter 3- Fourier Series

Exercises 3.2
Prove that
T T
1)
T
T
sin n x dx = cos
T
n T x dx = 0 , n = 1, 2, 3, .
T

2) sin n x cos n x dx = 0, n = 1, 2, 3, .
T
T T

T T
3)
T
T
cos n x cos m x dx =
T sin
T
n T x sin m T x dx =
0 if m n
=
T if m = n

Find the Fourier coefficients corresponding to the functions:


0 2 <x< 0
4) f (x) , period = 4.
5 0x2

[Answers: ao = 5, an = 0, b2m = 0, b2m+1 = 10/(2m+1) ]


5) f (x) = ( x)/ 2 , x (0, 2), period =2.
[Answers: ao = an = 0, bn = 1/n ]
6) f (x) = | x |, 1 < x < 1, period =2.
2 2
[Answers: ao = 1, a2n = 0, a2n+1 = 4/ (2n+1) , bn = 0.]

3.4 Convergence of Fourier series:


Definition: Piecewise continuity:
A function f(x) is said to be piecewise continuous in an interval if:
1) the interval can be divided into a finite number of subintervals in each of which
f(x) is continuous and,
2) the limits of f(x) as x approaches the endpoints of each subinterval are finite.
i.e. the piecewise continuous function is one that has only a finite number of finite
discontinuities.
59
Advanced Engineering Mathematics Chapter 3- Fourier Series

Now, we shall state a theorem that will yield sufficient conditions for
representing a function f(x) by a Fourier series.
Theorem 3.1 (Dirichlet conditions):
Suppose that:
1) f(x) is defined and single-valued except possibly at a finite number of
points in (T, T),
2) f(x) is periodic outside (T, T) with period 2T,
3) f(x) and f'(x) are piecewise continuous in (T, T).
Then the series (1) with coefficients (2) converges to:
a) f(x) if x is a point of continuity.
b) [f(x + 0) + f(x0)] if x is a point of discontinuity, where
f(x+0) = lim f(x+), f(x0) = lim f(x)
0 0

The conditions (1), (2) and (3) imposed on f(x) are sufficient but not
necessary, and are generally satisfied in practice. There are at present no known
necessary and sufficient conditions for convergence of Fourier series.
In Example (1.2); the Fourier series corresponding to f(x) is:
1+ (4/){ sin(x/3) + (1/3) sin(3x/3) + (l/5) sin(5x/3) + (l/7) sin(7x/3) +.}.
Since f(x) satisfies the Dirichlet conditions, we can say that the series
converges to f(x) at all points of continuity and to [f(x + 0) + f(x0)] at points of

60
Advanced Engineering Mathematics Chapter 3- Fourier Series

discontinuity. At x = 3, 0 and 3, which are points of discontinuity, the series


converges to [2 + 0] = 1 as seen from the graph.
If we redefine f(x) as follows:
1 x =3
2 3 x 0

f (x) 1 x 0 , period = 6.
2 0x3

1 x 3

Then the series will converge to f(x) for 3 x 3.

Example 3.3:
2
Expand f(x) = x , 0 < x< 2 with period 2, in Fourier series.
Solution:
The graph of f(x) with period 2 is shown below:

Period = 2T = 2 and then T = .

1 x 2 8
3 2
1 2 2
ao = x dx = = .
0 3 0 3

1 2 2 1 x 2 2 1 2 2
2
an = x cos nx dx=
0
x d(sin nx) = n [ x sin nx 0 2x sin nx dx] =
n x 0 0

61
Advanced Engineering Mathematics Chapter 3- Fourier Series

x 2 2
= 2
n 1
n
2 2
x d(cos n x) = 2 [x cos n x 0 cos nx dx ] =
x 0 n 0

sin nx 2 4 2n 4
= 22 [ 2 cos 2n 0 ] = 2 (1) = 2 , n 0
n n n n

1 2 2 1 [ x2 cos nx 2x sin nx +2 cos nx ] 2 = 4 .


bn =
0
x sin n x dx =
2 3 0
n n n n

2
4 2 4
Then f(x) = x = + [ 42 cos n x sin n x].
3 n 1 n n
This is valid for 0 < x < 2. At x = 0 and x = 2 the series converges to
2 2
[0 + 4 ] = 2 .
2
4
At x = 0 the Fourier series reduces to + [ 42 ].
3 n 1 n
2
By Dirichlet condition, the series converges at x = 0 to 2 . Then
2
4 2 4 ] = (2 4/3)2 = 2/32.
+ [ 42 ] = 2 , i.e. [ 2
3 n 1 n n 1 n


4 2
Hence [ 2 ] = .
n 1 n 6

Example 3.4:
Let us take the periodic function f(x) = x, regarded as being defined on the interval
(T, T) with period 2T, and expand it into a Fourier series
Solution:

It is clear that, this function is an odd function, then f(x) cos n x T is an odd

62
Advanced Engineering Mathematics Chapter 3- Fourier Series


function and f(x) sin n x is an even function. Hence
T

1 T 1 T
an =
T T
f (x) cos n
T
x dx =
T T
T
x cos n x dx = 0, n = 1, 2, 3,

1 T
ao = x dx = 0,
T T

1 T T T
bn =
T T
f (x) sin
n x dx = 1
T

T T
x sin
T T0
T
n x dx = 2 x sin n x dx =

T
2 xT
= [
T n
cos n
T
x T
0+
n 0

cos n x dx] =
T
2T
n

n+1
(1) .
Consequently, according to the convergence theorem, we have


n 1
2T (1)
S(x) = [
n 1 n
] sin n x =
T 0 for x=T
x for T< x < T
,

Since S(T) = [ f(T + 0) + f(T0)] = [ T + T] = 0 , for x = T.


It is clear that the function S(x) is periodic with period 2T, and S(x) = x ,
only for T < x < T. At the points x = (2m+l)T, m = 0, 1, 2, etc., the
function x has discontinuities, its left-hand and right-hand limits being,
respectively, equal to T and +T, but S(x) assumes the value which is the half-
sum of the left-hand and right-hand limits i.e. equal to (T+T) = 0 .
In the following graph we see the graph of the partial sum S5 (x):

2T x 1 2x 1 3x 1 4x
S5(x) = [ sin sin + sin sin +
T 2 T 3 T 4 T
1 5x
+ sin ],
5 T
constructed on the interval [T, T].

63
Advanced Engineering Mathematics Chapter 3- Fourier Series

APPENDIX 3.I
In the following, we give some important integrals:
1 [cos a x + a x sin a x] + C,
1) x cos a x d x = 2
a
2 1 [2 a x cos a x + (a2 x2 2) sin a x] + C,
2) x cos a x d x = 3
a
3 1 2 2 3 3
3) x cos a x d x = 4 [(3 a x 6) cos a x + (a x 6a x) sin a x] + C,
a
4 3 3 4 4 2 2
4) x cos a x d x = 1 [(4 a x 24ax) cos a x+(a x 12a x +24) sin a x]+ C,
5
a
5 1 [(5 a4 x460a2 x2+120) cos a x+
5) x cos a x d x = 6
a
5 5 3 3
+(a x 20a x +120) sin a x] + C,
n n n1
6) a x cos a x d x = x cos a x n x cos a x dx,
1 [sin a x a x cos a x] + C,
7) x sin a x d x = 2
a
2 1 [2 a x sin a x (a2 x2 2) cos a x] + C,
8) x sin a x d x = 3
a
3 1 2 2 3 3
9) x sin a x d x = 4 [(3 a x 6) sin a x (a x 6a x) cos a x] + C,
a
4 1 3 3 4 4 2 2
10) x sin a x d x = [(4 a x 24ax) sin a x(a x 12a x +24) cos a x] + C,
5
a
5 1 [(5 a4 x460a2 x2+120) sin a x
11) x sin a x d x = 6
a
5 5 3 3
(a x 20a x +120) cos a x] + C,
n n n1
12) a x sin a x d x = x cos a x + n x cos a x dx,

64
Advanced Engineering Mathematics Chapter 3- Fourier Series

ax
ax e
13) e sin nx dx = [ a sin n x n cos n x] + C,
2 2
a n
ax
ax e
14) e cos nx dx = 2 2
[ a cos n x + n sin n x] + C.
a n
Example 3.5:
2
Let us take the periodic function f(x) = x , on the interval [T, T] with period 2T.
This function being even, we have:

2 T 2 2 3 T 2 2
ao =
T 0
x dx =
3T
[x ] 0 = T .
3

2 T 2
an =
T 0
x cos
T
T n T

n x dx = 2 T 3 2n x cos n x +
T


T

2 2
+ n x 2 sin n x = 2T
2 2
[ 2n cos n + (n 2) sin n ] =
T 0 n
T
3 3

2 n
4T ( 1)
= 2 2
,
n

1 T 2
bn =
T T
x sin n x dx = 0.
T
2 2 ( 1) n nx 2
1 4T
Then S(x) = T + 2 cos =x , T x T.
3 n 1 n 2 T

65
Advanced Engineering Mathematics Chapter 3- Fourier Series

2
The function x and the sum S(x) of Fourier's series are continuous everywhere
including the points x = (2m+l)T , m = 0, 1, 2, , since for the function
2 2
f(x) = x we have, f(T + 0) = f(T 0) = T .
If T= we get:

2 2
( 1) n
x = 1 + 4 2 cos n x, x .
3
n 1 n

Substituting with x = 0, we obtain the useful relation:


2 m 1
= 1 1 + 1 1 + + ( 1) +
12 4 9 16 m
2

3.5 Fourier series for even and odd functions:


From the definition of an even function and odd function it follows that:
T
T T 2 f (x)dx, if f (x) is an even function,
f (x)dx = [f (x) f ( x)]dx = 0
T 0
0 if f (x) is an odd function.

Now, if an odd function f(x) is expanded in a Fourier series, then the



product f(x) cos n x is also an odd function, while f(x) sin n x is an even
T T
function; hence,

1 T 1 T
ao =
T T
f (x) dx = 0, an =
T T T
f (x) cos n x dx = 0, n = 1, 2, 3, .

1 T 2T
bn =
T T
f (x) sin n
T
x
T0

dx = f (x) sin n x dx.
T
Thus the Fourier series of an odd function contains only sines (sine terms);
(see Example 3.4).
If an even function f(x) is expanded in a Fourier series, then the product

T T
f(x) cos n x is also an even function, while f(x) sin n x is an odd function;

hence,
66
Advanced Engineering Mathematics Chapter 3- Fourier Series

1 T 2T
ao = f (x) dx = T f (x) dx,
T T 0

1 T 2T
an =
T T
f (x) cos n

T
x
T0 T
dx = f (x) cos n x dx
1 T
bn =
T T
T
f (x) sin n x dx = 0, n = 1, 2, 3, .

Thus, the Fourier series of even functions contains only cosines


(see Example 3.5).
The formulas obtained permit simplifying computations when seeking
Fourier coefficients in cases when the given function is even or odd. It is obvious
that not every periodic function is even or odd, for example f(x) = sin x + cos x is
neither even nor odd.

3.6 Half range Fourier sine or cosine series:


A half range Fourier sine or cosine series is a series in which only sine
terms or only cosine terms are present respectively. When a half range series
corresponding to a given function is desired, the function is generally defined in
the interval (0, T) and then the function is specified as odd or even, so that it is
clearly defined in the other half of the interval, namely (T, 0) . In such case, we
have:

2T

an = 0 , bn = f (x) sin n x dx, for half range sine series,
T0 T

2T
bn = 0 , an =
T0 T
f (x) cos n x dx, for half cosine series.

Example 3.6:
Expand f(x) = sin x, 0 < x < in a Fourier cosine series.
Solution:

67
Advanced Engineering Mathematics Chapter 3- Fourier Series

A Fourier series consisting of cosine terms alone is obtained only for an even
function. Hence we extend the definition of f(x) so that it becomes even. With this
extension, f(x) is then defined in an interval of length 2. Taking the period as 2,
we have 2T = 2 so that T = .

2 2 4
ao = sin xdx = (cos x) = ,
0 0

2 1
an =
0
sin x cos nx dx = sin(x nx) sin(x nx) dx =
0

1 cos(n 1)x cos(n 1)x 1 1 cos(n 1) cos(n 1) 1


= = =
n 1 n 1 0 n 1 n 1

1 1 cos n cos n 1 1 cos n 1 1 2 1 cos n


= = = , n 1.
n 1 n 1 n 1 n 1 2
(n 1)

2 1
a1 = sin 2x dx = [cos 2x] = 0.
0 0

2 2 1 cos n
Then s(x) = cos nx =
n 2 (n 2 1)

2 4 cos 2x cos 4x cos6x


= 2 .
2 1 42 1 62 1

Example 3.7:
Expand f(x) = x, 0 x T in a half range a) sine series , b) cosine series.
Solution:

68
Advanced Engineering Mathematics Chapter 3- Fourier Series

a) Extend the definition of the given function to that of the odd function of period
2T (See Example 3.4 ) i.e. f(x) = x in the interval [T , T] .
n 1
2T (1)
Then S(x) = [
n 1 n T
] sin n x .
b) Extend the definition of the given function f(x) to that of the even function of
period 2T as shown below: Thus bn = 0, n = 1, 2, 3, ,

2T 1 2 T
ao =
T0
xdx = (x ) = T,
T 0

2
2T

an = x cos n x dx=
T0 T
2 T
Tn
nx
[cos T + n T x sin n T x ]=
2T 2T n
= [ ( cos n + n sin n ) ( 1 + 0)] = [(1) 1 ].
2 2 2 2
n n
2T
Then S(x) =
T
+
2 n 1 n 2 2
n
T
[(1) 1 ] cos n x =
=
T 4T

1
2 2 m 1 (2m 1) 2
cos
(2m 1)
T
x=
x 3x cos 5x cos 7 x
T 4T cos T cos T
= T T .
2 2 12 3
2
5
2
7
2

It should be noted that the given function f(x) =x, 0 < x < T, is represented equally
well by two different series in (a) and (b).

69
Advanced Engineering Mathematics Chapter 3- Fourier Series

Example 3.8:
2
Expand f(x) = x , 0 x < T in a half range a) cosine series, b) sine series.
Solution:
a) Extend the definition of the given function to that of the even function of period
2
2T (See Example 3.5 ) i.e. f(x) = x , T < x < T.
2 cos x cos 2x cos 3x cos 4x
4T 2

1
Then S(x) = T T T T T .
3 2 2 2 2 2
1 2 3 4
b) Extend the definition of the given function f(x) to that of the odd function of
x 2 0xT
period 2T: i.e. f (x) .
2
x Tx0

Then ao = an = 0, n = 1, 2, 3, ,

2 T 2
bn =
T 0 T
x sin n x dx = 2n x
T


sin
T
T




n x n x 2 2 cos n x T =
T 0


2 2
2T 2 2 2T 2 2 n
= [ (n 2) cos n 2] = [ (n 2)(1) + 2]=
3 3 3 3
n n
70
Advanced Engineering Mathematics Chapter 3- Fourier Series

2 2 2
2T n 2 2 n n+1 2T 4T n+1
= [(1) n + 2{1(1) }] = (1) {1+(1) }.
3 3
n n n
3 3

2 2
Then S(x) = [ (1)
n 1
n+1 2T

n n

4T
3 3
{1+(1)
n+1

}] sin n x =
T
2 x 2x sin 3x sin 4x sin 5x
2T sin T sin T
= T T T
1 2 3 4 5

2 x 3x sin 5x sin 7 x sin 9x
8T sin T sin T
T T T .
3 3 3 3 3 3
1 3 5 7 9

3.7 Harmonic analysis:

This is the determination of the Fourier coefficients ao, a1, a2 ,; b1,b2,.

In technology it is frequently used to analyse periodic phenomena. An oscillation


is split up by harmonic analysis into a sum of pure sine oscillations (harmonic
oscillations) and a constant part.
Apart from the fundamental oscillations there occur the so-called "harmonics"
whose frequency is twice, three times etc, the fundamental frequency. Much labour
can be saved in harmonic analysis of one observes certain symmetry properties of
the function f(x) to be analysed.
i) For the function with the property f(x + T) = f(x) (odd harmonic) the absolute
term is ao = 0 and only coefficients with an odd index occur
i.e. a2 = a4 = = b2 = b4 = = 0. It is clear that:

71
Advanced Engineering Mathematics Chapter 3- Fourier Series

1 T 10 T
ao = f (x) dx = f (x)dx f (x)dx
T T T T 0
in the first integral, put x = u T and dx = du

0 T T T
f (x)dx = f (u T)du = f (u T)du = f (u)du .
T 0 0 0

1 T T
ao = f (x)dx f (x)dx = 0,
T 0 0

1 T 0 T
an =
T T
f (x) cos
T
n x dx = 1

T T
f (x)cos
T
n x dx f (x)cos n x dx =
T
0
Put x = u T and dx = du, in the first integral


0 T T

T

f (x) cos n x
T dx = f (u T)cos
0
n(u T)
T
du = f (u)cos
0
n(u T)
T
du =

T T
n
= f (u) cos nu cos n sin nu sin n du = (1) f (u)cos nu du =
T T T
0 0

T
n+1 nu du .
= (1) f (u)cos T
0

1T

Then an = 1 (1)n 1 f (x)cos n x dx .
T0 T
2T
Therefore a2m = 0 for all m = 1, 2, 3, , and a2m1 = f (x)cos
T0
(2m 1) x
T
dx .
2T
Similarly b2m = 0 for all m = 1, 2, 3, , and b2m1 = f (x)sin
T0
(2m 1) x
T
dx .
ii) For the function with f(x + T) = f(x) (even harmonic) the absolute term ao and
only coefficients with an even index occur.

72
Advanced Engineering Mathematics Chapter 3- Fourier Series

1 T 10 T
ao = f (x) dx = f (x)dx f (x)dx
T T T T 0

Put x = u T and dx = du, in the first integral, we get

0 T T T
f (x)dx = f (u T)du = f (u T)du = f (u)du .
T 0 0 0

1 T T 2T
ao = f (x)dx f (x)dx = f (x)dx .
T 0 0 T 0

Also

1 T 10 T
an =
T T
f (x) cos n
T
x dx=
T T
f (x)cos n
T
x dx
0
f
(x)cos
T
n x dx =



Put x = u T and dx = du, in the first integral


0 T T

T

f (x) cos n x
T
dx = f (u T)cos
0
n(u T)
T
du = f (u)cos
0
n(u T)
T
du =

T T
n
= f (u) cos nu cos n sin nu sin n du = (1) f (u)cos nu du =
T T T
0 0

T
= (1) f (u)cos nu du .
n
T
0

1T

Then an = 1 (1) n 1 f (x)cos n x dx .
T0 T

73
Advanced Engineering Mathematics Chapter 3- Fourier Series

2T

Therefore a2m = f (x)cos 2m x dx , and a2m1 = 0 for all m = 1, 2, 3, .
T0 T

2T

Similarly b2m = f (x)sin 2m x dx ,and b2m1 = 0 for all m = 1, 2, 3, .
T0 T

iii) For the function with the properties f(x + T) = f(x), f(x) = f(x)
(odd cosine harmonic functions) we have bn = 0 for all n = 1, 2, 3, ,

ao = a2n = 0

, where a
(2n 1) x 4 2 (2n 1) x
and f(x) = a 2n 1 cos 2n1 = f (x)cos dx .
T T0 T
n 1

iv) For the function with the properties f(x + T) = f(x) , f(x) = f(x)
(odd sine harmonic functions) We have ao = an = 0, b2n = 0 for all n = 1, 2, 3,


4 2 (2n 1) x
b2n1 =
T0
f (x)sin
T
dx .

v) For the function with the properties f(x + T) = f(x) , f(x) = f(x)

74
Advanced Engineering Mathematics Chapter 3- Fourier Series

(even cosine harmonic functions) We have bn = 0, a2n1 = 0,

for all n = 1, 2, 3,


4 2
a2n = f (x)cos 2n x dx .
T0 T

vi) For the function with the properties f(x + T) = f(x) , f(x) = f(x)
(even sine harmonic functions). We have an = 0 for all n = 1, 2, 3, ,


4 2
b2n1 = 0, b2n = f (x)sin 2n x dx .
T0 T

Example 3.9:
Expand f (x) x
xT
0 x<T
T x 2T
, Period = 2T in Fourier series.

75
Advanced Engineering Mathematics Chapter 3- Fourier Series

Solution:

It is easy to see that f(x + T) = f(x), then f(x) - even harmonic;

i.e. a1 = a3 = = b1 = b3 = = 0, and

2T 2T 1 2 T
ao =
T0
f (x)dx = xdx =
T0 T
x
0
= T.

2T 2T
a2n = f (x)cos
T0

2n
T
x dx =

T0
x cos 2n x dx =
T
2
2 T
=
T 2n
2n x

2n x

2n x T
cos T T sin T =
0

T
=
2 2
cos 2n 2n sin 2n 1 = 0,

2n

2
2T
b2n = f (x)sin
T0

2n
T
x

2 T
dx =
T 2n T T T
T
sin 2n x 2n x cos 2n x =
0
T T
= sin 2n 2n cos 2n = .

2n 2 2 n

Therefore

T T sin T
f(x) =
2nx = T T sin 2Tx sin 4Tx sin 6Tx sin 8Tx .
2 n 1 n 2 1 2 3 4

Example 3.10:
x 0 x<T
Expand f (x) , Period = 2T in Fourier series.
x T T x 2T

76
Advanced Engineering Mathematics Chapter 3- Fourier Series

Solution:

It is obvious that f(x + T) = f(x), then f(x) is odd harmonic;

i.e. ao = a2 = a4 = = b2 = b4 = = 0, and

a2n1 =
2T

T0
f (x)cos
(2n 1) x
T
dx =
2T

T0
x cos
T
(2n 1) x
dx =

2
2 T (2n 1) x (2n 1) x (2n 1) x T
= cos sin =
T (2n 1) T T T 0


=
2T

(2n 1) 2 2
cos(2n 1) (2n 1) sin(2n 1) 1 = 4T
,
(2n 1)2 2

2T
b2n1 = f (x)sin
T0 T
(2n 1) x
dx =

2
2 T (2n 1) x (2n 1) x (2n 1) x T
= sin cos =
T (2n 1) T T T 0

2T 2T
= sin(2n 1) (2n 1) cos(2n 1) = .
(2n 1)2 2 (2n 1)

Therefore


4T (2n 1) x 2T (2n 1) x

f(x) = cos sin =
2
n 1 (2n 1)
2 T (2n 1) T

77
Advanced Engineering Mathematics Chapter 3- Fourier Series

2T 1

(2n 1) x 2 (2n 1) x

= sin cos .
n 1 (2n 1) T
(2n 1)
2 T

Example 3.11:
Expand f (x) = x , 0 x < T/2 , period = 2T and satisfies the following properties
f(x + T) = f(x) , f(x) = f(x) , in Fourier series.

Solution:
Extend the definition of the given function to that of the odd cosine harmonic
function of period 2T, and then we get:
x if 0 x T / 2
x T if T / 2 x T

f (x)
T x if T x 3T / 2
2T x if 3T / 2 x 2T

and ao = a2 = a4 = = a2n = b1 = b2= b3 = = bn = = 0 n = 1, 2, 3, .

T T
4 2
a2n1 = f (x)cos
T 0
(2n 1) x
T
dx =
4 2

T 0x cos
(2n 1) x
T
dx =

2
4 T (2n 1) x (2n 1) x (2n 1) x T 2
= cos sin =
T (2n 1) T T T 0


4T
1 =
(2n 1) (2n 1)
= cos (2n 1) sin
(2n 1) 2 2 2 2 2

78
Advanced Engineering Mathematics Chapter 3- Fourier Series

n 1
=
4T
(1)n 1 (2n 1) 1 = 4T ( 1) 1
,
(2n 1) 2 2 2
2 2(2n 1) (2n 1)2

4T ( 1)n 1
Therefore f(x) = 1
2 n 1 2(2n 1) (2n 1)
2

cos
(2n 1) x
T
=

x 3x cos 5x cos 7 x
2T cos T cos T
= T T
2 1 3 5 7

x 3x cos 5x cos 7 x
4T cos T cos T
T T .

1 2
3
2
5
2
7
2

Example 3.12:
Expand f(x) = 1 , 0 < x < /2 period = 2 with
a) odd cosine harmonic, c) odd sine harmonic,
b) even cosine harmonic, d) even sine harmonic.

Solution:
a) Extend the definition of the given function to that of odd cosine harmonic
function
of period 2 as shown in the figure. f(x) satisfies the properties:
f(x + ) = f(x) , f(x) = f(x).
Therefore ao = a2 = a4 = = a2n = 0,and
b1 = b2 = b3 = = bn = = 0 n = 1, 2, 3, ,

79
Advanced Engineering Mathematics Chapter 3- Fourier Series


4 2 4 sin(2n 1)x 2 4
and a2n1 = cos(2n 1)xdx = = sin(2n 1) =
0 (2n 1) 0 (2n 1) 2

4 n 1
= (1) .
(2n 1)

n 1
4 (1)
Thus f(x) = cos(2n 1)x =
n 1 (2n 1)

4 cos x cos3x cos5x cos7x cos9x


=
1 3 5 7 9 .

b) The case of an even cosine harmonic function i.e. f(x) satisfies the following
properties f(x + ) = f(x) , f(x) = f(x).

Therefore bn = a2nl = 0, n = 1, 2, 3, ,

4 2 4 sin 2nx 2 2
a2n = cos 2nx dx = = sin n = 0, n = 1, 2, 3, ,
0 2n 0 n

4 2 4
ao = dx = = 2.
0 2

Thus f(x) = 1 , and it is obvious since f(x) is constant function from the beginning.
c) The case of an odd sine harmonic, i.e. f(x) satisfies the following properties ,
f(x + ) = f(x) , f(x) = f(x).
Therefore ao = an = b2n = 0, n = 1, 2, 3, ,

80
Advanced Engineering Mathematics Chapter 3- Fourier Series


4 2 4 cos(2n 1)x 2
b2n1 = sin(2n 1)xdx = =
0 (2n 1) 0

4 4
= cos(2n 1) 1 = .
(2n 1) 2 (2n 1)

4 sin(2n 1)x
Thus f(x) = =
n 1 (2n 1)

4 sin x sin 3x sin 5x sin 7x sin 9x


= .
1 3 5 7 9
d) The case of an even sine harmonic i.e. f(x) satisfies the following properties
f(x + ) = f(x) , f(x) = f(x).
Therefore ao = an = b2n1 = 0, n = 1, 2, 3, ,


4 2 4 cos 2nx 2 2 2
1 .
n
b2n = sin 2nx dx = = cos n 1 =

( 1)

0 2n 0 n n

81
Advanced Engineering Mathematics Chapter 3- Fourier Series

n
2 [1 (1) ]
Then f(x) = sin 2nx =
n 1 n

4 sin 2x sin 6x sin10x sin14x sin18x


= .
1 3 5 7 9

Example 3.13:
Find the Fourier series for the function given by : f(x) = cos x , 0 < x < 2.

Solution:
It is clear that f(x + ) = f(x) , f(x) = f(x). Therefore the function is even cosine
harmonic. Thus bn = a2n1= 0, n = 1, 2, 3, ,


4 2 4 2 4 2 4 4
ao = f (x)dx = cos x dx = cos x dx = [sin x] 2 = .
0 0 0 0

4 2 4 2
a2n = f (x) cos 2nx dx = cos x cos 2nx dx =
0 0

2 2
= cos(2n 1)x cos(2n 1)x dx =
0


2 sin(2n 1)x sin(2n 1)x 2 2 sin(2n 1) 2 sin(2n 1) 2
= =
(2n 1) (2n 1) 0 (2n 1) (2n 1)

n 1 n 1
2 (1)
n n
(1) 2(1) 1 1 4(1)
= =
(2n 1) (2n 1) = .
(2n 1) (2n 1) 2
(4n 1)

n = 1, 2, 3,.

82
Advanced Engineering Mathematics Chapter 3- Fourier Series

n 1
2 4 (1)
Therefore f(x) = + cos 2nx =
n 1 (4n 2 1)

2 4 cos 2x cos 4x cos6x cos8x


= + .
4(1)2 1 4(2) 2 1 4(3)2 1 4(4)2 1

Example 3.14:
Find the Fourier series for the function given by: f(x) = sin x , 0 < x < 2.

Solution:
It is clear that f(x + ) = f(x) , f(x) = f(x).

Therefore the function is even cosine harmonic.

Thus bn = a2n1 = 0, n = 1, 2, 3, ,


4 2 4 2 4 2 4 4
ao = f (x)dx = sin x dx = sin x dx = [ cos x] 2 = .
0 0 0 0

4 2 4 2
a2n = f (x) cos 2nx dx = sin x cos 2nx dx =
0 0

2 2
= sin(2n 1)x sin(2n 1)x dx =
0

2 cos(2n 1)x cos(2n 1)x 2 2 1 1 4 1


= = = .
(2n 1) (2n 1) 0 (2n 1) (2n 1) 4n 2 1

2 4 1
Therefore f(x) = cos 2nx =
n 1 (4n 2 1)

83
Advanced Engineering Mathematics Chapter 3- Fourier Series

2 4 cos 2x cos 4x cos6x cos8x


= .
4(1) 2 1 4(2) 2 1 4(3)2 1 4(4)2 1

Compare this result with Example 3.6

Example 3.15:
Find the Fourier series for the function given by: f(x) = x for 0 x < T/2,
with period = 2T, a) Odd sine harmonic, b) Odd cosine harmonic,
c) Even sine harmonic, d) Even cosine harmonic.
Solution:

a) The case of odd sine harmonic: It is clear that f(x + T) = f(x) ,


f(x) = f(x), i.e. ao = an = b2n = 0, n = 1, 2, 3, , and
T
n 1
4T(1)
4 2 (2n 1) x
b2n1 = x sin dx = .
T0 T 2 2
(2n 1)
n 1
4T (1) (2n 1)x
Therefore f(x) =
2 2
sin =
n 1 (2n 1) T

x 3x sin 5x sin 7 x sin 9 x


4T sin T sin T
= T T T
2 2 2 2 2 2
1 3 5 7 9
b) The case of odd cosine harmonic. See Example 3.11.

84
Advanced Engineering Mathematics Chapter 3- Fourier Series

c) The case of even sine harmonic. It is clear that f(x + T) = f(x) , f(x) = f(x) .

and i.e. ao = an = b2n-1 = 0, n = 1, 2, 3, , and

T
n 1
T(1)
4 2
b2n = x sin 2n x dx = .
T0 T n

n 1
T (1) 2nx
Therefore f(x) = sin =
n 1 n T

2x sin 4x sin 6x sin 8x sin 10x


T sin T
= T T T T .
1 2 3 4 5

d) The case of even cosine harmonic. It is clear that f(x + T) = f(x) , f(x) = f(x) .

and i.e. bn = a2n-1 = 0, n = 1, 2, 3, , and

85
Advanced Engineering Mathematics Chapter 3- Fourier Series

T T
4 2 4 2 2 2 T2 T
ao = f (x)dx = x dx = x =
T 0 T 0 T 0 2

T
4 2 T
a2n = f (x) cos 2nx dx =
n
[(1) 1].
T 0 T 2 2
n

Therefore

T T
f(x) = +



n
(1) 1
cos

2nx T T
= +
2
cos
2(2m 1)x
=
4 2 n 1 n
2 T 4 2
m 1 (2m 1)
2 T

2x cos 6x cos 10x cos 14 x cos 18x


T 2T cos T
= + T T T T .
4 2 12 3
2
5
2
7
2
9
2

3.8 Parseval's Identity:


We can say that the series u1(x) + u2(x) + converges to the sum S(x) in a
region R , if given > 0 there exists a number N, which in general depends on both
and x, such that S(x) Sn (x)|< whenever n > N , where Sn (x) = u1(x)+
u2(x)+ + un(x). If we can find N depending only on and not on x, we say that
the series converges uniformly to S(x) in R. Uniformly convergent series have
many important advantages as indicated in the following theorems.

Theorem 3.8.1

If un(x), n = 1, 2, 3, are continuous in the interval [a, b] and u n (x) is
n 1

86
Advanced Engineering Mathematics Chapter 3- Fourier Series

uniformly convergent to S(x) in the interval [a, b], then S(x) is continuous in the
interval [a, b].

Theorem 3.8.2

If u n (x) converges uniformly to S(x) in the interval [a, b] and un(x) ,
n 1

n = 1, 2, 3, are integrable in [a, b], then:

b b
S(x)dx = u1 (x) u 2 (x) u 3 (x) dx =
a a

b b b
= u1 (x)dx + u 2 (x)dx + u 3 (x)dx +
a a a

Theorem 3.8.3
If un(x) , n = 1, 2, 3, are continuous and have continuous derivatives in
'
the interval [a, b] and if u n (x) converges to S(x) while u n (x) is uniformly
n 1 n 1
convergent in the interval (a, b), then
d ' ' '
S'(x) =
dx
u1 (x) u 2 (x) u 3 (x) = u1 (x) u 2 (x) u 3 (x) .

Theorem 3.8.4: Parseval's identity


If the Fourier series corresponding to f(x) converges uniformly to f(x) in the
interval
2
1 T 2 ao 2 2
[T, T], then f (x) dx = (a n b n ) .
T T 2 n 1

Where the integral is assumed to exist.


Proof:
Let us consider

87
Advanced Engineering Mathematics Chapter 3- Fourier Series


f(x) = 1 ao +
2
n 1
[ an cos n T x + b sin n T x ],
n

Multiplying by f(x) and integrating term by term from T to T (which is justified


since the series is uniformly convergent) We obtain

T T T

T
2
f (x) dx = 12 ao
T
f (x) dx + [ an f (x) cos
n 1 T
n T x dx +
T 2
+ bn f (x) sin
T
n
T
x dx] = T
ao
2
2
2
T (a n b n ) ,
n 1

where we have used the results

T T T
f (x) dx = T ao,
T T
T
f (x) cos n x dx=T an, f (x) sin
T
n T x dx= T b n

Obtained from the Fourier coefficients. Therefore


2
1 T 2 ao 2 2

f (x) dx = (a n b n ) .
T T 2 n 1

Note:
Parseval's identity is valid under less restrictive conditions than the imposes here.

Example 3.16:
Write Parseval's identity corresponding to the Fourier series of the function
defined in Example 3.7 ; and then determine the sum of the series:
1 1 1 1 1 1 1

4 4 4 4 4 4 4
1 2 3 4 5 6 n
Solution:

From the Example 3.7 (b) :- f (x)


x
x
0xT
Tx0
.

and S(x) =
T 4T

1
2 2 m 1 (2m 1) 2
cos
(2m 1)
T
x,
88
Advanced Engineering Mathematics Chapter 3- Fourier Series

1 T 4T 1
ao = , a2m-1 =
2 2 2 2
(2m 1)

1 x T 2T
3 2
1 T 2 1 T 2
Therefore f (x) dx = T x dx = T 3 T = 3 .
T T T

Hence
2 2 16T 2 2 2
2T T 1 T 16T 1
= + = + .
3 2 m 1 4 (2m 1)4 2 4
m 1 (2m 1)
4

2 1 1 16 1
= = ,
3 2 6 4 m 1 (2m 1)4

4 4
1 1 1 1 1
i.e. = = = .
6 16 m 1 (2m 1)4 14 34 54 (2m 1)
4 96

4
1 1 1 1
Hence = .
4 4 4 4 96
1 3 5 (2m 1)
1 1 1 1 1 1 1 1
Thus S = = =
4 4 4 4 4 4 4 4
n 1 n 1 2 3 4 5 6 n
odd part even part

1 1 1 1 1 1 1 1
= + =
4 4 4 4 24 44 64 4
1 3 5 (2m 1) (2m)

1 1 1 1 1 1 1 1 1 1 1 1
= + 4 =
4 + 4 S.
14 34 54 24 1 2
4
3
4
4
4

1 3
4
5
4
2

1 1 1 1 4
i.e. S[1 ] = =
4 4 4 4
2 1 3 5 96
4 4
16
S= = .
96 15 90

89
Advanced Engineering Mathematics Chapter 3- Fourier Series

Example 3.17:
From the Fourier's expansion of the function f(x) = sin x, x
(See Example 3.6) find the sum of the following series
1 1 1 1
S=
2 2 2 2 2 2 2 2
1 .3 3 .5 5 .7 7 .9
Solution:
Using the result of Example 3.6 we get:

2 4 cos 2x cos 4x cos6x


f(x) = 2 .
2 1 42 1 62 1

4 4 1
i.e. ao = , a2n = .
(2n)2 1

1 2 1 2 1 x sin x cos x
Therefore f (x) dx = sin x dx = =
2 2

1
= ( ) = 1
2 2
From Parseval's identity we get:

16 16 1 1 1 1
1= +
22 2 (22 1)2 (42 1)2 (62 1)2 (82 1)2

2
1 1 1 1 1
= +
16 2 12.32 32.52 52.7 2 7 2.92

2 2
1 8
Therefore S = = .
16 2 16
3.9 Differentiation and integration of Fourier series:
Differentiation and integration of Fourier series can be justified by using the
theorems 1.8.2, 1.8.3. It must be emphasized however, that those theorems
provide sufficient conditions and are not necessary.

90
Advanced Engineering Mathematics Chapter 3- Fourier Series

The following theorem for integration is especially useful.

Theorem 3.9:
The Fourier series corresponding to f(x) may be integrated term by term from a
x
to x, and the resulting series will converge uniformly to f (u)du provided that
a

f(u) is piecewise continuous in T u T and x [T , T].

Example 3.18:
2
Find a Fourier series for f(x) = x , T < x < T, by integrating the series of

( 1) n 1
Example 3.4, and use it to evaluate the sum of the series .
2
n 1 n
Solution:
From Example 3.4,
n 1
2T (1)
x=
n 1 n T
sin n x , T < x < T

Integrating both sides from 0 to x ( applying Theorem 1.9.1) and multiplying by


2, we find :
n
4T (1)
2
x =C+
n 1 n
nT cos n T x ,
2 n
=C+
4T
2
(1)
n 1 n
2 T
cos n x .
2 n
4T (1)
2
Put x = 0 , we get C = .
2
n 1 n
2 n 1 2 n 1
4T (1)
2
i.e. x =
2
4T(1)
n 1 n
2

2
n 1 n
cos n x
T ,
2
and this represents the Fourier cosine series for x in [T, T].
Therefore:

91
Advanced Engineering Mathematics Chapter 3- Fourier Series

1 T 1 T 2 1 1 2
2C = ao = f (x) dx = x dx = x3 TT = [ T3 (T3)] = T2.
2
T T T T 3T 3T 3

2 n
1 2 4T (1)
2
i.e. C = T and since C = .
3 2
n 1 n

( 1) n 2 2
1 2
= T = .
2 3 2 12
n 1 n 4T
3.10 Complex notation for Fourier series:
Using Euler's identities:
ix
exp (i x) = e = cos x + i sin x = cis x, i = 1
ix
exp (i x) = e = cos x i sin x = cis (x)
ix ix
exp ix exp(ix) cis ix cis(ix) e e
and cos x = = = ,
2 2 2
ix ix
exp ix exp(ix) cis ix cis(ix) e e
sin x = = = .
2 2 2
Supposing that the Dirichlet conditions are satisfied for the function f(x) and
further that f(x) is continuous at x. Then



f(x) = 1 ao + [ an cos n x + bn sin n x ],
2
n 1
T T

Putting , we get
T
inx inx inx inx
e e e e
f(x) = 1 ao + [ an + bn ]=
2 2 2
n 1

inx a n ib n inx a n ib n
= 1 ao + [ e +e ].
2 2 2
n 1

a bn a ib n
Put n = Cn, n = Cn, 1 ao = Co , we get
2 2 2

92
Advanced Engineering Mathematics Chapter 3- Fourier Series

f(x) =

Cn e
inx
=

Cn
e inTx , and
n n

1 T
Cn =
a n ib n
2
=
2T T T T
f (x) [sin n x i sin n x ]dx

=
1 T
f (x) e

inx
T dx,
2T T

1 T
Cn =
a n ib n
2
=
2T T T T
f (x) [sin n x + i sin n x ]dx =

=
1 T
f (x) e
inTx dx.
2T T

1 T
Co = f (x) dx
2T T

Therefore the Fourier series for f(x) can be written as:

f(x) =

Cn
e inTx , (4)
n

where

Cn =
1 T
f (x) e
inTx dx, (5)
2T T

If f (x) is discontinuous at x, the left hand side of (4) shall be replaced by


[f(x+0) + f(x0)] / 2 .

Example 3.19:
ax
Expand the function f(x) = e , x [ , ], period = 2 in complex Fourier
series.
Solution:
93
Advanced Engineering Mathematics Chapter 3- Fourier Series

It is clear that:

1 ax inx 1 (a in)x 1 1 (a in)x


Cn =
2
e e dx =
2
e dx =
2 a in
e

=

1 e(a in) e(a in) ,


=
2(a in)
in n
and since e = cos n i sin n = cos n = (1) , then
n a a n
Cn =
1 ea .ein ea .ein = (1) e e = (1) sinh a ,
2(a in) (a in)
2 (a in)
x x
e e
where sinh x = . Therefore
2

ax sinh a (1)n inx


f(x) = e = e .
n (a in)

Exercises 3.3

1. Expand the following function in a Fourier series in the interval

( , ), period 2

f (x)
2x
x
0x
x 0
.

2 cos x cos3x cos5x cos 7x


[Answer: +
4 12 3
2
5
2
7
2

sin x sin 2x sin 3x sin 5x


+3 ] .
1 2 3 5

2. Expand f(x) = 1, 0 < x < , period = 2 in a Fourier sine series and using it to

1 1 1
calculate the sum of the series 1 + + . [Answer: ].
3 5 7 4

94
Advanced Engineering Mathematics Chapter 3- Fourier Series

3. Expand f (x) 2x
x6
0<x<4
4x < 8

in a Fourier series of period 8.

16 x 1 cos 3x 1 cos 5x 1 cos 7 x ]


[Answer: cos
2 4 2 4 2 4 2 4
3 5 7

4. Expand f(x) = cos x , 0 < x < , in a Fourier sine series. How should f (x) be

defined at x = 0 and x = so that the series will converges to f(x) for

8 n sin 2nx
0x ? [Answer: f(x) = , f(0) = f() = 0].
n 1 4n 2 1

5. Expand in a Fourier series f(x) = cos x , 0 < x < if the period is and compare

with the result of problem 4, explaining the similarities and difference if any.

[Answer: there is no difference between them.].

6. Expand f (x) x
8x
0<x<4
4x<8

in a series of (a) sines (b) cosines.

32 1
[Answer: (a) sin n sin nx , (b) 2 +
2 2 2 8
n 1 n

n
16 2cos 2 cos n 1
+ cos nx ]
2 2 8
n 1 n

7. Prove that for 0x

2
cos 2x cos 4x cos 6x cos8x
(a) x( x) =
6 12 2
2
3
2
4
2

95
Advanced Engineering Mathematics Chapter 3- Fourier Series

8 sin x sin 3x sin 5x sin 7x


(b) x( x) = 3
1 3
3
5
3
7
3

and using it to show that:

1
2 1
2 (1) n 1
2
i) , ii) , iii) .
2 6 2 6 3 32
n 1 n n 1 n n 1 (2n 1)

8. i) Prove that for x ,

sin x sin 2x sin 3x sin 5x


x = 2 ,
1 2 3 5

ii) By integrating the result of (i), show that for < x < ,

2 cos x cos 2x cos3x cos 4x


2
x = 4 ,
3 12 2
2
3
2
4
2

iii) By integrating the result of (ii), show that for < x < ,

sin x sin 2x sin 3x sin 4x


x( x) ( + x) = 12 .
3 3 3 3
1 2 3 4

9. (i) Show that for < x < ,

1 2 3 4 5
x cos x = sin x + 2 sin 2x sin 3x sin 4x sin 2x .
2 1.3 2.4 3.5 4.6

10. By differentiating the result of problem 7 (b) prove that for 0 x ,

96
Advanced Course in Mathematics Chapter 1- Fourier Series

4 cos x cos3x cos5x cos7x


x= .
2 12 3
2
5
2
7
2

11. By using problem 7 and Parseval's identity, show that :

1 4 1 6

i) , ii)
4 90 6 945
n 1 n n 1 n

12. Using the expansion of f(x) = sin x , 0 < x < in a Fourier cosine series to

1 1 1 1 2 8
show that = .
2 2 2 2 2 2 2 2 16
1 .3 3 .5 5 .7 7 .9

13. Show that

4 6
1 1
i) ii)
4 96 6 960
n 1 (2n 1) n 1 (2n 1)

2
1 1 1 4 391
14. Show that = .
2 2 2 2 2 2 2 2 2 2 2 2 16
1 .2 .3 2 .3 .4 3 .4 .5 4 .5 .6

2 2
x
15. Show that, the expansion of f (x) = in a Fourier series in the interval
12 4

cos x cos 2x cos3x cos 4x


( , ), period = 2 is .
2 2 2 2
1 2 3 4

16. Expand the function f(x) = cos2x in a series of sines in the interval (0, ) .

4 sin x 3sin 3x 5sin 5x 7sin 7x


[Answer: 2 2 2 ].
2 1 2 3
2 2
2 5
2 2
2 7
2

x
17*. Expand the function f (x) = e in a Fourier series in the interval (T, T).

97
Advanced Course in Mathematics Chapter 1- Fourier Series

T T n
e e T T (1) nx
[Answer: + T(e e ) cos( )+
2T 2 2 2 T
n 1 T n

n
T T (1) n nx
+T(e e ) sin( ) ].
2 2 2 T
n 1 T n


18*. Prove that if the function f(x) is even and we have f(x+ ) = f( x) then its
2 2

Fourier series in the interval (, ) represents as odd cosines harmonic, and if the


function f(x) is odd and we have f(x + ) = f( x), then its Fourier series in the
2 2

interval (, ) represents as odd sines harmonic.

19. Expand f(x) = | x |, 1< x < l in Fourier series period = 2.

1 4 cos(2n+1)x
[Answer: ].
2 2 n 0 (2n+1)2

2
20. Using the expansions of the functions x and x in the interval (0, ) in cosines

cos nx 3x 2 6x 22
series, prove the equality: = , (0 x ).
2 12
n 1 n

4
21. Expand the function f (x) = sin x in a Fourier series.

3 1 1
[Answer: cos 2x + cos4x].
8 2 8

22. For the periodic functions with the property f( 2 x) = f(x), prove that:

bn = 0 for all n = 1, 2, .

23. For the periodic functions with the property f(2 x) = f (x) prove that:

98
Advanced Course in Mathematics Chapter 1
ao = an = 0 for all n = 1, 2, 3, .

3
24. Expand the function f (x) = cos x, in a Fourier series.

3 1
[Answer: cos x + cos3x]
4 4

25. Expand the function f (x) = sin3x in a Fourier series x [ , ], period = 2.

[Answer: sin 3x]

99
Advanced Engineering Mathematics Chapter 4- Fourier Integrals

CHAPTER 4
FOURIER INTEGRALS
4.1 Theorem 4.1:
Let f(x) satisfies the Dirichlet conditions in every finite interval (T, T) and

f (x) dx converges (i.e. f(x) is absolutely integrable in (, ), then:


f(x) = A(u)cos ux B(u)sin ux du , (1)
0

1
where A(u) = f (x) cos ux dx ,

(2)

1
B(u) = f (x) sin ux dx .

(3)

The result (1) holds if x is a point of continuity of f(x). If x is a point of


discontinuity, we must replace f(x) by [f(x+0) +f(x0)]/2 as in the case of Fourier
series. Note that the above conditions are sufficient but not necessary. The right hand
side of (1) is sometimes called Fourier integral expansions of f(x).

Example 4.1:
1 x a

Find the Fourier integral of f (x) 1 x =a
2
0 x a

Solution:

f(x) = A(u)cos ux B(u)sin ux du ,
0

1 1 a 1
where A(u) = f (x) cos ux dx = cos ux dx = sin ux aa =
a u

1 2
= [sin u a sin (u a)] = sin u a,
u u

100
Advanced Engineering Mathematics Chapter 4- Fourier Integrals

1 1 a
B(u) = f (x) sin ux dx = sin ux dx = 0.
a

1 x a
21
Therefore f(x) = sin ua cos ux du = 1 x =a
0u 0
2
x a

4.2 Equivalent forms of Fourier's integral theorem:


Fourier's integral theorem can also be written in the forms:

1
f(x) = f(t) cos u(t x) dt du ,
u 0 t
(4)

1 iux iut 1 iu(t x)


f(x) =
2
e du f (t) e dt =
2
f (t) e dtdu . (5)

Where it is understood that if f(x) is not continuous at x the left side must be replaced
by [f(x+0) +f(x0)]/2.
The case of even functions:

1
f(x) = cos ux f(t) cos ut dt du , if f(x) is even. (6)
u 0 t 0
The case of odd functions:

1
f(x) =
u 0
sin ux f(t) sin ut dt du , if f(x) is odd. (7)
t 0

1 iut
If we put F (u) = f (t) e dt , (8)
2

1 iux
in (5) , we get : f(x) = F(u) e du (9)
2
The function F (u) is called the Fourier transform of f (x) and is sometimes
written F(u) = F{f (x)}. The function f (x) is the inverse Fourier transform of F (u)
1
and is written f(x) = F {F(u)}.

The constants preceding the integral signs in (8) and (9) were here taken as equal to

101
Advanced Engineering Mathematics Chapter 4- Fourier Integrals

1
. However, they can be any constants different from zero so long as their
2
1
product is . The above is called the symmetric form.
2
Equation (8) can be written as follows:

1 iut 1
F(u) = f (t) e dt = f (t) cos ut isin ut dt =
2 2

1 i
= f (t) cos ut dt + f (t)sin utdt . (10)
2 2
Equation (9) can be written as follows:

1 iux 1
f(x) = F(u) e du = F(u) cos ux isin ux du =
2 2

1 i
= F(u) cos ux du + F(u) sin ux du . (11)
2 2
If f(x) is an even function, then f(t) sin u t is odd and f(t) cos u t is even. Then
the second integral on the right of (10) is zero and the result can be written

2 2
Fc(u) = f (t) cos ut dt = f (t) cos ut dt .
0
(12)
2 0

From (12) it is clear that Fc (u) = Fc (u) = so that Fc (u) is an even function and
Fc(u) cos u x is even and Fc(u) sin u x is odd. Then the second integral on the right of
(11) is zero and the result can be written

2 2
f(x) = Fc (u) cos ux du = Fc (u) cos ux du . (13)
2 0 0

and we call Fc(u) and f(x) Fourier cosine transforms of each other.
A similar result holds for odd functions

2
Fs(u) = f (t) sin ut dt .
0
(14)

102
Advanced Engineering Mathematics Chapter 4- Fourier Integrals

2
f(x) = Fs (u) sin ux du . (15)
0

Example 4.2:
1 x a

Find the Fourier integral of the function f (x) 1 x =a
2
0 x a

Solution
It is clear that f(x) - an even function, then the Fourier transform of f (x) is

2 2a 2 sin ux a 2 sin ua
Fc(u) = f (t) cos ut dt = cos ut dt = = , u 0.
0 0 u 0 u

2 2a 2
For u = 0, we obtain Fc(0) = f (t) dt = dt =
0
a.
0

The graphs of f(x) and Fc(u) for a = 3 are shown in the figure below

103
Advanced Engineering Mathematics Chapter 4- Fourier Integrals

2 sin ua 2 2 sin ua
Since we have Fc (u) =
u
, then f(x) =
0 u
cos ux du

1 x a
2 sin ua cos ux
i.e. du = 1 x = a.
0 u 0
2
x a

x a
sin ua cos ux
Hence 2 du = x =a
u 2
0 0 x a

sin u sin u
Putting x = 0 and a = 1, we get 2 du = du = .
0 u u

Example 4.3:
Solve the integral equation


sin 2 u
1 u 0 u 1
f (x) cos ux dx = 0 u >1
and then prove that 2
du = .
2
0 0 u
Solution:
2
2 1 u 0 u 1
f (x) cos ux dx = Fc(u), and then Fc(u) =
0

0 u >1

2 2 1 2 21
Then f(x) = Fc (u) cos ux du = (1 u)cos uxdu = (1 u)cos uxdu =
0 0 0

2(1 cos x)
= .
2
x
2
2 2(1 cos x)
cos ux dx = 1 u
0 u 1.
Hence
0
2
x 0 u >1
(1 cos x)
Taking the limit as u 0+ , we find dx = .
2 2
0 x
But this integral can be written as :

104
Advanced Engineering Mathematics Chapter 4- Fourier Integrals

2
(1 cos x) 2sin x sin 2 u
2
2
du =
2
dx =
2
du =
2
0 x 0 x 0 u

Example 4.4:
ax
Find the Fourier cosine (sine) transform of f(x) = e (a > 0, x > 0), and then prove
cos ux x u sin ux x
that du = e , x 0, 2 du = e , x 0.
2 2 2
0 u 1 0 u 1

Solution:
From (12) we determine the Fourier cosine transform

2 2 at
Fc(u) = f (t) cos ut dt =
0 0
e cos ut dt =

at
2 e 2 a
= [a cos ut + u sin ut] 0 = ,
a2 u2 a2 u2

and from (13) , we get

2 2 a 2a cos ux
f(x) =
0 a2 u2
cos ux du =
0 a2 u2
du ,

2a cos ux ax
i.e.
0 a2 u2
du = e (i)

From (14) we determine the Fourier sine transform:

2 2 at
Fs(u) = f (t) sin ut dt =
0 0
e sin ut dt =

at
2 e 2 u
= [a sin ut u cos ut] = ,
a2 u2 0 a2 u2

2 2 u
and therefore f(x) =
0 a2 u2
sin ux du ,

2 u sin ux ax
i.e.
0 u a
2 2
du = e (ii)

105
Advanced Engineering Mathematics Chapter 4- Fourier Integrals

In (i), (ii) put a = 1, we get


cos ux x
2
du =
2
e , x 0
0 u 1
u sin ux x
2 du = e ,
2
x 0.
0 u 1

4.3 Parseval's identities for Fourier integrals:


If Fc(u) and Gc(u) are Fourier cosine transforms of f(x) and g(x) respectively,

then: Fc (u) G c (u)du = f (x) g(x) dx (16)
0 0

Similarly if Fs(u) and Gs(u) are Fourier cosine transforms of f(x) and g(x), then

Fs (u) Gs (u)du = f (x) g(x) dx (17)
0 0
In the special case where f(x) = g(x), (16) and (17) becomes respectively :

2 2
Fc (u) du = f (x) dx , (18)
0 0

2 2
Fs (u) du = f (x) dx , (19)
0 0

Relations (16) (19) are known as Parseval's identities for integrals. Similar relations
hold for general Fourier transforms. Thus if F(u) and G(u) are Fourier transforms of
f(x) and g(x) respectively, we can prove that

F(u) G(u) du = f (x) g(x) dx , (20)

where the bar signifies the complex conjugate.

Example 4.5:
Verify Parseval's identity for Fourier integral for the Fourier transforms of
Example 2.2.
Solution:

106
Advanced Engineering Mathematics Chapter 4- Fourier Integrals


2 2
We must show that f (x) dx = F(u) du ,

1 x a
2 sin ua
where f (x) = 1 x = a and F (u) =
0
2 u
x a

a
2 2
L.H.S. = f (x) dx = 1 dx = 2a
a

2 2
2 2 sin ua 2a sin v 2a
R.H.S. = F(u) du = du = dv = . = 2a.
u
2 v
2

(Using the result of Example 2.3) then L.H.S = R.H.S.


sin 2 u
The method can also be used to find du , directly.
2
u

4.4 The convolution theorem:


If F(u) and G(u) are the Fourier transforms of f(x) and g(x) respectively, then

iux
F(u)G(u)e du = f (t) g(x t) dt (21)

If we define the convolution, denoted by f* g , of the functions f and g to be

1
f*g = f (u) g(x u) du . (22)
2

Then (21) can be written in the form F { f*g } = F{f} F{g}

i.e. the Fourier transform of the convolution of two functions is equal to the product
of their Fourier transforms. This is called the convolution theorem for Fourier
transforms.

Example 4.6:
Solve for g(x) the integral equation
g(u) 1
2 2
du =
2 2
; a, b are constants and 0 < a < b.
(x u) a x b

107
Advanced Engineering Mathematics Chapter 4- Fourier Integrals

Solution:
Taking the Fourier transform of both sides of the given integral equation, we have
the convolution theorem
1 1
2 F {g} F { }=F{ }, (i)
2 2 2 2
x a x b
iut
1 1 e 2 cos ut 1 2 cos buv
we have F { }= dt = dt = dv =
2
x b
2 2 t 2 b 2 0 t 2 b2 b 0 v2 1

1 2 ub 1 ub
= .e = .e .
b 2 b 2
cos ut sin ut iut
(Since is even and is odd and e = cos ut + i sin ut ).
2 2 2 2
t b t b

1 1 ub
Similarly: F { }= .e
2 2 b 2
x b
Substituting in (i) we get
1 ua 1 bu
2 F {g} .e = e .
a 2 b 2
a (a b)u
Then G(u) = e .
b 2
Thus

1 iux a (a b)u iux


g(x) = e G(u) du = 2b e e du =
2
(a b)u
a (a b)u a e
= e cos ux du = [(b a)cos ux u sin ux] =
b 0 b (b a) x
2 2 0

a (b a)
= .
b (b a) 2 x 2

a(b a)
i.e. g(x) = .
b x (b a)
2 2

108
Advanced Engineering Mathematics Chapter 4- Fourier Integrals

Exercises 4.1
1. Prove that the Fourier transforms of
1 x 2 x <1 , is 2 2 u cos u sin u and then prove that
f(x) =
0 x >1 u
3

sin x x cos x x 3
3 cos dx =
2 16
.
0 x

2. If f(x) =
1
0
0 x <1
x 1
, find

(a) Fourier sine transform, (b) Fourier cosine transform of f(x).


2 1 cos u 2 sin u
[Answers: a) , b) ]
u u

1 0 u 1

3. Solve for g(x) the integral equation g(x) sin ux dx = 2 1 u 2 .
0 0 u2

[Answers: g(x) = (2 + 2 cos x 4 cos2x)/( x).]


x
4. Using the Fourier sine and cosine transforms of e , x > 0 , and Parseval's identity
2
dx x dx
to prove that: a) = , b) = .
2 2 4 2 2 4
0 (x 1) 0 (x 1)

1 cos x 2 sin 2 x

5. Use problem (2) to show that a) dx = , b) dx = .
2
0 x 2 0 x 2

x cos x sin x 2
6. Show that dx = .
6 15
0 x
7. Find the Fourier transform of the following functions:
x x 1
sgn x x 1 x x 1
a) f(x) = , b) f(x) = c) f (x) 0 x >1 ,
0 x 1 0 x 1 1 ,
2 x 1

109
Advanced Engineering Mathematics Chapter 4- Fourier Integrals

cos x
x sin x
x
d) f(x) = 2 e) f(x) = 2
x x
0 2 0 2

2 1 cos u
[Answers: a) f(x) = u sin ux du , x 1,
0

2 sin u cos u
b) f(x) = sin ux du , x 1,
0 u 2 u

2 sin u cos(u 1)
c) f(x) = cos ux du ,
0 u u
2

u
2 cos 2
d) f(x) =
0 1 u2
cos ux du ,

2 sin u
e) f(x) =
0 1 u2
sin ux du .]

110
Advanced Engineering Mathematics Chapter 5 - Laplace transforms

CHAPTER 5
LAPLACE TRANSFORMS
5.1 The Laplace Integral:
Let f (t) be a function of t specified for all t, 0 < t < , and consider
T
lim lim est f (t) dt , (7.1)
0 T

for some complex s. If the double limit exists, we shall write this as the improper
integral

st
e f (t) dt , (7.2)
0
and denote it by L{f (t)} = F(s) .

If the function of t is indicated in terms of a small (lower case) letter, such as


f(t), g(t), y(t) , etc., the Laplace integral or the Laplace transform of the function is
denoted by the corresponding capital letter, i.e. F(s), G(s), Y(s), etc. In other cases, a
tilde (~) can be used to denote the Laplace integral (or the Laplace transform). Thus,
for example, the Laplace transform of f(t) is f (s) or L{f(t)} or F(s). We shall write

f(t) . . F(s) to denote that L{f(t)} = F(s).

Definition: Exponential order


A function f(t) is said to be of exponential order for t > T if we can find
t
constants M and such that |f(t)| M e for t > T .

Theorem 5.1
If f(t) is piecewise continuous in every finite interval 0 t T and is of exponential
order for t > T, then the double limit (7.1) exists.
It must be emphasized that the stated conditions are sufficient to guarantee the
existence of the Laplace transform. If the conditions are not satisfied, however, the
Laplace transform may or may not exist. Thus the conditions are not necessary for the
existence of the Laplace transform.

111
Advanced Engineering Mathematics Chapter 5 - Laplace transforms

5.2 Some important properties of Laplace transforms:


In the following list of theorems we assume that all functions f(t) possesses a
Laplace transform i.e. the double limit (7.1) exists.
1. Linearity property:
Theorem 5.2:
If L{f(t)} = F(s) and L{g(t)} = G(s), then for any constants a and b

L{a f(t) + b g(t)} = a F(s) + b G(s). (7.3)

Proof:

L{a f(t) + b g(t)} = est [ a f (t) b g(t)] dt =
0

st
= a e f (t) dt + b est g(t) dt .
0 0
The result is easily extended to more than two functions. This implies that the taking
of a Laplace transform is a linear operation.
2. Translation property or Shifting property:
Theorem 5.3: First translation theorem
at
If L{f(t)} = F(s), then L{e f(t) } = F(s a).

Proof:

st at
f (t) dt = es(t a) f (t) dt = F(s a).
at
L{e f(t) } = e e
0 0

Theorem 5.4 Second translation theorem


f (t a) t>a as
If L{f(t)} = F(s), and g(t) = , then L{g(t) } = e F(s).
0 t<a

Proof:
a
st st
L{g (t) } = e g(t)dt = e g(t)dt + est g(t)dt =
0 0 a
a
= est (0)dt + est f (t a)dt = es(u a) f (u)du =
0 a 0

112
Advanced Engineering Mathematics Chapter 5 - Laplace transforms


sa su as
=e e f (u)du = e F(s).
0

Theorem 5.5 Change of scale property


1 s
If L{f (t)} = F(s), then L{f (at)} = F( ).
a a
Proof:
s ( u ) su 1 s
st u 1
L{f (at)} = e f (at)dt = e a f (u) d( ) = ( ) e a f (u) du = F( ).
a a a a
0 0 0

Theorem 5.6 Laplace transform of derivatives:


If f (t) is continuous for 0 t N and of exponential order for t > N while f '(t) is
piecewise continuous for 0 t N and L{f(t)} = F(s), then L{f '(t)} = s F(s) f(0).

Proof:
Using integration by parts, we have:
T st T T
L{f '(t)} = e f '(t)dt = lim e f '(t)dt = lim e f (t) s est f (t)dt =
st st
0 T 0 T 0 0
sT st

T
st
= lim e f (T) f (0) s e f (t)dt = s e f (t)dt f (0) lim esT f (T) =
T 0 0 T

= s est f (t)dt f (0) = s F(s) f (0).
0
Using the fact that f(t) is of exponential order as t , so that lim esT f (T) = 0
T

for s > .

Theorem 5.7:
If in Theorem 5.6, f (t) fails to be continuous at t = 0, but lim f (t) =f(0+) exists (but
t 0
is not equal to f(0), which may or may not exist) , then L{f '(t)} = s F(s) f(0+).

Theorem 5.8:
If in Theorem 5.6, f (t) fails to Continuous at t = a, then:
as
L{f '(t)} = s F(s) f(0) e [f(a+) f(a)],

113
Advanced Engineering Mathematics Chapter 5 - Laplace transforms

where [f (a+) f (a)] is called the jump at the discontinuity t = a.

Theorem 5.9:
If f (t) and f '(t) are continuous for 0 t N and of exponential order for t > N while
f "(t) is piecewise continuous for 0 t N, and L{f (t)} = F(s), then
2
L{f "(t)} = s F(s) s f (0) f '(0).

Proof:
By Theorem 5.6: L{g '(t)} = s G(s) g (0). Let g (t) = f '(t). Then

L{f "(t)} = s G(s) g(0) = s L{g(t)} f '(0) = s L{f '(t)} f '(0) =


2
= s{s F(s) g (0)} f '(0) = s F(s) s f(0) f '(0).
The generalization to higher order derivatives can be proved by using
Mathematical induction. If f (t) and f '(t) have discontinuities, appropriate
modification can be made as in Theorems 7.7 and 7.8.

Theorem 5.10:
(n1)
If f (t), f '(t),, f (t) are continuous for 0 t N and of exponential order for
(n)
t > N while f (t) is piecewise continuous for 0 t N, and L{f (t)} = F(s), then
(n) n n1 n2 (n2) (n1)
L{f (t)}= s F(s) s f (0) s f '(0) s f (0) f (0).

Theorem 5.11: Multiplication by powers of t:


If L{f (t)} = F(s), then

n n dn n (n)
L{t f (t)} = (1) F(s) = (1) F (s), where n = 1, 2, 3, .
n
ds
Proof:

We have F(s) = est f (t)dt . Then by Leibnitz's rule for differentiating under the
0
integral sign,
dF d st
st

= F '(s) = e f (t)dt = e f (t)dt = est {t f (t)}dt = L{t f (t)}.
ds ds 0 0 s 0

114
Advanced Engineering Mathematics Chapter 5 - Laplace transforms

dF
Thus L{t f (t)} = = F '(s), which proves the theorem for n = 1.We shall using
ds
Mathematical induction. Assume the theorem true for n = k, i.e. assume

st k k (k)
e [t f (t)]dt = (1) F (s) (1)
0
d st k
Then e [t f (t)]dt = est {t k 1 f (t)}dt ,
ds 0 0

i.e. est {t k 1 f (t)}dt = (1)
k+1 (k+1)
F (s) (2)
0
It follows that if (1) is true, i.e. if the theorem holds for n = k, then (2) is true, i.e. the
theorem holds for n = k + 1. But the theorem is true for n = 1. Hence it is true for
n = 2 and n = 3, etc., and thus for all positive integer value of n.

Theorem 5.12 Laplace transform of integrals:


t F(s)
If L{f (t)} = F(s) , then L{ f (u)du } = .
0 s

Proof:
t
Let g(t) = f (u)du . Then g '(t) = f (t) and g (0) = 0. Taking the Laplace transform of
0
both sides, we have L{f (t)} = L{g '(t)} = s L{g (t)} g(0) = s L{g(t)}.

1 F(s) t F(s)
Thus L{g (t)} = L{f (t)} = , or L{ f (u)du } = .
s s 0 s

Theorem 5.13 Division by t:


f (t) f (t)
If lim exists and L{f (t)} = F(s), then L{ } = F(u)du .
t 0 t t s
Proof:
Let g (t) = f (t) / t or f (t) = t g (t) ,
d d
F(s) = L{f (t)} = L{t g (t)} = L{g (t)} = G(s).
ds ds
s c
Thus G(s) = F(u)du = F(u)du .
c s

115
Advanced Engineering Mathematics Chapter 5 - Laplace transforms

Now since g (t) satisfies the conditions of Theorem 5.1, it follows that lim G(s) = 0.
s
f (t)
Then c must be infinite and so G(s) = L{ } = F(u)du .
t s

Theorem 5.14 Convolution theorem:


t
If L{f (t)} = F(s), L{g (t)} = G(s), then L{ f (u)g(t u)du } = F(s) G(s).
0
We call the above integral the convolution of f and g; and write
t
f * g = f (u)g(t u)du .
0
5.3 Laplace Transforms of elementary functions:
Example 5.1:
2
Prove that (a) L{1} = 1/s, s > 0, (b) L{t} = 1/s , s > 0,
at
(c) L{e } = 1/(s a), s > a

Solution:
T T
st st st (1 e sT ) 1
(a) L{1} = e (1)dt = lim e dt = lim e = lim = , s > 0.
0 T 0 T ( s) 0 T s s
T st st
T
(b) L{t} = e st
(t)dt = lim t e st
dt = lim t e (1) e 2 =
0 T 0 T ( s) s 0
1 st sT 1
= lim e 2 T e = , if s > 0.
T s 2 s s
s2
T T
(s a)t
st at (s a)t
dt = lim e
at
(c) L{e } = e e dt = lim e =
0 T 0 T (s a) 0
(s a)T 1
= lim 1 e = , if s > a.
T (s a) s a
Another method:
It is clear that L{f ' (t)} = s L{f (t)] f (0) ( see Theorem 5.6 )

(a) Let f(t) = 1 , then f '(t) = 0 , f (0) = 1 then L{0} = 0 = s L{1} 1 or L{1} = 1/s.

116
Advanced Engineering Mathematics Chapter 5 - Laplace transforms

(b) Let f(t) = t , then f '(t) = 1 , f (0) = 0, using part (a) we get L{1} = 1/s = s L{t} 0
2
or L{ t } = 1/ s .

By using mathematical induction we can similarly


n (n+1)
show that L{ t } = n!/s for any positive integer n.
at at at at
(c) Let f(t) = e . Then f '(t) = a e , f (0) = 1, then L{a e } = s L{e } 1.
at at at
i.e. a L{e } = s L{ e } 1 or L{ e } = 1/(s a).

Example 5.2:
a s
Prove that (a) L{ sin at } = , (b) L{ cos at } = , s > 0,
a 2 s2 a 2 s2
Solution:
T
a) L{ sin at } = est sin at dt = lim est sin at dt =
0 T 0
esT [ssin aT a cos aT] a a
= lim =
T
s2 a 2 s 2 a 2 a 2 s 2

Another method:
2
i) From Theorem 5.9 we have L{f "(t)} = s L{ f (t)} s f(0) f '(0).
2
(a) Let f (t) = sin at, f "(t) = a sin at, f (0) = 0, f '(0) = a,
2 2
then L{ a sin at} = = s L{sin at} a.
2 2 a
i.e. (s + a ) L{sin at} = a or L{sin at} = .
a 2 s2
2
(b) Let f (t) = cos at, f "(t) = a cos at, f (0)= 1, f '(0) = 0 , then
2 2
L{ a cos at} = s L{cos at} s.
2 2 s
i.e. (s + a ) L{cos at} = s or L{sin at} = .
a 2 s2
ii) From Example 5.1 part (c) we have
iat 1 s ia
L{e } = = . (*)
s ia s 2 a 2
iat
But e = cos at + i sin at. Hence
117
Advanced Engineering Mathematics Chapter 5 - Laplace transforms


st st
cos atdt i est sin atdt =
iat
L{e } = e [cosat isin at]dt = e
0 0 0
s ia
= L{cos at} + i L{sin at} = .
s2 a 2
On equating real and imaginary parts, we get:
s a
L{cos at} = , L{sin at} = .
s2 a 2 s2 a 2

Example 5.3:
Using the linearity property to prove that:
a s
(a) L{sinh at} = , (b) L{cosh at} = if s > |a|.
2 2 2 2
s a s a
Solution:
at at at at
a) L{sinh at} = L{(e e )} = L{e } L{e } =
1 1 1 a
= = for s > |a|.
2 s a s a s 2 a 2
at at at at
b) L{cosh at} = L{(e + e )} = L{e } + L{e } =
1 1 1 s
= = for s > |a|.
2 s a s a s 2 a 2
Short Table Of Special Laplace Transforms
o
N f(t) L{f(t)} = F(s) conditions

1 1 1/s s>0
2
2 t 1/s s>0
3 tn n!/sn+1 s > 0, n=0,1,2,
4 eat 1/(sa) s>a
2 2
5 sin at a/(s + a ) s>0
2 2
6 cos at s/(s + a ) s>0
2 2
7 sinh at a/(s a ) s > |a|
2 2
8 cosh at s/(s a ) s > |a|
p (p+1)
9 t (p+1)/s s > 0 , p > 1

118
Advanced Engineering Mathematics Chapter 5 - Laplace transforms

Example 5.4:
p (p 1)
Prove that: L{ t } = , if s > 0 and p > 1.
p 1
s
Solution:

L{ t } = est t p dt . Let s t = u and note that in order for the integral to converge we
p

0
p 1 p u (p 1)
must have s > 0. Then L{ t } = u e du =
s p 1 0 s p 1
the restriction p > 1 occurs because the integral defining the gamma function
converges if and only if p > 1.

Example 5.5:
Find the Laplace transforms of each of the following:
3t 2 5
a) 4 e , b) 3 t , c) 5 cos 4t , d) sin t , e) L{ }.
t
Solution:
3t 3t 4 4
a) L{4 e } = 4 L{e }= , s > 3.
s (3) s 3
2 2 2! 6
b) L{3 t } = 3 L{ t } = 3 = , s > 0.
3 3
s s
s 5s
c) L{5 cos 4t} = 5 L{cos 4t} = 5 .
2 2 2
s 4 s 16

d) L{ sin t} = , s > 0.
s 2 2
5
( 1 )
e) L{ } = 5 L{ t } = 5 5 2 5 , s > 0.
t s s

Example 5.6:
Find the Laplace transforms of each of the following:
t 2 t 2 3t 4t
a) e cos 2t, b) 4t 3cos2t + 5 e , c) t e , d) e cosh 5t,
2t 2
e) e (3 cos 6t 5 sin 6t), f) t sin at , g) t cos at .
Solution:
a) Using Theorem 5.3 we get
119
Advanced Engineering Mathematics Chapter 5 - Laplace transforms

t s 1 s 1
L{e cos 2t} = F(s+1) = ,
(s 1) 2 4 s 2 2s 1
2 t 2 t
b) L{4t 3cos2t + 5 e } = 4L{ t } 3 L{ cos2t } + 5L{e } =
2! 1 8 s 3s 5
=4 3
5 ,
s3 s2 4 s 1 s3 s 2 4 s 1
2 3t 2 2! 2
c) L{t e } = F(s3) = , where F(s) = = L{t },
(s 3)3 s3
s 4t s4 s4
d) L{cosh 5t} = . Then L{e cosh 5t} = ,
s 2 25 (s 4)2 25 s 2 8s 9
e) L{3 cos 6t 5 sin 6t} = 3L{cos 6t} 5L{sin 6t} =
s 6 3s 30
=3 5 .
2 2 2
s 36
s 36 s 36
2t 3(s 2) 30 3s 24
Then L{e (3 cos 6t 5 sin 6t)} = F(s+2) = = ,
(s 2)2 36 s 2 4s 40
a
f) Since L{sin at} = , we have
2 2
s a
d a 2as
L{t sin at} = ,
ds s 2 a 2 2
s a 2 2

s
g) Since L{cos at} = , we have
2 2
s a
'
"

2 d2 s s a 2 s 2 2s(s 2 3a 2 )
L{t cos at} = 2 = = = .
ds 2 s a 2 s2 a 2 s2 a 2 2

2
s a 2 3

Example 5.7:
f (t)
a) Prove that dt F(s) ds provided that the integrals converge.
0 t 0
sin t
b) Show that dt .
t 2
0
Solution:

st f (t)
a) From Theorem 5.13, e dt = F(u)du .
0 t s

120
Advanced Engineering Mathematics Chapter 5 - Laplace transforms

Then taking the limit as s 0+, assuming the integrals converge and the required
f (t)
result is obtained. i.e. dt F(s) ds .
0 t 0
2
b) Let f (t) = sin t . Then F(s) = 1/(s + 1).
sin t ds
Hence dt = tan 1 s .
2 2
0 t 0 s 1 0

Example 5.8:
Find L{f (t)} if
cos(t )3 t>
3 0<t<5 3 3
a) f(t) = b) f(t) =
0 t 5 0 t
3

Solution:
5
5 est
a) By definition, L{f (t)} = est f (t)dt = est (3)dt est (0)dt = 3 =
0 0 5 s
0
3(1 e5s )
= ,
s

3 s(u )
st st
b) L{f (t)} = e (0) dt e cos(t )dt = e 3 cos udu =
3
0 0
3
s su s s
=e 3
e cos udu =
2
e 3 .
0 s 1

Theorem 5.15: Periodic functions:


1 T
st
Let f (t) have period T > 0 then L{f (t)} e f (t)dt .
(1 esT ) 0
Proof:
f (t) periodic function with period T, then f(u) = f(u + T) = f(u + 2T) == f(u + n T).
We have
T 2T 3T
st st st
L{f (t)} = e f (t)dt = e f (t)dt e f (t)dt est f (t)dt .
0 0 T 2T
In the second integral let t = u + T, in the third integral let t = u + 2T, etc. Then

121
Advanced Engineering Mathematics Chapter 5 - Laplace transforms

T T T
su s(u T)
L{f (t)} = e f (u)du e f (u T)du es(u 2T) f (u 2T)du
0 0 0
T T T
= esu f (u)du esT esu f (u)du e2sT esu f (u)du =
0 0 0
T 1 T
+] esu f (u)du = su
sT 2sT 3sT
= [1 + e +e +e
sT e f (u)du .
0 1 e 0

Example 5.9:
Find the Laplace transform of the function of period 2 which in the interval
sin t 0 t<
0 t < 2 is given by f (t) .
0 t < 2

Solution:
The graph of this function, often called a rectified sine wave or a half wave rectified
sine curve.

2s
Since T = 2, let R = 1/(1 e ) , we have:

su
2
st
est (ssin t cos t)
L{f(t)} = R e f (u)du = R e sin t dt = R =
2
0 0 s 1 0
1 es 1
=R = .
2 s 2
s 1 (1 e )(s 1)

Theorem 5.16: Behaviour of F(s) as s :


If L{f(t)} = F(s) , then lim F(s) = 0.
s

Theorem 5.17: Initial-value and Final-value theorem:


If the indicated limits exists, then

122
Advanced Engineering Mathematics Chapter 5 - Laplace transforms

a) lim f(t) = lim s F(s) , b) lim f(t) = lim sF(s).


t 0 s t s 0

Example 5.10:
3t
Illustrate Theorems 7.16 and 7.17 for the function f(t) = 2 e .
Solution:
3t 3t
We have f(t) = 2 e , F(s) = L{2 e } = 2/(s+3).

It is clear that:
2
lim F(s) lim = 0, (1)
s s s 3
2s
lim sF(s) lim = 2, (2)
s s s 3
2s
lim sF(s) lim = 0, (3)
s 0 s 0 s 3
lim f (t) lim 2e3t = 2, (4)
t 0 t 0
lim f (t) lim 2e3t = 0, (5)
t t
i.e. lim F(s) 0 , lim f (t) = lim sF(s) ; lim f (t) = lim sF(s)
s t 0 s t s 0
5.4 Some Special Functions:
I. Bessel Functions:
We define a Bessel function of order n by
tn t2 t4
Jn (t) = 1 .
n
2 (n 1) 2.(2n 2) 2.4.(2n 2)(2n 4)
Some important properties are
n
1. Jn (t) = (1) Jn(t) if n is a positive integer.
2n
2. Jn+1(t) = Jn (t) Jn1(t),
t
3.
d n
dt

t J n (t) t n J n 1 (t) , J '0 (t) = J1(t).

t(u 1)
4. exp = J n (t)u n .
2u n
This is called the generating function for the Bessel functions.
5. Jn (t) satisfies Bessel's differential equation:

123
Advanced Engineering Mathematics Chapter 5 - Laplace transforms

2 2 2
t y"(t) + t y'(t) + (t n ) y(t) = 0.
n
It is convenient to define Jn (it) = i In(t), where In(t) is called the modified Bessel
function of order n.

Example 5.11:
a) Find L{Jo (t)} and then use it to find a) L{Jo(at)}, b) Find L{J1(t)}.

Solution:
It is clear that
t2 t4 t6
Jo(t) = 1 .
22 22.42 22.42.62
Then by using the binomial theorem, we get
1 1 2! 1 4! t6 6!
L{ Jo (t)} = =
s 22 s3 22.42 s5 22.42.62 s7
1
1 1 1 1.3 1 1.3.5 1 1 1 2 1
= 1 = 1 = ,
s 2 s 2 2.4 s 4 2.4.6 s6 s s2 s 1 2
1 s
To find L{Jo (at)} we use Theorem 5.5, i.e. L{f (at)} = F( ) .
a a
1 1
L{Jo (at)} = = .
2 2 2
s s a
a 1
a
b) Since J '0 (t) = J1(t), L{f ' (t) } = s F(s) f(0), then

s s2 1 s
L{J1 (t)} = L{ J '0 (t) } = [ s L{ Jo(t)} 1] = 1 = .
2 2
s 1 s 1
Another method: Using differential equations,
a) The function Jo(t) satisfies the differential equation, t J"o (t) + J 'o (t) + t J o (t) = 0 .
Taking the Laplace transform of both sides and using
Jo (0) = 1, J 'o (t) = 0, y = L{ Jo(t)}, we have
d 2 dy
s y s.1 0 [sy 1] 0,
ds ds

124
Advanced Engineering Mathematics Chapter 5 - Laplace transforms

dy sy dy sds
from which . Thus and by integrating we get
ds s2 1 y s2 1
c
y= .
2
s 1
cs
Now lim sy(s) lim c , and lim J o (t) 1 . Thus by the initial-value
s s s 2 1 t 0
1
Theorem 5.17(a), we have c = 1 and so L{Jo (t)}= .
2
s 1
II The Unit Step Function:
The unit step function, also called Heaviside's unit step function, is defined as

0 t<a
U(t a) =
1 t>a

It is possible to express various


discontinuous functions in terms of the
unit step function. It is clear that:


st st
a
st
est e as
L{U(ta)} = U(t a)e dt e (0)dt e (1)dt = 0+ = , if s > 0.
0 0 a s s
a

Example 5.12:
4 t<3
Express the function f (t) = , in terms of the unit step function and thus
2 t>3

obtain L{f (t)}.

Solution:
0 t<3 0 t<3
We have f (t) = 4 + =42 = 4 2 U(t3).
2 t>3 1 t>3
4 e3s 4 2e3s
L{f(t)} = L{4 2 U (t3)} = L{4} 2 L{U (t3)}= 2 = .
s s s
It is clear that, the second translation Theorem 5.4 can be written as follows:

125
Advanced Engineering Mathematics Chapter 5 - Laplace transforms

Theorem 5.4':
If L{f (t)} = F(s) and g (t) = f (t a) U (t a), then
as
L{g (t)} = L{f (t a) U (t a)} = e F(s).

Example 5.13:
0 t<0

Find L{f (t)} where f(t) = t 0 < t < a , a > 0.
a t>0

Solution:
L{f(t)}=
a
st st
f (t)e dt = t e dt + a est dt =
0 0 a
a
test est aest
= =
s 2 s
a
0 a
aeas aeas 1 aeas 1 eas
= = .
s s2 s2 s s2
Using the unit function, we can express f(t) as follows :
f(t) = t [ U(t) U(t a)] + a U(t a) = t U(t) (t a)U(t a).
1 eas
Hence L { f (t) } = .
s2 s2

Example 5.14:
0 t<0
Find L { f(t) } where f(t) = , n = 0, 1, 2, 3,
n na < t < (n+1)a

Solution:

126
Advanced Engineering Mathematics Chapter 5 - Laplace transforms


L{f(t)} = f (t)est dt =
0
(n 1)a
(n 1)a ne st
= nest dt
n 0 na n 0 s na
n
= e nas e(n 1)as =
n 0 s

1 nas
= e
s n 0
eas 1
= = .
s(1 eas ) s eas 1
We can express f(t) as follows:
f(t) = [U(t a) U(t 2a)] + 2[U(t 2a) U(t 3a)] + +
+ n [U(t n a) U(t {n+1}a)] + ... =

= U(t a) + U(t 2a) + U(t 3a) ++ U(t n a) = U(t na) .
n 1
e nas e as
1
and L{f(t)} = L{U(t na)} = = = .
n 1 n 1 s s(1 eas ) s(eas 1)
III The Unit Impulse Function:
The unit impulse function, also called
Dirac delta function. Consider the function
1 / 0 t
F(t) =
0 t>

where > 0

It is geometrically evident that as 0 the height of the rectangular region increases


indefinitely and the width decreases in such a way that the area is always equal to 1,

i.e. F (t)dt 1 .
0

127
Advanced Engineering Mathematics Chapter 5 - Laplace transforms

This idea has led some engineers and physicists to think of a limiting function
denoted by approached by F (t) as 0. This limiting function they have called the
impulse function of Dirac delta function. Some of its properties are

1) (t)dt = 1, 2) (t)g(t)dt = g(0), 3) (t a)g(t)dt = g(a),
0 0 0
for any continuous function g (t).
It is clear that
1 st
st st 1 st st
L{F(t)} = e F (t)dt = e ( )dt + e (0) dt = ( ) e dt = 1 e ,
0 s
0 0
1 (1 s s 2 2 / 2! ) s
and lim L{F (t)} lim = lim (1 ) = 1.
0 0 s 0 2!
Although mathematically speaking such a limit: lim F(t) does not exist, so
0
that L{ lim F(t) } is not defined, manipulations or operations using it can be made
0
rigorous. It proves useful to consider (t) = lim F(t) to be such that L{(t) } = 1.
0

Example 5.15:
as
Show that L{(ta)} = e .

Solution:

st
Let g(t) = e and from (t a)g(t)dt = g(a), we get
0

L{(ta)} = (t a)est dt = e .
st

0
IV Null Functions:
t
If N (t) is a function of t such that for all t > 0, N(u)du = 0
0
1 t=1
4
We call N(t) a null function. The function f (t) = 1 t=2
0 otherwise

128
Advanced Engineering Mathematics Chapter 5 - Laplace transforms

is a null function.
In general, any function which is zero at all but a countable set of points is a null
function.

Example 5.16:
Indicate which of the following null functions are:
12 t=0
2 t = 3
a) f(t) = , b) f(t) = 1 1 t 0 , c) f(t) = (t).
2 otherwise 1 otherwise
2
Solution:
t
a) Since f (u)du = 0, for all t > 0, f(t) is a null function.
0
t
b) f (u)du = 0, if t < 1,
0
t t
f (u)du = (1)du = t 1, if 1 t 2,
0 1
t 2
f (u)du = (1)du = 1, if t > 2.
0 1
t
Since f (u)du 0 for all t > 0, f(t) is not a null function.
0
t
c) (u)du = 1, for all t > 0, (t) is not a null function.
0

Example 5.17:
Find L{f(t)} where f(t) is given as shown in the figure.

Solution:
It is clear that
t 1 1 t 2

f(t) = 1 2 t 4,
5 t 4 t 5

129
Advanced Engineering Mathematics Chapter 5 - Laplace transforms

2 4 5
st st st
L{f(t)} = f (t)e dt = (t 1)e dt e dt (5 t)est dt =
0 1 2 4
s 2s 5s 4s
e e e e
= .
2 2 2 2
s s s s
We can express f(t) by using the unit step function U(t a) as follows :
f(t) =(t 1)[U(t 1) U(t 2)]+[U(t 2) U(t 4)] + (5 t)[U(t 1) U(t 2)]=
= (t 1)U(t 1) (t 2)U(t 2) (t 4)U(t 4) + (t 5)U(t 5).
e s e2s e5s e4s
Then L{f (t)} = .
2 2 2 2
s s s s

Example 5.18:
Find L{f (t)} where f (t) is the square wave function
1 2na < t < (2n+1)a,

f (t) = f (t + 2a) = 1 (2n+1)a < t < (2n+2)a, ,
1 t < 0,

n = 0, 1, 2, 3, .
Solution:
f (t) = [U(t) U(t a)] [U(t a) U(t 2a)] + [U(t 2a) U(t 3a)] =
= U(t) 2U(t a) + 2 U(t 2a) 2U(t 3a)+ =

= U (t) 2 (1) n 1U(t na) .
n 1
L{f (t)} = L{ U(t)}

2 (1)n 1 L{U(t na)} =
n 1
nas
1 n 1 e
= 2 (1) =
s n 1 s
1 2eas
= 2 .
s as
s(1 e )

L{f (t)} = L{U (t)} 2 (1) n 1 L{U(t na)} =
n 1

130
Advanced Engineering Mathematics Chapter 5 - Laplace transforms

nas
1
n 1 e 1 2eas
= 2 (1) = 2 .
s s s as
n 1 s(1 e )

Example 5.19:
Find L{f (t)} where f (t) is saw tooth wave function
t
n, na < t < (n+1)a
f(t) = f(t + a) = a , n = 0, 1, 2, 3, .
0, t<0
Solution:
f (t) is a periodic function,
then
a a
st
e f (t)dt est t dt =
a
0 0

1 1 st a 1 1 1 1 as 1 eas eas 1 eas eas


= t e (a )e = = .
as s 0 as s as s as 2 s as 2 as 2 s
1 eas eas as 1 eas
L{f (t)} = / (1 e ) = 2 .
2 s as
as as
s(1 e )

Example 5.20:
t sin u 1 1 1
Prove that L{ du} tan ( )
0 u s s

Solution:
t sin u
Let f (t) = du . Then f(0) = 0 and
0 u
sin t
f '(t) = or t f '(t) = sin t. Taking the Laplace transform,
t
d 1
L{ t f '(t)} = L{sin t} or [sF(s) f (0)] = .
ds s2 1
d 1
i.e. [sF(s)] = .
ds 1 s2
1
Integrating we get s F(s) = tan s + C. By the initial value Theorem 5.17(a),

131
Advanced Engineering Mathematics Chapter 5 - Laplace transforms

lim s F(s) = lim f(t) = f(0) = 0, so that C = /2.


s t 0
1 1 1
Thus s F(s) = /2 tan s = tan (1/s) or F(s) = tan (1/s) / s.
5.5 The Inverse Laplace Transform:
If the Laplace transform of a function f(t) is F(s) , i.e. if L{f(t)} = F(s), then

f(t) is called an inverse Laplace transform of F(s) and we write symbolically f(t) =
1 1
L {F(s)}, where L is called the inverse Laplace transformation operator. For
2t 1 2t
example, since L{e } = 1/(s+2), we can write L {1/(s+2)} = e .

Since the Laplace transform of a null function N(t) is zero, it is clear that if
L{f (t)} = F(s), then also L{f(t)+N(t)} = F(s). From this it follows that we can have

two different functions with the same Laplace transform. If we disallow null
functions (which do not in general arise in cases of physical interest). This result is
indicated in

Theorem 5.18: Lerch's theorem:


If we restrict ourselves to functions f(t) which are piecewise continuous in every
finite interval 0 t N and of exponential order for t > N, then the inverse Laplace
1
transform of F(s) [i.e. L {F(s)} = f(t)] , is unique .
We shall always assume such uniqueness unless otherwise stated.
Short Table Of Special Inverse Laplace Transforms
o
N F(s) f(t) = L
1
F(s) Conditions
1 1/s 1 s>0
2
2 1/s t s>0
3 1/sn tn1/ (n1)! s > 0, n = 1,2,
4 1/(sa) eat s>a
2 2
5 1/(s + a ) (sin at)/a s > 0, a 0
2 2
6 s/(s + a ) cos at s>0
2 2
7 1/(s a ) (sinh at)/a s > |a|
2 2
8 s/(s a ) cosh at s > |a|
p p1
9 1/s t /(p) s>0,p>0

132
Advanced Engineering Mathematics Chapter 5 - Laplace transforms

Some Important Properties Of Inverse Laplace Transforms:


In the following list we have indicated various important properties of inverse
Laplace transforms. Note the analogy of properties 1-8 with the corresponding
properties on Laplace transforms.
1. Linearity Property:
1 1 1
L {a F(s) + b G(s)} = a L {F(s)} + b L {G(s)}.

Proof:
Since L{a f (t) + b g (t)} = a L{f(t)} + b L{g(t)} = a F(s) + b G(s).
1 1 1
Then L {a F(s) + b G(s)} = a f (t) + b g (t) = a L {F(s)} + b L {G(s)}.

2. First translation or shifting property:


1 1 at
If L {F(s)} = f (t), then L {F(s a)} = e f (t).

3. Second translation or shifting property:


1 1 as
If L {F(s)} = f(t) , then L {e F(s)}= f (t a)U(t a).

4. Change of scale property:


1 1
If L {F(s)} = f (t), then L {F(as)} = f (t/a) / a.

5. Inverse Laplace transform of derivatives:


1 1 (n) n n
If L {F(s)} = f (t), then L {F (s)} = (1) t f (t).

6. Inverse Laplace transform of integrals:



1 1
If L {F(s)} = f (t), then L { F(u)du } = f (t) / t.
s
7. Multiplication by sn:
1 1
If L {F(s)} = f (t) and f (0) = 0, then L {s F(s)} =f '(t).

Thus the multiplication by s has the effect of differentiating f (t).


1
* If f(0) 0, then L { s F(s) f(0)} = f '(t) .

8. Division by s:
F(s) t
1 1
If L {F(s)} = f (t), then L { }= f (u)du .
s 0

133
Advanced Engineering Mathematics Chapter 5 - Laplace transforms

9. The convolution property:


1 1
If L {F(s)} = f (t) and L {G(s)} = g (t), then
t
1
L {F(s) G(s)} = f (u)g(t u)du = f*g.
0
We call f*g the convolution or faltung of f and g. The proofs are left to the students.

Example 5 .21:
1 6 5s 4 2s 18 3 4s
Find L { + + } = A.
2s 3 s3 s 2 9 9s 2 16
Solution:

1 3 5 4 2s 18 1 4 1
A=L
3 2
s 2 s s3 s 2 9 s 2 9 3(s 2 16 ) 9 (s 2 16 )
9 9
3t/2 2
=3e + 5t + 4(t /2!) 2cos 3t + 18(sin 3t)/3 + (1/3)(3 sinh 4t/3)/4 + 4(cosh 4t/3)/9 =
3t/2 2
=3e + 5t + 2t 2cos 3t + 6 sin 3t + (1/4) sinh (4t/3) + (4/9) cosh (4t/3) =

Example 5.22:
Find each of the following:
3s 2 s3 3s 7
a) L1 , b) L1 , c) L1 ,
2 2 2
s 4s 20 s 8s 16 s 2s 3
1 s 1
d) L1 , e) L1 ,
f) L1
1
.
2s 3 s2 s 1 5
(s 4) 2

Solution:
3s 2 1
3s 2 1 3(s 2) 4
a) L1 = L = L =
2 2 2
s 4s 20 (s 2) 16 (s 2) 16
s2 1 4
= 3 L1 + L =
2 2
(s 2) 16 (s 2) 16
2t 2t 2t
= 3 e cos 4t + e sin 4t = e [3 cos 4t + sin 4t].
s3 s 3 (s 4) 1
b) L1 = L1 = L1 =
2
s 2 8s 16 2
(s 4) (s 4)

134
Advanced Engineering Mathematics Chapter 5 - Laplace transforms

1 1
1 4t
= L1
4t 4t
L = e t e = e (1 t).
s 4 2
(s 4)
3s 7 3s 7 3(s 1) 10
c) L1 = L1 = L1 =
s 2 2s 3 2
(s 1) 4
2
(s 1) 4
s 1 1
2
= 3 L1 + 5 L =
2 2
(s 1) 4 (s 1) 4
t t t 3t t
= 3e cosh 2t + 5 e sinh2t = e [3cosh 2t + 5 sinh2t] = 4 e e .
1
d) L

1
2s 3
=

1 1
2
3
L (s )
2
12

=
2
e
(

1 3t / 2 t 1/ 2
1 )
=
1 1 3t / 2
2 t
e .
2
(s 1 ) 1
1 s 1 1 s 1 1 2 2 =
e) L =L =L
s2 s 1 1 2 3 1 )2 3
(s 2 ) 4
(s
2
4
s 1 3
1 2 1 1 2
=L + L =
(s 1 )2 3 3 (s 1 )2 3
4
2 2 4

t/2 3 1 t/2 3 1 t/2 3 3


= e cos t+ e sin t= e [ 3 cos t + sin t ].
2 3 2 3 2 2

1 1 4t 1 1 4t 3/2 4 3/ 2 4t
f) L 5
= e L 5 = e t / (5/2) = t e .
(s 4)
2 s
2 3

Example 5.23:
Find each of the following:
s 1 1
a) L1{ }, b) L1{n(1 )} , c) L1{ },
(s 2 a 2 ) 2 s2 2 2
s (s 1)
1 3s 71 2s 2 4
d) L1{ }, e) L1{ }, f) L {1
}
3 2 2
s (s 1) s 2s 3 (s 1)(s 2)(s 3)
Solution:
d 1 2s s 1 d 1
a) , i.e.
ds s 2 a 2 (s 2 a 2 )2 (s 2 a 2 )2 2 ds s 2 a 2
1 sin at
Then since L1 , we have by property (5)
s2 a 2 a

135
Advanced Engineering Mathematics Chapter 5 - Laplace transforms

s 1 d 1 t sin at
L1 = L1 .
2 2 2 2 ds 2 2 2a
(s a ) s a
1 2 1 s
b) Let F(s) = n 1 = L{f (t)}. Then F'(s) = = 2 .
2 2 s 2
s s(s 1) s 1
1 2(1 cos t)
Since L {F'(s)} = 2(1 cos t) = t f (t), then f (t) = .
t
1
c) Since L1 sin t , we have by repeated application of property 8,
2
s 1
1 t
L1 sin udu = 1 cos t,
2
s(s 1) 0
1 t
L1 (1 cos u)du = t sin t.
2 2
s (s 1) 0
1
1 t 1
d) L (u sin u)du = t 2 + cos t 1.
3 2 2
s (s 1) 0
3s 7 4 1
e) By using the method of partial fractions we find that .
2
s 2s 3 s 3 s 1
3s 7 1 1
Then L1 4 L1 L1
3t t
=4e e .
s 2 2s 3 s 3 s 1
f) By partial fractions method we get
2s 2 4 1 4 7
.
(s 1)(s 2)(s 3) 6(s 1) 3(s 2) 2(s 3)

1 2s 2 4
Thus L 1 t 4 2t 7 3t
= 6 e e e .
(s 1)(s 2)(s 3) 3 2

Example 5.24:
Find each of the following:
2
1 5s 15s 11
3s 1
a) L , b) L1 ,
3 2

(s 1)(s 2) (s 1)(s 1)
s 2 2s 3 2s3 10s 2 8s 40
c) L1 , d) L1 ,
2 2 2 2
(s 2s 2)(s 2s 5) s (s 9)
s s 2
e) L1 , f) L1 es .
2 2
(s 1)(s 1) s 4

136
Advanced Engineering Mathematics Chapter 5 - Laplace transforms

Solution:
5s 2 15s 11 1 7 4 1
a) It is clear that .
(s 1)(s 2)3 3(s 1) (s 2)3 (s 2)2 3(s 2)
2
1 5s 15s 11
1 7 1
Thus L = e t t 2 e2t 4te2t e2t .
3 3 2 3
(s 1)(s 2)
3s 1 2 2s 1
b) Since , therefore
2
(s 1)(s 1) s 1 2
s 1
3s 1 1 s 1
L1 = 2L1 2 L1 L1 =
(s 1)(s 2
1) s 1 s 2
1 s 2
1
t
= 2 e 2 cos t + sin t.
s 2 2s 3 1 2
c) It is clear that = =
(s 2 2s 2)(s 2 2s 5) 3(s 2 2s 2) 3(s 2 2s 5)
1 2
= .
3[(s 1)2 1] 3[(s 1) 2 4]
Then

1 s 2 2s 3 1 t 2 1 1 t
L = e sin t . e t sin 2t = e [sin t + sin 2t].
2 2
(s 2s 2)(s 2s 5) 3 3 2 3
1 1 1 1
d) Since , we have
2 2 9 2 2
s (s 9) s s 9
2s3 10s 2 8s 40 1 8 40 10s 50
(2s 10 ) (2s 10 ) =
s 2 (s 2 9) 9 s s2 s2 9
1 8 40 10s 50
= .
9 s s 2 s 2 9 s 2 9
and so
3 2
1 2s 10s 8s 40 1 50
L = 8 40t 10cos3t sin 3t =
s 2 (s 2 9) 9 3
1
= [24 + 120 t + 30 cos 3t + 50 sin 3t].
27
We can also use the method of partial fractions.
1 t 1 s
e) We can apply property 8. Since L1 e and L 2 cos t ,
s 1 s 1

137
Advanced Engineering Mathematics Chapter 5 - Laplace transforms

1 1 s t (t u) t
t u
then L 2 e cos udu = e e cosdu =
s 1 s 1 0 0
1 t
= [ cos t + sin t e ].
2
s2 s 2
f) It is clear that F(s) = es = e s e s
s2 4 s2 4 s2 4
s 2
Now L1 = cos 2t, L1 = sin 2t.
2 2
s 4 s 4
Application of property 3 yields
s 2
L1 es . = cos 2(t 1) U(t 1), L1 es . = sin 2(t 1) U(t 1),
s2 4 s2 4
Then property 1 (linearity) gives: L1 F(s) = [cos 2(t 1) + sin 2(t 1)] U(t 1).
5.6 Solution of differential equations:
The method of Laplace transforms is particularly useful for solving linear
differential equations with constant coefficients and associated initial conditions. To
accomplish this we take the Laplace transform of the given differential equation (or
equations in the case of a system), making use of the initial conditions. This leads to
an algebraic equations (or system of algebraic equations) in the Laplace transform
and then taking the inverse, the required solution is obtained.

Example 5.25:
Solve y"(t) + y(t) = t, given y'(0) = 2, y(0) = 1.
Solution:
Take the Laplace transform of both sides of the given differential equation and let
2 2
Y = Y(s) = L{y(t)}. Then L{y"(t) + y(t)} = L{t} or s Y s y(0) y'(0) + Y=1/s .

Since y(0) = 1, y'( 0 ) = 2 this becomes


2 1 1 s2
s Ys+2+Y= ,Y= .
s2 s 2 (s 2 1) s 2 1
1 1 s 2 1 s 3
Then Y = = ,
2 2 2 2 2 2 2
s s 1 s 1 s 1 s s 1 s 1
1
and y = L {Y} = t + cos t 3 sin t.

138
Advanced Engineering Mathematics Chapter 5 - Laplace transforms

Example 5.26:
2t
Solve y" 3y' + 2y = 4 e , y(0) = 3, y'(0) = 5.
Solution:
2t
We have L{y"} 3 L{y'} + 2L{y} = 4 L{e }
2 4
[s Y s y (0) y'(0)] 3[ s Y y(0)] + 2Y =
s2
2 4
[s Y + 3s 5] 3[s Y + 3] + 2Y =
s2
2 4
(s 3s + 2)Y + 3s 14 = .
s2
4 14 3s 3s 2 20s 24 7 4 4
Y= = =
(s 2 3s 2)(s 2) s 2 3s 2 (s 1)(s 2)2 s 1 s 2 (s 2)2
1 t 2t 2t
Thus y (t) = L {Y} = 7 e + 4 e + 4t e .

Example 5.27:
Solve y" + y = cos t given y(0) = 0, y'(0) = 0.
Solution:
2 s
L{y"} + L{y} = L{cos t} i.e. s Y+Y=
s2 1
s s 1
Hence Y = =
(s 2 1)2 s 2 1 s 2 1
t
1
Then y (t) = L {Y} = cos u sin(t u)du =
0
1 1 1 t 1
= sin t( u sin u cos u) cos t( sin 2 u) = t sin t.
2 2 2 0 2

Example 5.28:
Solve y' + 4y + 4z = 0, z' + 2y + 6z = 0 given y(0) = 3, z(0) =15,
Solution:
[s Y y(0)] + 4Y +4Z = 0 , [s Z z(0)] + 2Y + 6Z = 0
(s + 4)Y + 4 Z = 3, (s + 6) Z + 2Y = 15

139
Advanced Engineering Mathematics Chapter 5 - Laplace transforms

3(s 6) 15(4) 3s 42 8 11
Then Y = ,
s 2 10s 16 (s 2)(s 8) s 2 s 8
3(2) 15(s 4) 15s 54 4 11
Z= .
2
s 10s 16 (s 2)(s 8) s 2 s 8
1 2t 8t 1 2t 8t
Hence y (t) = L {Y} = 8 e + 11 e , z(t) = L {Z} = 4 e + 11 e .

Example 5.29:
2 t
Find the general solution of y'" 3y" + 3y' y = t e .
Solution:
In this case, we assume y(0) = A , y'(0) = B, y"(0) = C, and we have:
3 2 2 2
[s Y s y(0) s y'(0) y"(0)] 3[s Y s y(0) y'(0)] +3[sY y(0)] Y = ,
(s 1)3
3 2 2 2
i.e. (s Y As B s C) 3(s Y As B) + 3(sY A) Y = ,
3
(s 1)
As 2 (B 3A)s 3A 3B C 2
Y= + .
(s 1)3 (s 1)6
Since A, B and C are arbitrary, so also is the polynomial in the numerator of the first
2
term on the right. We can thus write Y = , and
(s 1)3 (s 1)2 s 1 (s 1)6
invert to find the required general solution
1 1
1 2 t t t 5 t 2 t t t 5 t
y = t e + t e + e + 60 t e = t e + t e + e + 60 t e ,
2
where , , , are arbitrary constants.
It should be noted that finding the general solution is easier than finding the
particular solution since we avoid the necessity of determining the constants in the
partial fraction expansion.

140
Advanced Engineering Mathematics Chapter 5 - Laplace transforms

EXERCISES 5
1. Find the Laplace transforms of each of the following functions. In each case
specify the values of s for which the Laplace transform exists.
4t 2t
a) 2 e , b) 3 e , c) 5t 3 ,
2 t
d) 2t e , e) 3 cos 5t, f) 10 sin 6t,
2 2
g) 6 sin2t 5 cos2t, h) (t + 1) , i) sin t cos t,
j) 3 cosh 5t 4 sinh 5t.

2
[Answers: a) 2/(s 4), s > 4, b) 3/(s+2) , s > 2, c) (5 3s)/s , s > 2,
3 3 2 2
d) (4 + 4s s )/s (s+1), e) 3s/(s + 25), f) 60/(s + 36),
2 4 2 3
g) (12 5s)/(s + 4), h) (s + 4s + 24)/s ,
2 2 2
i) (s 2s + 4)/s(s + 4), s > 0, j) (3s 20)/(s 25), s > 5. ]
2. Find the Laplace transforms of each of the following functions:
4 3 3t 3 3t t
a) 4t 2t + 4e 2sin 5t + 3cos 2t, b) t e , c) e cos2t,
3t 2 t 2t
d) 2e sin4t, e) (t + 2) e , f) e (3 sin 4t 4 cos4t),
4t t
g) e cosh 2t, h) e (3 sinh2t 5cosh2t).
5 4 2 2
[Answers: a) 72/s 12/s 4/(s + 3) + 3s/(s + 4) 10/(s + 25),
4 2 2 2 3
b) 6/(s+3) , c) (s+1)/(s + 2s + 5), d) 8/(s 6s + 25), e) (4s 4s + 2)/(s 1) ,
2 2 2
f) (20 4s)/(s 4s + 20), g) (s + 4)/(s + 8s + 12), h) (1 5s)/(s + 2s 3).]
t t 3
3. Find a) L{e sin 2t}, b) L{(1 + t e ) }.
2 2 3 4
[Answers: a) 2/[(s + 1)(s + 2s + 5)], b) 1/s + 3/(s + 1) + 6/(s + 6) +6/(s + 3) .]
4. Show that:
s 3 (t 1)2 t > 1,
a) L{f (t)} = 2e /s if f (t) =
0 0 < t < 1.
2 2 2 2
b) L{f (2t)} = (s 2s + 4)/4(s + l) (s 2) , if L{f (t)} = (s s + 1)/(2s + 1) (s 1),
t 3/(s+1) 1/s
c) L{e f (3t)} = e /(s+1), if L{f (t)} = e /s,

2 s s 2, 2t 0 t 1,
d) L{f (t)} = 2/s e /s e /s if f (t) =
t t > 1.

5. Verify that
141
Advanced Engineering Mathematics Chapter 5 - Laplace transforms

s t 2 0 < t 1,
a) L{f "(t)} = 2(1 e )/s if f(t) =
0 t > 1.
t 2
b) L (u u e u )du = L{t t + e }/s.
2 t

0
t 1 e u 1 1
c) L du n(1 ) .
0 u s s

s2 a 2 2as
d) L{t cos at} = , e) L{t sin at} =
(s 2 a 2 )2 (s 2 a 2 )2
2 2 2
f) L{t(3 sin2t 2cos2t)}= (8 + 12s 2s )/(s + 4) ,
2 2 2 3 2 2
g) L{t sin t} = (6s 2)/(s + 1) , h) L{t sinh 2t} = 4s/(s 4) ,
2 2 2 2 3 2 3
i) L{t cosh3t} = (s + 9)/(s 9) , j) L{t cos t} =(2s 6s)/(s + 1) ,
2 4 3 2 2 3
k) L{(t 3t + 2) sin3t} = (6 s 18s +126 s 162 s + 432)/(s + 9) .
at bt
6. Show that: a) L{e e )/t} = n[(s + a)/(s + b)],
2 2 2 2
b) L{(cos at cos bt)/t}= n [(s + a )/(s + b )],

c) L{(sinh t)/ t] = n[(s 1)/(s + 1)],


sin 2 t 1
d) dt = .
2 2
0 t
7. Find L{f (t)} if
3t 0<t<2
a) f (t) = , where f (t) has period = 4.
6 2 t < 4
2
b) f (t) = t , 0 < t < 2 and f (t + 2) = f (t).
t 0<t<1
c) f (t) = , where f (t + 2) = f (t) for t > 0.
0 1 t < 2
2s 4s 2 4s
[Answers: a) (3 3e 6s e )/s (1 e ),
2s 2s 2 2s 3 2s s 2 2s
b) (2 2e 4se 4s e )/s (l e ), c) [l e (s + l)]/s (1 e ) .]
t sin u tan 1 (s 2)
8. Show that a) L e2t du = ,
0 u s 2

142
Advanced Engineering Mathematics Chapter 5 - Laplace transforms

tan 1 s
t sin u 1
b) L t
u du = 2 2 .

0 s s(s 1)
g(t) 0<t<a
9. Show that f (t) =
h(t) t>a

can be written as f(t) = g(t) + [h(t) g(t)] U(t a).


e t 0 < t < 3 3(s + l)
10. Find L{f (t)} if f (t) = [Answer: L{f (t)} = (1 e /(s+1)]
0 t>3
11. Express in terms of Heaviside's unit step functions
sin t 0<t<
t 2 0<t<2
a) f (t) = b) f (t) = sin 2t < t < 2
4t t > 2 t > 2
sin 3t
2 2
[Answers: a) f (t) = t + (4t t ) U (t 2),
b) f (t) = sin t + (sin 2t sin t) U (t ) + (sin 3t sin 2t) U(t 2).]

t 2
12. Show that a) e U(t 2)dt = e ,

2 3 2s 2 3
b) L{t U(t 2)} = (2/s ) 2e (1 + 2s + 2s )/s , s>0

13. Find L{f(t)} if


sin t 0<t< cos t 0<t< 3
c) f(t) = sin t.
a) f (t) = , b) f (t) = ,
0 t> sin t t>
s 2 s 2 2 2
[Answers: a) (l + e )/(s + 1), b) [s + (s 1) e ]/(s + 1), c) 6/(s + 1)(s + 9).]
14. Find the inverse Laplace transform of the following functions:
2 2
a) 3/(s + 4), b) 1/(2s 5), c) 8s/(s + 16), d) 6/(s + 4),
2 2 5 7/2
e) (3s 12)/(s + 8), f) (2s 5)/(s 9), g) 1/s , h) 1/s ,
3/4 2 2
i) 12/(4 3s), j) (s+1)/s , k) ( s 1) /s , ) (2s + 1)/s(s + 1).
4t 5t/2
[Answers: a) 3e , b) e , c) 8 cos 4t,
d) 3 sin 2t, e) 3 cos2 2 t 3 2 sin 2 2 t, f) 2 cosh 3t (5 sinh 3t)/2,
4 5/2 4t/3
g) t /24, h) 8 t /15, i) 4e ,
2/3 1/3 t
j) (t + 3t )/(1/3), k) 1+t4t / , ) 1 + e .]
15. Find the inverse Laplace transforms of the following functions:
2 2 2 2
a) (3s 8)/(4s + 25), b) (5s + 10)/(9s 16), c) (3s8)/(s + 4)(4s 24)/(s 16),

143
Advanced Engineering Mathematics Chapter 5 - Laplace transforms

5/2 5/2
d) (3s 2)/s 7/(3s + 3) , e) s/(s + 1) ,
2 2 5 2 5/2
f) 3(s 1) /2s + (4s 18)/(9 s ) + (s+1)(2 s )/s ,
5 2 1/3
g) s/(s + 1) , h) (3s 14)/(s 4s + 8), i) 1/(8s 27)
2 2 2
j) (8s + 20)/(s 12s + 32), k) (3s + 2)/(4s + 12s + 9), ) (5s 2)/(3s + 4s + 8).
[Answers: a) (3/4) cos (5t/2) (4/5) sin (5t/2) ,
b) (5/9) cosh (4t/3) + (5/6) sin (4t/3), c) 3cos 2t 4sin2t 4cosh 4t + 6 sinh 4t,
3/2 2t/3
d) 6t / 8 t /3 7e /3 , e) 2t (3 2t)/3 ,
2 4 3/2
f) t 3t /2 + t /16 + 4 t / + 8t /3 4cosh 3t + 6 sinh 3t,
t 3 4 2t 2/3 27t/8
g) e (4t t )/24, h) e (3 cos 2t 4 sin 2t), i) t e / [2 (1/3)],
6t 8t 4t
j) 2e (4 cosh 2t + 7 sinh 2t) = 11 e 3 e ,
3t/2 3t/2
k) (3/4) e (5/8) t e ,
2t/3
) e [25 cos (2 5 t/3) 24 5 sin (2 5 t/3)]/15.]
16. Find the inverse Laplace transforms of the following functions:
2s 2 3s 2 s 3s 2 2s 2
a) e /s , b) 8e /(s + 4), c) e / (s + 1) , d) e /(s 2s + 5) , e) se /(s 3s + 2),
2 2 3 2
f) (s + 1)/(s + 2s+2) , g) n[(s + 2)/(s + 1)], h) 1/s (s + 1), i) (s + 2)/s (s + 3) ,
3 5 5
j) 1/s(s+1) , k) 1/(s 1) (s + 2), ) s/(s 2) (s+1).

[Answers: a) (t 2)U(t 2), b) 4U(t 3) sin 2(t 3) , c) (t 1) U(t 2)/ ,
(t 3) 2(t 2) (t 2) t
d) U(t 3) e sin 2(t 3), e) [2e e ] U(t 2), f) t e sin t,
t 2t 2 t 3t
g) (e e )/t, h) l t + t e , i) 2t/3 + (l e )/9,
t 2 et 4 3 2 2t
j) 1 e (1 + t + t ), k) [ t (4t /3) + (4t /3) (8t/9) + (8/27) e /243,
72
2t 4 3 2 t
) e (t /36) + (t /54) + (t /54) + (t/81) (1/243) + e / 243.]
17. Find the inverse Laplace transforms of the following functions:
1 2 2
a) tan (2/s ), b) 1/(s + 3)(s 1), c) 1/(s + 2) (s 2),
2 2 2 2 2 3
d) 1/(s + 1)(s + 1), e) s /(s + 4) , f) 1/(s + 1) ,
2 3 2 3
g) s/(s + 4) , h) (3s + 16)/(s s 6), i) (2s 1)/(s s),
2
j) (s + 1)/(6s + 7s + 2).
t 3t 2t 2t 2t
[Answers: a) (2 sin t sinh t)/t, b) (e e ), c) t e + (e e )/16,
t 2
d) (sin t cos t + e ), e) t cos 2t + sin 2t , f) [(3 t ) sin t 3t cos t]/8 ,
2 3t 2t t t/2 2t/3
g) (t sin 2t)/64 (t cos 2t)/32, h) 5e 2e , i) 1 3e , j) e e /3.]

144
Advanced Engineering Mathematics Chapter 5 - Laplace transforms

18. Use partial fractions to show that:


2
a) L1 11s 2s 5 = 5 e 3 e /2 + 2 e ,
2t t/2 t

(s 2)(2s 1)(s 1)

b) L1 27 12s = 3 e 3 cos 3t,
4t
2
(s 4)(s 9)
3
c) L1 s 16s 24 = sin 4t + cos 2t sin2t,
s4 20s2 64
2
L1 s 2s 3 = (2t 1) e + 3 e /2,
t t
d)
2
(s 1) (s 1)
3 2
e) L1 3s 3s 40s 36 = (5t + 3) e 2t e ,
2t 2t

(s2 4)2
s2 3
f) L1
3t 2t t t
= 3e /50 e /25 (e cos 2t)/50+ (9e sin 2t)/25.
(s 2)(s 3)(s2 2s 5)

g) L1 s
= sin t sinh t,
2 2
(s 2s 2)(s 2s 2)
3 2
h) L1 2s s 1 = sin t + t cos t t e .
t
2 2 2
(s 1) (s 1)
2
i) L1 2s 6s 5 = e e + 5 e /2,
t 2t 3t
s3 6s2 11s 6
s 5
j) L1
t
= 2 e + 3 sin t 2 cos t.
2
(s 1)(s 1)
t
19. Solve y" + 2y' + 5y = e sin t, y(0) = 0, y'(0) = 1.
t 2 2 2
[Answer: y= e (sin t + sin 2t), Y = (s + 2s + 3)/(s + 2s + 2)(s + 2s + 5).]
20. Solve y" + 9y = cos 2t, if y(0) =1, y() = 1.
2 2 2
[Answer: Y = (4s)/5(s + 9) + 12/5(s + 9) + s/5(s + 4),
y = (4/5) cos 3t + (4/5) sin3t + (1/5) cos2t.]
2
21. Solve ty" + y' + 4ty = 0, y(0) = 3, y'(0) = 0. [Answer: Y = 3/(s + 4) , y =3Jo(2t).]
22. Solve y" t y' + y = 1, y(0) =1 , y'(0) = 2.
12 s2
2 e
[Answer: Y = (l/s) + (2/s ) + (c ), y = 1 + (c + 2) t, c = 0.]
2
s

145
Advanced Engineering Mathematics Chapter 5 - Laplace transforms

23. Solve each of the following by using L.T.


a) y"(t) + 4y (t) = 9t, y (0) = 0, y'(0) = 7,
t
b) y"(t) 3y'(t) + 2y = 4t + 12 e , y (0) = 6, y'(0) = 1,
2
c) y" 4y' + 5y = 125t , y(0) = y'(0) = 0,
d) y" + y = 8 cos t, y(0) = 1, y'(0) = 1,
t
e) y"'(t) y = e , y(0) = y'(0) = y"(0) = 0,
(4)
f) y + 2y" + y = sin t , y(0) = y' (0) = y"(0) = y'"(0) = 0.
t 2t t
[Answers: a) 3t + 2sin 2t, b) 3e 2e + 2t + 3 + 2e ,
2 2t
c) 25t + 40 t + 22 + 2e (2 sin t 11cos t), d) cos t 4 sin t + 4t cos t,
t t t
e) t e /3 + e ( cos ( 3 t/2) + 5 3 sin ( 3 t/2)/36 e ,
2
f) ((3 t ) sin t 3t cos t)/8.]
t
24. Solve y' + 2z' = t, y" z = e , subject to conditions
y(0) = 3 , y'(0) = 2 , z(0) = 0.
2 t t
[Answer: y = 2 + t + e (3 sin t)/2 + cos t, z = 1e + (3 sin t)/2 cos t.]
25. Solve y' = 2y 3z, z' = z 2y subject to y(0) = 8, z(0) = 3.
t 4t t 4t
[Answer: y = 5e + 3e , z = 5e 2e .]
t
26. Solve y" + z' + 3y = 15 e , z" 4y' + 3z = 15sin 2t subject to y(0) = 35,
y'(0) = 48, z(0) = 27, z'(0) = 55.
t
[Answer: y = 30 cos t 15 sin 3t + 3e + 2 cos 2t,
t
z = 30cos 3t 60 sin t 3e + sin 2t.]
27. Solve y' z' 2y + 2z = sin t, y" + 2z' + y = 0 if y(0) = 0, y'(0) = z(0) = 0.
t 2t t/3
[Answer: y = e /9 + 4e /45 (1/5) cos t (2/5) sin t + t e ,
t 2t t
z = e /9 e /9 + t e /3.]
t
28. Solve y' + 2z" = e , y' + 2y z = 1, if y (0) = z(0) = z'(0) = 0.
t at bt t at bt
[Answer: y = 1 + e e e , z = l + e be ae ,
where a = (2 2 ), b= (2 + 2 )]
t
29. Solve t y" z' = sin t, 3y" + 3z" = te 3cos t, given that y(0) = 1, y'(0) = 2,
z(0) = 4, z"(0) = 0.
2 t
[Answer: y = (2t /3) + (5t/3) (2/3) (e /3),

146
Advanced Engineering Mathematics Chapter 5 - Laplace transforms

2 t t
z = (2t /3) + (8/3) + (e /3) + (t e /3) + cos t.]
30. Solve 2y" y' + 9y z" z' 3z = 0,
2y" + y' + 7y z" + z' 5z = 0, given that y(0) = y'(0) = 1, z(0) = z'(0) = 0.
t t
[Answer: y = (e + 2cos 2t + sin 2t)/3, z = (2e 2cos 2t sin2t)/3.]

147
Advanced Engineering Mathematics Chapter 5 - Laplace transforms

REFERENCES
(1) Advanced Mathematics for Engineers and Scientists By Murray R. Spiegel.
(2) Advanced Calculus, By Murray R. Spiegel.
(3) Laplace Transforms, By Murray R. Spiegel.
(4) Vector Analysis, By Murray R. Spiegel.
(5) Differential Equations, By Jr. Frank Ayres.
(Schaum's outline series in Mathematics-McGraw-Hill book company)
(6) A course in Mathematics Parts I, II, III. By Dr. Naguib G. Bakhoom.
(7) Mathematics for Engineers, By Dr. Abdel-Latif El-Sedeek.
(8) Mathematics for Engineering Students (1984)
By Dr. A. A. Azim, Dr. M. Shams, and Dr. Labib Iskandar
(9) Solved Examples for Engineering Students (1983)
By Dr. A. A. Azim, Dr. M. Shams, and Dr. Labib Iskandar.
(10) Mathematics for Engineering Students Part II
By Dr. Abdel-Lateef El Sedeek and Dr. S. Elnahwy
(Faculty of Engineering Cairo University)
(11) Special functions By Earl D. Rainville.
(12) Special functions of Mathematical Physics and Chemistry By Ian N. Sneddon
(Oliver & Boyd LTD.)
(13) The theory of ordinary differential equations By J. C. Burkill
(Oliver and Boyd LTD.)
(14) Integration of ordinary differential equations By E. L. Ince
(Oliver and Boyd LTD.)
(15) An elementary treatise on differential equations and their applications.
By H.T.H. Piaggio.
(16) Volume and integral By W. W. Rogosinski (0 & B)
(17) Vector Methods By D.E. Rutherford (0 & B)
(18) Multiple Integrals, Field theory and Series By B. M. Budak and S. V. Fomin.
(19) University Mathematics By J. Blakey
(A textbook for students of Science and Engineering)
(20) Laplace Transform theory By M. G. Smith.
(21) Problems in Mathematical Analysis - Under the editorship of B. Demidovich.
(22) Differential and Integral Calculus By N. Piskunov.

148
Advanced Engineering Mathematics Chapter 6 - Special Functions

CHAPTER 6
SPECIAL FUNCTIONS
6.1 The Gamma Function:
nd
The gamma function denoted by (s) is defined by the 2 Eulerian integral

(s) = e x x s 1dx , (s > 0) (6.1)
0

This is improper integral with respect to the upper limit, s < l improper with respect
to the lower limit. This integral convergent for all s > 0.
Integrating by parts we get



x s x s
(s+1) = e x dx = e x + s e x x s 1dx = s (s), (6.2)
0 0 0

this is recurrence formula for the gamma function. The recurrence relation (6.2) is a
difference equation which has (6.1) as a solution.
By repeating the application of the recurrence relation (6.2) we get:
(s+1) = s (s) = s(s l) (s1) = s(s l)(s 2) (s2) =
= s(s l)(s 2) (s k)(sk) , (k < s) (6.3)
It is clear that
(n+1) = n(n l)(n 2) 2.1 (1) , if n is positive integer,



x 0 x x
But (1) = e x dx = e dx = e = 1.
0 0 0

Therefore (n + l) = n! , n = 1, 2, 3, (6.4)
For this reason (n) is sometimes called the factorial function.
2
Putting x = t in the integral (6.1) we get
2
(s) = e x x s 1dx = 2 e t t 2s 1dt , s>0 (6.5)
0 0

149
Advanced Engineering Mathematics Chapter 6 - Special Functions


t2
It is clear that ( 1 ) = 2 e dt .
2
0
2
t
To calculate ( 1 ) , we shall prove that Gauss integral e dt = .
2
0

2 2
Put A = e x dx = e y dy .

2 2
Then A = e x dx . e y dy =
2



(x 2 y 2 ) 2 2
= e dxdy = 4 e(x y ) dxdy
00

Changing to polar coordinates (r, ) where x = r cos , y = r sin , the last integral
becomes

2
r 2 2
r dr d = e r d(r 2 ) = ,
2
A =4 e
0 r 0 0
and so A = . It follows that ( 1 ) = .
2
It is clear that
1.3.5 (2n 1) (2n)!
(n+ 1 ) = (n 1 )(n 3 ) 1 ( 1 ) = = . (6.6)
2 2 2 2 2
2n 2(2n) n!

Table of values and graph of the gamma function


n (n)
1.00 1.0000
1.10 0.9514
1.20 0.9182
1.30 0.8975
1.40 0.8873
1.50 0.8862
1.60 0.8935
1.70 0.9086
1.80 0.9314
1.90 0.9618
2.00 1.0000

150
Advanced Engineering Mathematics Chapter 6 - Special Functions

For any positive p (p+1) = p (p)


For a positive integer n (n+1) = n!

For p = 1/2 ( 1 )
2

(2n)!
For p = n + (n+ 1 ) = (n+ 1 ) =
2
2 2(2n) n!

Values of Gamma Function



(s) e x x s 1dx, for (1 x 2)
0
The value of the (x) for x < 1 (x 0, 1, 2, ... ) and x > 2 can be calculated
by means of the following formulas:
(x+1) = x (x), and (x) = (x1). (x1)
How To Use:
You need to find the (l.075) Find the row with x = 1.07
Find the column 5, the number Inside the cell is 0.961918

See How To Read Numbers In Tables. The result is 0.961918

x 0 1 2 3 4 5 6 7 8 9
1.00 1 0.999424 0.99885 0.998277 0.997707 0.997139 0.996572 0.996008 0.995445 0.994885
1.01 0.994326 0.993769 0.993214 0.992661 0.99211 0.991561 0.991014 0.990469 0.989925 0.989384
1.02 0.988844 0.988306 0.987771 0.987236 0.986704 0.986174 0.985645 0.985119 0.984594 0.984071
1.03 0.98355 0.983031 0.982513 0.981997 0.981484 0.980972 0.980461 0.979953 0.979446 0.978941
1.04 0.978438 0.977937 0.977437 0.97694 0.976444 0.975949 0.975457 0.974966 0.974477 0.97399
1.05 0.973504 0.97302 0.972538 0.972058 0.971579 0.971103 0.970627 0.970154 0.969682 0.969212
1.06 0.968744 0.968277 0.967812 0.967349 0.966887 0.966427 0.965969 0.965512 0.965057 0.964604
1.07 0.964152 0.963702 0.963254 0.962807 0.962362 0.961918 0.961476 0.961036 0.960598 0.960161
1.08 0.959725 0.959292 0.958859 0.958429 0.958 0.957573 0.957147 0.956723 0.9563 0.955879
1.09 0.955459 0.955042 0.954625 0.954211 0.953797 0.953386 0.952976 0.952567 0.95216 0.951755

151
Advanced Engineering Mathematics Chapter 6 - Special Functions

x 0 1 2 3 4 5 6 7 8 9

1.10 0.951351 0.950948 0.950548 0.950148 0.94975 0.949354 0.948959 0.948566 0.948174 0.947784

1.11 0.947396 0.947008 0.946623 0.946238 0.945856 0.945474 0.945095 0.944716 0.944339 0.943964

1.12 0.94359 0.943218 0.942847 0.942477 0.942109 0.941743 0.941378 0.941014 0.940652 0.940291

1.13 0.939931 0.939574 0.939217 0.938862 0.938508 0.938156 0.937805 0.937456 0.937108 0.936761

1.14 0.936416 0.936072 0.93573 0.935389 0.935049 0.934711 0.934374 0.934039 0.933705 0.933372

1.15 0.933041 0.932711 0.932382 0.932055 0.931729 0.931405 0.931082 0.93076 0.93044 0.930121

1.16 0.929803 0.929487 0.929172 0.928858 0.928546 0.928235 0.927925 0.927617 0.92731 0.927004

1.17 0.9267 0.926397 0.926095 0.925794 0.925495 0.925197 0.924901 0.924606 0.924312 0.924019

1.18 0.923728 0.923438 0.923149 0.922862 0.922575 0.92229 0.922007 0.921724 0.921443 0.921164

1.19 0.920885 0.920608 0.920332 0.920057 0.919783 0.919511 0.91924 0.91897 0.918702 0.918435

x 0 1 2 3 4 5 6 7 8 9

1.20 0.918169 0.917904 0.91764 0.917378 0.917117 0.916857 0.916599 0.916341 0.916085 0.91583

1.21 0.915576 0.915324 0.915073 0.914823 0.914574 0.914326 0.91408 0.913834 0.91359 0.913348

1.22 0.913106 0.912865 0.912626 0.912388 0.912151 0.911916 0.911681 0.911448 0.911216 0.910985

1.23 0.910755 0.910526 0.910299 0.910072 0.909847 0.909623 0.909401 0.909179 0.908959 0.908739

1.24 0.908521 0.908304 0.908088 0.907873 0.90766 0.907447 0.907236 0.907026 0.906817 0.906609

1.25 0.906402 0.906197 0.905992 0.905789 0.905587 0.905386 0.905186 0.904987 0.904789 0.904593

1.26 0.904397 0.904203 0.904009 0.903817 0.903626 0.903436 0.903247 0.90306 0.902873 0.902688

1.27 0.902503 0.90232 0.902137 0.901956 0.901776 0.901597 0.901419 0.901242 0.901067 0.900892

1.28 0.900718 0.900546 0.900375 0.900204 0.900035 0.899867 0.8997 0.899533 0.899368 0.899204

1.29 0.899042 0.89888 0.898719 0.898559 0.898401 0.898243 0.898086 0.897931 0.897776 0.897623

152
Advanced Engineering Mathematics Chapter 6 - Special Functions

x 0 1 2 3 4 5 6 7 8 9

1.30 0.897471 0.897319 0.897169 0.89702 0.896872 0.896724 0.896578 0.896433 0.896289 0.896146

1.31 0.896004 0.895863 0.895723 0.895584 0.895446 0.89531 0.895174 0.895039 0.894905 0.894772

1.32 0.89464 0.89451 0.89438 0.894251 0.894123 0.893997 0.893871 0.893746 0.893623 0.8935

1.33 0.893378 0.893257 0.893138 0.893019 0.892901 0.892784 0.892669 0.892554 0.89244 0.892327

1.34 0.892216 0.892105 0.891995 0.891886 0.891778 0.891671 0.891565 0.89146 0.891356 0.891253

1.35 0.891151 0.89105 0.89095 0.890851 0.890753 0.890656 0.89056 0.890464 0.89037 0.890277

1.36 0.890185 0.890093 0.890003 0.889913 0.889825 0.889737 0.88965 0.889565 0.88948 0.889396

1.37 0.889314 0.889232 0.889151 0.889071 0.888992 0.888914 0.888836 0.88876 0.888685 0.888611

1.38 0.888537 0.888465 0.888393 0.888323 0.888253 0.888184 0.888116 0.888049 0.887983 0.887918

1.39 0.887854 0.887791 0.887729 0.887667 0.887607 0.887548 0.887489 0.887431 0.887375 0.887319

x 0 1 2 3 4 5 6 7 8 9

1.40 0.887264 0.88721 0.887157 0.887105 0.887053 0.887003 0.886953 0.886905 0.886857 0.88681

1.41 0.886765 0.88672 0.886676 0.886633 0.88659 0.886549 0.886509 0.886469 0.88643 0.886393

1.42 0.886356 0.88632 0.886285 0.886251 0.886217 0.886185 0.886153 0.886123 0.886093 0.886064

1.43 0.886036 0.886009 0.885983 0.885958 0.885933 0.88591 0.885887 0.885865 0.885844 0.885824

1.44 0.885805 0.885787 0.885769 0.885753 0.885737 0.885722 0.885708 0.885695 0.885683 0.885672

1.45 0.885661 0.885652 0.885643 0.885635 0.885628 0.885622 0.885617 0.885612 0.885609 0.885606

1.46 0.885604 0.885603 0.885603 0.885604 0.885606 0.885608 0.885611 0.885616 0.885621 0.885626

1.47 0.885633 0.885641 0.885649 0.885658 0.885668 0.885679 0.885691 0.885704 0.885717 0.885732

1.48 0.885747 0.885763 0.88578 0.885798 0.885816 0.885836 0.885856 0.885877 0.885899 0.885922

1.49 0.885945 0.88597 0.885995 0.886021 0.886048 0.886076 0.886104 0.886134 0.886164 0.886195

153
Advanced Engineering Mathematics Chapter 6 - Special Functions

x 0 1 2 3 4 5 6 7 8 9

1.50 0.886227 0.88626 0.886293 0.886328 0.886363 0.886399 0.886436 0.886474 0.886512 0.886551

1.51 0.886592 0.886633 0.886675 0.886717 0.886761 0.886805 0.88685 0.886896 0.886943 0.88699

1.52 0.887039 0.887088 0.887138 0.887189 0.887241 0.887293 0.887346 0.8874 0.887455 0.887511

1.53 0.887568 0.887625 0.887683 0.887742 0.887802 0.887863 0.887924 0.887986 0.888049 0.888113

1.54 0.888178 0.888243 0.888309 0.888376 0.888444 0.888513 0.888582 0.888653 0.888724 0.888796

1.55 0.888868 0.888942 0.889016 0.889091 0.889167 0.889244 0.889321 0.8894 0.889479 0.889559

1.56 0.889639 0.889721 0.889803 0.889886 0.88997 0.890055 0.89014 0.890226 0.890313 0.890401

1.57 0.89049 0.890579 0.890669 0.89076 0.890852 0.890945 0.891038 0.891132 0.891227 0.891323

1.58 0.89142 0.891517 0.891615 0.891714 0.891814 0.891914 0.892015 0.892117 0.89222 0.892324

1.59 0.892428 0.892533 0.892639 0.892746 0.892854 0.892962 0.893071 0.893181 0.893292 0.893403

x 0 1 2 3 4 5 6 7 8 9

1.60 0.893515 0.893628 0.893742 0.893857 0.893972 0.894088 0.894205 0.894323 0.894441 0.894561

1.61 0.894681 0.894801 0.894923 0.895045 0.895169 0.895292 0.895417 0.895543 0.895669 0.895796

1.62 0.895924 0.896052 0.896182 0.896312 0.896443 0.896574 0.896707 0.89684 0.896974 0.897109

1.63 0.897244 0.897381 0.897518 0.897655 0.897794 0.897933 0.898074 0.898215 0.898356 0.898499

1.64 0.898642 0.898786 0.898931 0.899076 0.899223 0.89937 0.899518 0.899666 0.899816 0.899966

1.65 0.900117 0.900269 0.900421 0.900574 0.900728 0.900883 0.901039 0.901195 0.901352 0.90151

1.66 0.901668 0.901828 0.901988 0.902149 0.90231 0.902473 0.902636 0.9028 0.902965 0.90313

1.67 0.903296 0.903464 0.903631 0.9038 0.903969 0.904139 0.90431 0.904482 0.904654 0.904827

1.68 0.905001 0.905176 0.905351 0.905527 0.905704 0.905882 0.90606 0.90624 0.90642 0.9066

1.69 0.906782 0.906964 0.907147 0.907331 0.907515 0.907701 0.907887 0.908074 0.908261 0.90845

154
Advanced Engineering Mathematics Chapter 6 - Special Functions

x 0 1 2 3 4 5 6 7 8 9

1.70 0.908639 0.908829 0.909019 0.909211 0.909403 0.909596 0.909789 0.909984 0.910179 0.910375

1.71 0.910572 0.910769 0.910967 0.911166 0.911366 0.911567 0.911768 0.91197 0.912173 0.912376

1.72 0.912581 0.912786 0.912991 0.913198 0.913405 0.913613 0.913822 0.914032 0.914242 0.914453

1.73 0.914665 0.914878 0.915091 0.915306 0.915521 0.915736 0.915953 0.91617 0.916388 0.916607

1.74 0.916826 0.917046 0.917267 0.917489 0.917712 0.917935 0.918159 0.918384 0.918609 0.918835

1.75 0.919063 0.91929 0.919519 0.919748 0.919978 0.920209 0.920441 0.920673 0.920906 0.92114

1.76 0.921375 0.92161 0.921846 0.922083 0.922321 0.92256 0.922799 0.923039 0.923279 0.923521

1.77 0.923763 0.924006 0.92425 0.924494 0.92474 0.924986 0.925233 0.92548 0.925728 0.925977

1.78 0.926227 0.926478 0.926729 0.926981 0.927234 0.927488 0.927742 0.927997 0.928253 0.92851

1.79 0.928767 0.929026 0.929285 0.929544 0.929805 0.930066 0.930328 0.930591 0.930854 0.931119

x 0 1 2 3 4 5 6 7 8 9

1.80 0.931384 0.93165 0.931916 0.932184 0.932452 0.93272 0.93299 0.933261 0.933532 0.933804

1.81 0.934076 0.93435 0.934624 0.934899 0.935175 0.935451 0.935728 0.936006 0.936285 0.936565

1.82 0.936845 0.937126 0.937408 0.937691 0.937974 0.938258 0.938543 0.938829 0.939115 0.939402

1.83 0.93969 0.939979 0.940269 0.940559 0.94085 0.941142 0.941434 0.941728 0.942022 0.942317

1.84 0.942612 0.942909 0.943206 0.943504 0.943803 0.944102 0.944402 0.944703 0.945005 0.945308

1.85 0.945611 0.945915 0.94622 0.946526 0.946832 0.947139 0.947447 0.947756 0.948066 0.948376

1.86 0.948687 0.948999 0.949311 0.949625 0.949939 0.950254 0.95057 0.950886 0.951203 0.951521

1.87 0.95184 0.95216 0.95248 0.952801 0.953123 0.953446 0.953769 0.954093 0.954419 0.954744

1.88 0.955071 0.955398 0.955726 0.956055 0.956385 0.956715 0.957047 0.957379 0.957711 0.958045

1.89 0.958379 0.958714 0.95905 0.959387 0.959725 0.960063 0.960402 0.960742 0.961082 0.961424

155
Advanced Engineering Mathematics Chapter 6 - Special Functions

x 0 1 2 3 4 5 6 7 8 9

1.90 0.961766 0.962109 0.962453 0.962797 0.963142 0.963488 0.963835 0.964183 0.964531 0.964881

1.91 0.965231 0.965582 0.965933 0.966286 0.966639 0.966993 0.967347 0.967703 0.968059 0.968416

1.92 0.968774 0.969133 0.969492 0.969853 0.970214 0.970576 0.970938 0.971302 0.971666 0.972031

1.93 0.972397 0.972764 0.973131 0.973499 0.973868 0.974238 0.974609 0.97498 0.975352 0.975725

1.94 0.976099 0.976473 0.976849 0.977225 0.977602 0.97798 0.978358 0.978738 0.979118 0.979499

1.95 0.979881 0.980263 0.980647 0.981031 0.981416 0.981802 0.982188 0.982576 0.982964 0.983353

1.96 0.983743 0.984133 0.984525 0.984917 0.98531 0.985704 0.986098 0.986494 0.98689 0.987287

1.97 0.987685 0.988084 0.988483 0.988883 0.989285 0.989687 0.990089 0.990493 0.990897 0.991302

1.98 0.991708 0.992115 0.992523 0.992931 0.993341 0.993751 0.994162 0.994573 0.994986 0.995399

1.99 0.995813 0.996228 0.996644 0.997061 0.997478 0.997896 0.998315 0.998735 0.999156 0.999578

2.00 1 1.00042 1.00085 1.00127 1.0017 1.00212 1.00255 1.00298 1.00341 1.00384

156
Advanced Engineering Mathematics Chapter 6 - Special Functions

This is the graph of the absolute value of the Gamma function:

This is the graph of the real pat of the Gamma function

157
Advanced Engineering Mathematics Chapter 6 - Special Functions

This is the graph of the imaginary part of Gamma function:

Gamma Function Calculator


Since (s+ 1) = s (s) ,then
(1.5) = (0.5) (1/2) = 1 1 = 0.886226925453
2 2
(2.5) = (1.5)*(0.5) (1/2) = 3 3 = 1.329340388179
4 4
(3.5) = (2.5)*(1.5)*(0.5) (1/2) = 5 3 1 15 = 3.323350970448
222 8
(1.461) = 0.885603
(1.496) = 0.886104
(5.461) = 4.461*3.461*2.461*1.461*(1.461) =
= 4.461*3.461*2.461*1.461*0.885603 = 49.162587369672505923
(0.99713) = 1.00166478
(1.496) = 0.8861042
Since (s ) = (s + 1)/s ,then
(-1.496) = (-0.496)/( -1.496) = (0.504)/[( -1.496)* ( -0.496)] =

158
Advanced Engineering Mathematics Chapter 6 - Special Functions

= (1.504)/[( -1.496)* ( -0.496) )* 0.504]= (0.886363)/1.496*0.496* 0.504


= (0.886363)/ 0.373976064 = 2.3701062322534096727645114741889

(-5.461) = (-4.461) /[-5.461] = (-3.461) /[-5.461*-4.461] =


= (-2.461) /[-5.461*-4.461*-3.461] = (-1.461) /[-5.461*-4.461*-3.461*2.461] =
= (-0.461) /[-5.461*-4.461*-3.461*-2.461*-1.461]
= (0.539) /[-5.461*-4.461*-3.461*-2.461*-1.461*-0.461]=
= (1.539) /[-5.461*-4.461*-3.461*-2.461*-1.461*-0.461*0.539] =
= 0.888113/ 75.328187751537172939179 = 0.0117899159
= 0.0117899161514461426821

(-5.461) = 0.0117899161514461426821
(-1.496) = 2.37010599
(-5.461) = 0.011789916
(5 /2) = 8 /15
(3 /2) = 4 /3
(1 /2) = 2
(1 /2) =
(3 /2) = 1 /2
(5 /2) = 3 /4
(5.25) = 35.2116118528
(-5.461) = 0.011789916151446
(-1.496) = 2.3701059895317
6.2: The Beta function:
The beta function, denoted by B (p, q) is defined by the first Eulerian integral
1
B (p, q) = x p 1 (1 x)q 1 dx , (6.7)
0

159
Advanced Engineering Mathematics Chapter 6 - Special Functions

this is convergent for p > 0, q > 0. In case p < 1 this integral is improper with respect
to the lower limit, in case of q < 1 it is improper with respect to the upper limit of
integration.
2 2
If we put x = cos , we get 1 x = sin , dx = 2 cos sin d and

2
B(p, q) = 2 cos 2p 1 sin 2q 1 d , (p > 0, q > 0) (6.8)
0
The beta function is connected with the gamma according to the relation
(p)(q)
B(p,q) , (6.9)
(p q)
To prove this relation, we use equation (6.5)

2
(p) = 2 e x x 2p 1dx , p>0,
0

x 2 2q 1 2
(q) = 2 e x dx = 2 e y y 2q 1dy , q>0
0 0
2 2
Then (p) (q) = 4 x 2p 1y 2q 1e(x y ) dxdy
00

Transforming to polar coordinates, x = r cos , y = r sin , it follows that the


Jacobean of the transformation J = r and

2 2 2 2
(p) (q) = 4 e(x y ) x 2p 1y 2q 1dxdy = 4 e(x y ) x 2p 1y 2q 1 =
00 D
2
= 4 e r r 2p 2q 1 cos 2p 1 sin 2q 1 dr d =

160
Advanced Engineering Mathematics Chapter 6 - Special Functions


r 2 2p 2q 1 2 2p 1 2q 1

= 2 e r
dr 2 cos sin d =

0 0
= (p + q) B (p, q).
It is clear that the beta function is symmetric with respect to p and q.
i.e. B (p, q) = B (q, p).
Since (n) = (n 1)! , (m) = (m 1)!
(m 1)!(n 1)!
Then B (m, n) = , m, n are positive integers. (6.10)
(m n 1)!

2 2
0 0
Also B ( 1 , 1 ) = 2 cos sin d = 2 d = .
2 2
0 0
2
[( 1 )]
1 1
But B ( , ) = 2 = [( 1 )]2 = , then ( 1 ) = (another proof.)
2 2 (1) 2 2

If p > 0 any real number, n > 0 positive integer then


(p)(n) (p).(n 1)!
B (p, n) = = =
(p n) (p n 1)(p n 2) p.(p)
(n 1)!
= . (6.11)
(p n 1)(p n 2) p
2
If we put x = y/(l + y) then we get dx = dy/(l + y) and
p 1 q 1
y 1 dy y p 1
B (p, q) = = dy , (6.12)
pq
0 1 y 1 y (1 y) 2 0 (1 y)
this is the third expression for the beta function.
Example 6.1:
x p 1
Given 1 x dx = sin p , show that (p)(1 p) = sin p , where 0 < p < 1.
0
Solution:
Letting q = 1 p in equation (6.12) we get
y p 1
B (p, 1 p) = dy = .
0 1 y sin p
(p)(1 p)
From (6.9) we get B (p, 1 p) = = .
(1) sin p

161
Advanced Engineering Mathematics Chapter 6 - Special Functions


Then (p)(1 p) = , (6.13)
sin p
Example 6.2:
2q1
Prove the duplication formula: 2 (q).(q+ 1 ) = (2q).
2
Solution:

2 2
Let I = sin 2p xdx , J = sin 2p 2xdx . From (6.8), (6.9) it is clear that
0 0

2
2p 1 (p)(q) 1
sin sin 2q 1 d = = B (p , q).
0 2 (p q) 2

1 1 1 (p 1 )( 1 )
Then I = B (p+ , ) = 2 2 , (6.14)
2 2 2 2(p 1)
In the integral J, let 2x = u, we get

1 2p 2
J = sin 2udu = sin 2p udu = I.
20 0

2 2
2p 2p 2p 2p
But J = (2sin x cos x) dx = 2 sin x cos x dx =
0 0

2p 1
2(2p 1) [(p 1 )]2
=2 B(p 1 ,p 1 ) = 2 . (6.15)
2 2 (2p 1)
Then since I = J,
(p 1 )( 1 ) 2(2p 1) [(p 1 )]2
2 2 = 2
2(p 1) (2p 1)
Therefore
2p
2 (p + 1 ) (p+1) = (2p + 1).
2
Let p + 1 = q, we get
2
2q1
2 (q).(q+ 1 ) = (2q). (6.16)
2

162
Advanced Engineering Mathematics Chapter 6 - Special Functions

6.3.1 Bessel Functions:


Bessel functions were first introduced by Bessel, in 1824, in the discussion of
a problem in dynamical astronomy. Bessel differential equation appears in the
solution of Laplace's equation 2 U = 0 in cylindrical coordinates. Now we shall
show how Bessel's equation emerges from physical problems. The wave-equation,
with x, y, z as Cartesian coordinates and t as time, is
2V 2V 2V 1 2V
(6.17)
x 2 y 2 z 2 C2 t 2
In cylindrical coordinates with x y r sin G the equation is x = r cos , y = r sin ,
the equation is
2V 1 V 1 2 V 1 2 V
. (6.18)
r 2 r r r 2 2 C2 t 2
Seek solutions of the form where V = R(r).().T(t), (6.19)
where, by the method of separation of variables, we get after dividing both sides by
V:
R " 1 R ' 1 " Z" 1 T"
(6.20)
R r R r2 Z C2 T
The left-hand side is independent of t, being only a function of r, and z; and the
right-hand side is independent of r, and z, being only a function of t. Since the two
sides are identically equal, this implies that each is independent both of r, , z and t,
2
and must therefore be constant. Putting this constant equal to p , we find
2 2
T" = C p T, (6.21)
2
The constant (p ) which we introduced, is known as the separation constant. There
2
was no reason why the separation constant should have a negative value of (p )
except that this enabled us to obtain harmonic solutions. Equation (6.20) reduces to
R " 1 R ' 1 " Z"
p2 (6.22)
R r R r2 Z
2
Similarly, put Z" = q Z (6.23)
R " 1 R ' 1 "
We get q 2 p2 n 2 . (6.24)
R r R r 2

163
Advanced Engineering Mathematics Chapter 6 - Special Functions

2
Now let " = m (6.25)
Equation (6.24) becomes
R " 1 R ' m2
n2
R r R r2
2 2 2
or r R" + r R' + (r n ) R = 0 (6.26)
The equation (6.26) for R is Bessel's differential equation.
Bessel function arise as solution of the differential equation
2 2 2
x y" + x y' + (x n ) y = 0, n 0 (6.26')
To obtain a solution in series by the Frobenius method, let

y = a r x (C r)
r 0
Substituting we get

[(C r)2 n 2 ]a r x (C r) + a r x (C r 2) = 0.
r 0 r 0
Equating coefficients of various powers of x to zero, we obtain the indicial equation
2 2
is C n = 0, i.e. C = n.
2 2
and [(C+1) n ] a1 = 0. Hence a1 = 0 and
a r 2
ar = , r = 2, 3, 4, .
(C r) 2 n 2
a r 2
Now, let C = n and ao = 1, we get ar = , r = 2, 3, 4, , and
2 2
(n r) n
n 1 1 1
y1 = x 1 x2 x4 x 6
4(n 1) 42.2!(n 1)(n 2) 43.3!(n 1)(n 2)(n 3)
a r 2
If C = n, ao = 1, we get ar = , r = 2, 3, 4, ,
2 2
(r n) n
k 1 1 1
y2 = x 1 x2 x4 x 6
4(1 n) 42.2!(1 n)(2 n) 43.3!(1 n)(2 n)(3 n)
The coefficients of y1, are finite except when n is a negative integer. Excluding this
n
case, we standardise the solution by taking ao = l/2 (n + l) in general and
n
ao = l/(2 . n!) when n is a positive integer. Since

164
Advanced Engineering Mathematics Chapter 6 - Special Functions

a 2r 1
= 0 as r,
a 2r 2 4r(n r)
The series converges for any finite value of x, and thus we have the first solution
y = Jn(x), where
n 2r
(1)r x
Jn(x) = , (6.27)
r 0 r!(n r 1) 2
in general. In particular, where n is a positive integer,
n 2r
(1) r x
Jn(x) = .
r 0 r!(n r)! 2
This function Jn(x) is known as the Bessel Function of the first kind of order n.
When n is not integer the second solution may be obtained by replacing n by
n; it is therefore
2r n
(1)r x
Jn(x) = ,
r 0 r!(r n 1) 2
and the general solution of the Bessel equation may be expressed as
y = A Jn(x) + B Jn(x).
But when n is an integer, and since n appears in the differential equation only
2
as n there is no loss of generality in taking it to be a positive integer, Jn (x) is not
distinct from Jn(x). For the denominators of the first n terms of the series Jn(x)
contains respectively the factors (l n), (2 n), , (0) which are infinite, and so
these terms vanish. Thus
2r n
(1)r x
Jn(x) = ,
r n r!(r n 1) 2
let r = k + n, we get
2k n
(1)k n x n
Jn(x) = = (1) Jn(x), (6.28)
k 0 (k n)!(k 1) 2
In this case a modification of the method of Frobenius is necessary to give an
independent second solution.
We shall define the Bessel function of the second kind of order n as

165
Advanced Engineering Mathematics Chapter 6 - Special Functions

J n (x)cos n J n (x)
n 0,1,2,3,
sin n
Yn (x) (6.29)
J (x)cos p J p (x)
lim p n = 0,1,2,3,
p n sin p

The Bessel function of the second kind of order n is given also by


1 J (x) (1)n J (x)
Yn(x) = (6.30)
n n
The general solution of equation (6.26') is given by y = A Jn(x) + B Yn(x).
6.3.2 Asymptotic formulas:
In certain physical problem it is desirable to know the value of a Bessel
function for large values of its argument. In the following we shall derive the
asymptotic formulas. We put y = z/ x in Bessel's differential equation
2 2 2
x y" + x y' + (x n )y = 0, we get
3/2
y' = z' x x z,
3/2 5/2
y" = z" x z' x +zx , and
1 4n 2
z" 1 z 0
2
4x

For large values of x the differential equation is reduced to
z" + z = 0 then z = A sin x + B cos x
sin x cos x
Therefore yA B ,
x x
cos x sin x
where J n (x) , Yn (x)
x x
6.3.3 Recurrence formulas:
It is clear that
d n d (1)r x 2n 2r ( 1) r x 2n 2r 1
dx r
x J n (x) = = =
dx 0 2
n 2r
r!(n r 1) r 0 2 n 2r 1
r!(n r)
n
(1)r x (n 1) 2r n
=x = x Jn1(x).
(n 1) 2r
r 0 2 r![(n 1) r 1)]
d n
i.e. x J n (x) = x n J n 1 (x) . (6.31)
dx

166
Advanced Engineering Mathematics Chapter 6 - Special Functions

Similarly, we can prove that


d n
x J n (x) = x n J n 1 (x) . (6.32)
dx
From (6.31) it is clear that
x n J 'n (x) nx n 1J n x n J n 1 (x) .
i.e. xJ 'n (x) xJ n 1 (x) nJ n (x) . (6.33)
From (6.32) we get
x n J 'n (x) nx n 1J n x n J n 1 (x) .
or xJ 'n (x) nJ n (x) xJ n 1 (x) . (6.34)
Adding (6.33) and (6.34) and dividing by 2x gives
1
J 'n (x) [J n 1 (x) J n 1 (x)] . (6.35)
2
Subtracting (6.34) from (6.33) and dividing by x we get
2n
J n 1 (x) J n 1 (x) J n (x) .
x
2n
i.e. J n 1 (x) J n (x) J n 1 (x) . (6.36)
x
The functions Yn(x) satisfy exactly the same results as those above (6.31) to (6.36),
where Yn(x) replaces Jn(x).
Example 6.3:
Find J3(x) in terms of J0(x) and J1(x).
Solution:
In (6.36), put n = 2 we get
4
J3(x) = J2(x) J1(x).
x
2
In (6.36), put n = 1, J2(x) = J1(x) J0(x).
x
4 2 8 4
Therefore J3(x) = [ J1(x) J0(x)] J1(x) = ( 1) J1(x) J0(x).
x x x2 x
8 4
J3(x) = ( 1) J1(x) J0(x).
x2 x

Example 6.4:

167
Advanced Engineering Mathematics Chapter 6 - Special Functions

2 2
Prove that a) J 1 (x) sin x , b) J 1 (x) cosx .
2 x 2 x
Solution:
2r 1
r
( 1) ( x ) 2
a) J 1 (x) 2
3
2
r 0 r!(r 2 )

(r 1 )(2r)!
Since (r+3/2) = (r+) (r+) = 2 ,
22r r!
2r 2r 1
r
(1) ( x ) 22
2x (1)r x 2r
then J 1 (x) 2 = =
2 1
r 0 (r 2 )(2r)! r 0 (2r 1)!

2 (1)r x 2r 1 2
= = sin x .
x r 0 (2r 1)! x
1 1
r x 2r 2 r x 2r 2 2r
( 1) ( ) (1) ( ) 2 2 (1)r x 2r
b) J 1 (x) 2 = 2 = =
2
r 0 r! (r 1)
r 0 (2r)! x r 0 (2r)!
2
2
= cos x .
x

6.3.4 Integrals involving Bessel functions:


From equations (6.31) and (6.32) we have the relations:
n n
x J n 1 (x)dx = x Jn(x) + C, (6.37)
n n
x J n 1 (x)dx = x Jn(x) + C. (6.38)

Example 6.5:
J (x)
Evaluate a) x 3J 2 (x)dx , b) 1 dx , c) x 2 J1 (x)dx
x
Solutions:
a) In (6.37) put n = 3, we get x 3J 2 (x)dx = x J3(x) + C
3

2
b) In (6.36) put n = 1, we get J1(x) = J0(x) + J2(x).
x
J (x) 1 1
Therefore 1 dx = J 0 (x)dx + J 2 (x)dx ,
x 2 2

168
Advanced Engineering Mathematics Chapter 6 - Special Functions

J (x) J (x) J (x)


and J 2 (x)dx = x 2 dx = x d 1 = J1(x) + 1 dx .
x x x
J (x) 1 1 1 J (x)
Hence 1 dx = J 0 (x)dx J1(x) + 1 dx .
x 2 2 2 x
J (x)
i.e. 1 dx = J 0 (x)dx J1(x).
x
c) In (6.32) put n = 0 and in (6.37) put n = 1, we get
d
J0(x) = J1(x), xJ 0 (x)dx = x J1(x) + C.
dx
and x 2 J1 (x)dx = x 2 d[J 0 (x)] = x J0(x) + 2x J 0 (x)dx =
= x J0(x) + 2 x J 0 (x)dx = x J0(x) + 2 x J1(x) + C.

Example 6.6:
J (x)
Evaluate a) J3 (x)dx , b) 2 dx ,
2
x
Solutions:
J (x) J (x) J (x)
a) J3 (x)dx = x 2 3 dx = x 2 d 2 = J2(x) + 2 2x dx =
x2 x x2
J (x) J (x)
= J2(x) + 2 2 dx = J2(x) 2 1 + C.
x x
4
b) In (6.36) put n = 2 we get J3(x) = J2(x) J1(x).
x
J 2 (x) J1 (x) J3 (x)
i.e. .
x2 4x 4x
J3 (x) J3 (x) J 2 (x) J 2 (x) J 2 (x)
x dx = x 2 dx = x d 2 = x 2 2x dx .
x x x
From Example (6.5)(b) we have
J1 (x)
x dx = J1(x) + J 0 (x)dx .
Therefore
J 2 (x) 1 1 J (x) 1 J 2 (x)
dx = J1(x) + J 0 (x)dx 0 + dx
x2 4 4 4x 4 x2
and
3 J 2 (x) 1 J (x) 1
dx = J1(x) 0 + J 0 (x)dx ,
4 x2 4 4x 4

169
Advanced Engineering Mathematics Chapter 6 - Special Functions

J 2 (x) 1 J (x) 1
i.e. dx = J1(x) 0 + J 0 (x)dx .
x2 3 3x 3
6.3.5 Orthogonality of Bessel functions:
It is clear that, y = Jn( x), z = Jn( x) are solutions of the equations
2 2 2 2
x y" + x y' + ( x n ) y = 0,
2 2 2 2
x z" + x z' + ( x n ) z = 0,
Multiplying the first equation by z/x, the second by y/x and subtracting, we find
2 2
x [z y" y z"] + [z y' y z'] = ( ) x y z,
this can be written as
d 2 2
[ x (z y' y z')] = ( ) x y z.
dx
Then by integrating we get
2 2
( ) x y z dx = x (z y' y z') + constant.
d 2 2
[ x (z y' y z')] = ( ) x y z.
dx
d d
Jn( x) = J 'n (x) , and if ( ) 0,
2 2
Since y' = Jn( x) = then
dx d(x)
1 ' '
xJ n (x)J n (x)dx = 2 2 x[Jn( x) J n ( x) Jn( x) J n ( x)] + C.

Thus

170
Advanced Engineering Mathematics Chapter 6 - Special Functions

1 1
xJ n (x)J n (x)dx = .[Jn( x) J 'n ( x) Jn( x) J 'n ( x)] (6.39)
0 2 2
Let , and using L'Hospital's rule, ( is considered constant) we find
1
2 J n ()J 'n () J n ()J 'n ()
xJ n (x)dx = lim 2 2
=
0
J 'n ()J 'n () J n ()J 'n () J n ()J"n ()
= lim =
2
[J 'n ( )]2 J n ()J 'n () J n ()J"n ()
= .
2
But since J"n () + J 'n () + ( ) Jn() = 0, we find
2 2 2

[J 'n ()]2 J n ()J 'n () J n ()J"n () 1 ' 2 2


2 ( n ) 2
= [J n ()] J n ( ) .
2 2 2 2
1
2 1 ' 2 n2 2
i.e. xJ n (x)dx = [J n ()] (1 2 )J n () . (6.40)
0 2
Now, if and are any two different roots of the transcendental equations
A Jn(x) + B x J 'n (x) = 0, (6.41)
where A and B are constants, then
A Jn() + B J 'n () = 0, A Jn() + B J 'n () = 0.
Multiplying the first equation by Jn(), the second by Jn() and subtracting, we find
Jn() J 'n () Jn() J 'n () = 0 if A B 0.
1
Therefore xJ n (x)J n (x)dx 0
0
In case A = 0, B 0 or A 0, B = 0, the result is also easily proved.
Expansion of functions in terms of Bessel functions:
If we now suppose that we can expand an arbitrary function f(x) in the form
f(x) = a i J n (i x) (6.42)
i
where the sum is taken over the positive roots of the equation (6.41), then we can
determine the coefficients ai as follows:
Multiply both sides of equation (6.42) by x Jn (i x) and integrate with respect to x
from 0 to 1; then

171
Advanced Engineering Mathematics Chapter 6 - Special Functions

1 1
xf (x)J n (i x)dx = a i xJ n (i x)J n ( jx)dx ,
0 i 0
11
from which it follows that aj =
C0
x f(x) J n ( j x) dx , (6.43)

[J n ( )]2 2 2 2 2 2
where C = [B + A B n ].
2 2
2B
Because of its similarity to a Fourier series, a series of the type (6.42) is called a
Fourier-Bessel series. We can show that if f (x) satisfies the Dirichlet conditions then
at every point of continuity of f (x) in the interval [0, l] the series (6.42) converges to
1
f(x). At any point of discontinuity the series (6.42) converges to [f(x+0) + f(x0)].
2
6.3.6 Generating function for Jn(x):
x
(t 1 )
The function e 2 t = J n (x) t n (6.44)
n
is called the generating function for Bessel functions of the first kind of integral
order.
6.3.7 Functions related to Bessel functions:
1. Hankel functions of the first and second kinds are defined respectively by

n J n x i Yn x ,
H (1)
(6.45)
H (2)
n J n x i Yn x .
2. Modified Bessel functions:
The modified Bessel function of the first kind of order n is defined as
n ni/2
In(x) = i Jn( ix) = e Jn(ix) (6.46)
If n is an integer In(x) = In(x), but if n is not an integer, In(x) and In(x) are linearly
independent.
The modified Bessel function of the second kind of order n is defined as
I n (x) In (x)
2 n 0,1, 2,3,
sin n
K n (x) (6.47)
lim I p (x) I p (x)
n 0,1,2,3,
p n 2 sin p


172
Advanced Engineering Mathematics Chapter 6 - Special Functions

These functions satisfy the differential equation


2 2 2
x y" + x y' (x + n ) y = 0
and the general solution of this equation is y = A In(x) + B Kn(x),
or if n 0, l, 2, 3, y = R In(x) + S In(x), where A, B, R and S are constants.
Equality (6.46) is very useful for deducing properties of the modified Bessel function
In(x) from those of the Bessel function Jn(x). For instance, when n is an integer it
follows from equation (6.28) that
I n(x) = In(x) (6.48)
and from equations (6.31) to (6.36) respectively that
d n n
[x I n (x)] = x In1(x), (6.49)
dx
d n n
[x In (x)] = x In+1(x), (6.50)
dx
x I'n (x) = x In1(x) n In(x), (6.51)
x I'n (x) = n In(x) + x In+1(x), (6.52)
1
I'n (x) = [In1(x) + In+1(x)], (6.53)
2
2n
In (x) = In1(x) In+1(x), (6.54)
x
I'0 (x) = I1(x). (6.55)
3. Ber. Bei. Ker. Kei functions:
Kelvin introduced two new functions Bern (x) and Bein (x) which are respectively the
3/2
real and imaginary parts of Jn (i x) where
3/2 3i/4 2 3/2
i =e = [1 i], i.e. Jn( i x) = Bern(x) + i Bein(x)
2
In a similar way the functions Kern (x) and Kein (x) are defined to be respectively the
ni/2 i/2 i/4 2
real and imaginary parts of e Kn( i x), where i = e = [1 + i],
2
in/2
i.e. e Kn( i x) = Kern(x) + i Kein(x).
2 2 2
The functions are useful in connection with the equation x y"+x y' (i x + n ) y = 0,

173
Advanced Engineering Mathematics Chapter 6 - Special Functions

which arise in electrical engineering and other fields. The general solution of the
3/2
equation is y = A Jn(i x) + B Kn( i x). The four functions Ber, Bei, Ker and Kei
are used more often in electrical engineering than they are in physics or chemistry.

Exercises 6
1) Evaluate each of the following:
(7) (3)(3 / 2) 1 3 5
a) b) c) ( )( )( )
2(4)(3) (9 / 2) 2 2 2
(6) 5 1 6(8 / 3)
d) e) ( ) / ( ) f)
4(3) 2 2 5(2 / 3)
3/2 3/2
[Answers: a) 30, b) 16/105, c) (3/8) , c) (3/8) , e) 3/4, f) 4/3].
2) Prove that
45 3
3 x 6 2x x
a) x e dx = 6 , b) x e dx = , c) xe dx = 3 .
0 0 8 0
e st 1
x3 1 1
d) dt ,s>0, e) e dx = ( ) , f) (n x) 4 dx = 24 ,
0 t s 0 3 3 0
1 3
g) (x n x)3dx = .
0 128

3) Evaluate each of the following:


a) B(3, 5), b) B(3/2, 2), C) B(1/3, 2/3),
1 a
4 3
d) x (1 x) dx , e) x 4 a 2 x 2 dx .
0 0
6
[Answers: a) 1/105, b) 4/15, c) 2/ 3 , d) 1/280, e) a /32 ].
4) Prove that
q 1 p 1
B (p, q) = B(p, ql) = B(pl, q).
p q 1 p q 1
5) Prove that a B (a, b) = (b 1) B(a + l, b 1).
6) Show that B (a, b) = B(a, b 1) B(a + l, b 1).
/ 2 n 1 m 1 nm
7) Show that cos m x sin n x dx = [ ].[ ]/2[ +1]
0 2 2 2

174
Advanced Engineering Mathematics Chapter 6 - Special Functions


dx [( 1 )]2
8) Show that = 4
0 3 cos x 4
x 2 dx
dx
9) Show that
4
= , 4
= .
0 1 x 2 2 0 1 x 2 2

1
p
10) Show that x p 1 (1 x m )q 1 dx = 1 B( , q), p , q , m > 0. [Hint: put x = y].
m
m m
0
11) Show that (a) (a + ) = 2 21/ 2 2s (2a), a > 0.
12) Prove that
/ 2 / 2 8
6
a) sin xdx = 5/32, b) sin 4 x cos5 x dx = .
0 0 315
3 2 16
3
c) cos 4 x dx = , d) x 8 x 3 dx = .
0 8 0 9 3
13) Show that
/ 2 / 2
m m
sin x dx = cos x dx =
0 0
1.3.5 (m 1)
2.4.6 m 2 if m is an even positive integer
=
2.4.6 (m 1) if m is an odd positive integer
1.3.5 m
14) Show that
/ 2 2
a) sin 3 x cos 2 x dx = 2/15, b) 8
sin x dx = 35/64,
0 0
/ 2 xdx 3 dx
c) tan x dx = / 2, d) , e) .
6 3 3
0 0 1 x 0 3x x 2

2 sin x x cos x 2 x sin x cos x


15) Show that a) J 3 (x) , b) J (x) .
x x 3 x x
2 2

16) Show that a) x 4 J1 (x)dx = x J2(x) 2 x J3(x) + C.


2 3

b) x 3J3 (x)dx = x J2(x) 5 x J1(x) 15 x J0(x) + 15 J 0 (x)dx ,


3 2

1
c) x 3J 0 (x)dx = 2 J0(l) 3 J1(1),
0

175
Advanced Engineering Mathematics Chapter 6 - Special Functions

d) x 2 J 0 (x)dx = x J1(x) + x J0(x) J 0 (x)dx .


2

2 (3 x 2 )sin x 3x cos x
17) Show that a) J 5 (x) ,
x x 2
2
2 3x sin x (3 x 2 )cos x
b) J 5 (x) .
2 x x2

18) Show that J 0 (x)sin xdx = x J0(x) sin x x J1(x) cos x + C.
1
19) Prove that a) J "n (x) = [ Jn2(x) 2 Jn(x) + Jn+2(x)]
4
1
b) J "n (x) = [Jn3(x) 3 Jn1(x) + 3Jn+1(x) Jn+3(x) ]
8
J (x) 1 1 1 1
20) Prove that a) 3 dx = J1(x) J2(x) J3(x) + J 0 (x)dx .
3 15 15x 15x 15
x
3
b) J1 ( 3 x )dx = 6 3 x J1 ( 3 x ) 3 x 2 J 0 ( 3 x ) C .

176
Advanced Engineering Mathematics Chapter 7- Partial Differential Equations

CHAPTER 7
Partial Differential Equations
7.1 Classification of equations:
An equation containing partial derivatives of unknown functions ul, u2,, un
is said to be partial differential equation. A partial differential equation is said to be
th th
of the n order if it contains at least one partial derivative of the n order and no
partial derivatives of order higher than n.
A partial differential equation is called linear if it is linear in the unknown
functions and in their derivatives; it is called quasi-linear if it is linear in the highest
order derivatives of the unknown functions.
2u 2
Thus, for instance, the equation = 5x 3y , is a partial differential
xy
equation of order two, or a second order partial differential equation.
2u 2u
The equation + f(x, y) = 2u is a linear partial differential equation
x 2 y 2

of the second order with respect to u.


u 2 u u 2 u 2
The equation + + u = 0, is quasi -linear partial
x x 2 y y 2

differential equation of the second order with respect to the unknown function u.
2 u 2
u
The equation + = u, is neither linear nor quasi-linear with respect to u.
x y
By a solution of an equation containing partial derivatives we mean a system
of functions which, when we put in the equation in place of the unknown functions,
turns the equation into an identity in the independent variables. The general solution
is a solution, which contains a number of arbitrary independent functions equal to the
order of the equation. A particular solution is one which can be obtained from the
general solution by particular choice of the arbitrary functions.
For example; as seen by substitution, u = 5 x y 3 x y + F(x) + G(y) is a solution of
3 2
3 2
the following partial differential equation

177
Advanced Engineering Mathematics Chapter 7- Partial Differential Equations

2u 2
= 5x 3y. Because it contains two arbitrary independent functions F(x) and
xy
3
G(y), it is the general solution. If in particular F(x) = 2 cos x, G(y) = 2y 7, we
obtain the particular solution u = 5 x y 3 x y + 2 cos x + 2y 7.
3 2 3
3 2
A singular solution is one which cannot be obtained from the general solution by
particular choice of the arbitrary functions.
A boundary-values problem involving a partial differential equation seeks all
solutions of a partial differential equation which satisfy conditions called boundary
conditions.
We shall be primarily interested in linear equations of the second order in one
unknown function. The general linear partial differential equation (will be
abbreviated to P.D.E.) of order two in two independent variables has the form
2u 2u 2u u u
A +B +C +D +E + F u = G, (7.1)
x 2 x y y 2 x y

where A, B, C, D, E, F and G; may depend on x and y but not on u. If G = 0 the


equation is called homogeneous, while if G 0 it is called non-homogeneous.
Because of the nature of the solutions of the equation (7.1) is often classified as
2
elliptic, hyperbolic or parabolic according as B 4 A C = is less than, greater than or
equal to zero respectively
< 0 EllipticEquation,

i.e. B2 4AC = 0 Parabolic Equation,
> 0 Hyperbolic Equation.

Example 7.1:
Determine whether each of the following partial differential equations is linear or
non-linear, state the order of each equation, and name the dependent and independent
variables.
u 2
2 u
a) = a . It is linear of order 2, dependent variable u, independent
y x 2

variables x and y.

178
Advanced Engineering Mathematics Chapter 7- Partial Differential Equations

3
2 z
2
2 z
b) x =y . It is linear of order 3, dependent variable z, independent
y 3 x 2

variables x and y.
2 2u 2u 2u
c) u = 0. i.e. + + = 0. It is linear of order 2, dependent variable
x 2 y 2 z 2

u, independent variables x, y and z.


2 u 2
u
d) x + y = 1. It is nonlinear of order 1, dependent variable u,

independent variables x and y.

Example 7.2:
Classify each of the following equations as elliptic, hyperbolic or parabolic.
2u 2u 2u
a) + + = 0, A = B = C = 1.
x 2 y 2 z 2
2
Then B 4AC = 3 < 0, and the equation is elliptic.
2u 2
2 u 2
b) =a , A = a , B = 0 , C = 1.
t 2 x 2
2 2
Then B 4AC = 4 a > 0, and the equation is hyperbolic.
u 2
2 u 2
c) =a , A = a , B = 0 , C = 0.
t x 2
2
Then B 4AC = 0 and the equation is parabolic.
2u 2u 2u u u
d) 4 +3 + +2 3 + 2 u = 3x 2y,
x 2 xy y 2 x y
2
A = 4, B = 3, C = l. Then B 4AC = 9 16 = 7 < 0, and the equation is elliptic.
2u 2u 2 u
e) y +x + 3y = 0 , A = y, B = 0, C = x.
x 2 y 2 x
2
Then B 4AC = 4xy. In the region x y > 0, the equation is elliptic. In the region
x y < 0, the equation is hyperbolic. If x y = 0, the equation is parabolic.

179
Advanced Engineering Mathematics Chapter 7- Partial Differential Equations

7.2 Basic types of equations of Mathematical Physics:


The basic equations of Mathematical Physics are the following second order
partial differential equation.
I) Wave equation:

2u 2
2 u
=a
t 2 x 2

This equation describes the motion of a beam, which can


vibrate longitudinally in the x-direction. The variable u(x, t) is the longitudinal
displacement from the equilibrium position of the cross section at x. The constant
2
a = g E/, where g is the acceleration due to the gravity, E is the modulus of
elasticity (stress divided by strain) and depend on the properties of the beam, is the
density (mass per unit volume).
This equation is applicable also to the small transverse vibrations of a taut,
flexible string, such as a violin string, initially on the x-axis and set into motion.
The function u(x, t) is the
displacement of any point x
of the string at time t.
2
The constant a = T/ where T is the tension in the string (constant), and is the
mass per unit length of the string (constant). It is assumed that no external forces act
on the string but it vibrates only due to its elasticity. This equation is the simplest of
the class of hyperbolic equations,
II. Fourier equation for heat conduction:
u 2u
=k . One-dimensional heat conduction.
t x 2
u 2
= k u. Three-dimensional heat conduction.
t
This equation describes the flow of heat. Its derivation is based on Newtons low.
Also this equation describes the filtration of liquids and gasses in a porous medium
(e.g. the filtration of oil and gas in subterranean sandstones). This equation is the

180
Advanced Engineering Mathematics Chapter 7- Partial Differential Equations

simplest of the class of parabolic equation. The constant k, called the diffusivity, is
equal to k / , where the thermal conductivity k, the specific heat and the density
are assumed constants.
III Laplaces equation:
2
u=0
This equation occurs in many fields. In the theory of electric and magnetic fields u
represents the magnetic or electric potential respectively. For this reason the equation
is often called the potential equation. In the theory of heat conduction, u is the steady
state temperature, i.e. the temperature after a long time has elapsed, and is equivalent
u
to putting = 0, in the heat conduction equation.
t
7.3 Formation of partial differential equation by elimination:
Partial differential equations can often be formed by elimination of either
arbitrary constant or arbitrary functions.
Example 7.3:
Eliminate the arbitrary functions f and g from y = f(x at) + g(x + at)
Solution:
y
It is clear that = f (x at) + g(x + at), and
x
2 y
= f (x at) + g(x + at). (1)
x 2
y
Similarly = a f (x at) + a g(x + at), and
t
2 y 2 2
= a f (x at) + a g(x + at). (2)
t 2
2
2 y 2 y
From (1) and (2) we get a = a partial differential equation of the second
x 2 t 2

order (see wave equation).


Example 7.4:
Eliminate the arbitrary function f from z = f( x ).
y

181
Advanced Engineering Mathematics Chapter 7- Partial Differential Equations

Solution:
z 1 x z z z
It is clear that = f '( ) , = x2 f '( x ) . So x + y = 0.
x y y y y y x y

Example 7.5:
t
Eliminate A and from z = a e sin x.
Solution:
Differentiating partially with respect to x, we get
z t
= A e cos x,
x
2z 2 t
and = A e sin x. (1)
x 2
Differentiating partially with respect to t, we get
z t
= A e sin x,
t
2z 2 t
and = A e sin x. (2)
t 2
2z 2z
From (1) and (2) we get + = 0.
x 2 t 2
th
As we know, every ordinary differential equation of the n order may be
regarded as derived from a solution containing n arbitrary constants. It might be
th
supposed that every partial differential equation of the n order was similarly
derivable from a solution containing n arbitrary functions. However, this is not true.
In general, it is impossible to express the elimination of n arbitrary functions as a
partial differential equation of order n. An equation of higher order is required, and
the result is not unique.
Example 7.6:
t
Show that u(x, t) = e sin x, is a solution to the boundary-value problem
u 2 u
= , u(0 , t) = u( 2 , t) = 0, u(x , 0) = sin x.
t x 2

Solution:
182
Advanced Engineering Mathematics Chapter 7- Partial Differential Equations

t
From u(x, t) = e sin x, we have
sin 0 = 0, u( 2 , t) = e sin 2 = 0, u(x, 0) = e sin x = sin x.
t t 0
u (0, t) = e

i.e. the boundary conditions are satisfied.
u t u t 2u 2 t
Also = e sin x, = e cos x, = e sin x.
t x x 2
Then substituting into the differential equation, we have
u 2 t 2u
= e sin x = , which is an identity.
t x 2

7.4 Methods of finding solutions:


There are many methods by which partial differential equation can be solved.
The following theorems are of fundamental importance.
7.4:1 General solution:
Theorem 7.1: Superposition principle:
If u1, u2, ..., un are solutions of a linear homogeneous partial differential equation,
then cl u1+ c2 u2+...+ cnun, where cl, c2, ..., cn are constants is also a solution.
Theorem 7.2:
The general solution of a linear non-homogeneous partial differential equation is
obtained by adding a particular solution of the non-homogeneous equation to the
general solution of the homogeneous equation.
If A, B, C, D, E and F in equation (7.1) are constants, then the general
ax + by
solution of homogeneous equation can be found by assuming that u = e , where
a and b are constants to be determined.
We can sometimes find general solutions by using the methods of ordinary
differential equation.

Example 7.7:
2u 2
a) Solve the equation =xy .
xy
2
b) Find the particular solution for which u(x, 0) = x , u(1,y) = cos y.

183
Advanced Engineering Mathematics Chapter 7- Partial Differential Equations

Solution:
u 2
a) Write the equation as = x y .
x y
Then integrating with respect to x, we find
u 1 2 2
= x y + f(y), (1)
y 2
where f(y) is an arbitrary function. Integrating (1) with respect to y we get
2 3
u = 1 x y + f(y) dy + g(x), (2)
6
where g(x) is an arbitrary function. The result (2) can be written
2 3
u = u(x, y) = 1 x y + h(y) + g(x), (3)
6
which has two arbitrary functions and is therefore a general solution,
2
b) Since u(x, 0) = x , we have from (3)
2 2
x = h (0) + g(x), or g(x) = x h (0) (4)
2 3 2
Thus u = 1 x y + h(y) + x h (0). (5)
6
Since u(l , y) = cos y, we have from (5)
3
cos y = 1 y + h(y) + 1 h (0),
6
3
or h(y) = cos y 1 y 1 + h (0), (6)
6
Thus using (6) in (5) we find the required solution
2 3 2 3
u = 1 x y + x + cos y 1 y 1.
6 6
Example 7.8:
2u u
Solve t + 3 = x.
xt x
Solution:
We can write this equation in the form
u
[t + 3 u] = x
x t
Integrating with respect to x, we get
u 2 u u x2 f (t)
t + 3 u = 1 x + f (t) or +3 = + .
t 2 t t 2t t
184
Advanced Engineering Mathematics Chapter 7- Partial Differential Equations

This is a linear equation having integrating factor


3
dt 3 n t n t 3 3
e t =e =e =t .
3 2 2 2
Then [ t u ] = 1 t x + t f(t).
t 2
Integrating, we get
3 3 2 2 3 2
t u = 1 t x + t f(t) dt + h(x) = 1 t x + g(t) + h(x),
6 6
the required solution.
Example 7.9:
2u 2u 2u
Find solutions of +2 3 =0
x 2 x y y 2

Solution:
ax + by
Assume u = e . Substituting in the given equation, we find
2 2 ax + by 2 2
(a + 2 a b 3b ) e =0 or a + 2 a b 3b = 0 .
b(x + y)
Then (a b)(a + 3b) = 0 and a = b or a = 3b. If a = b, e is a solution for any
b(y 3x)
value of b. If a = 3b, e is a solution for any value of b.
Since the equation is linear and homogenous, sum of these solutions is a
solution. The general solution found by addition is given by u = f ( x + y) + g(y 3x).
Now, we shall obtain some special types of the partial differential equations.
7.4:2 D'Alembert methods:
Let the equation to be solved is
2 1 2
= (7.2)
x 2 c2 t 2
We change to new variables u = x ct and v = x + ct.
1
i.e. x = 1 (u + v), t = [v u].
2 c

Then it is easily verified that transforms to 1 ( + ),
x 2 u v

transforms to c + c ;
t u v

185
Advanced Engineering Mathematics Chapter 7- Partial Differential Equations

2 2 2 2
becomes +2 + and finally
x 2 u 2 u v v 2
2
2
2 2 2
becomes c [ 2 + ]. When these changes are made, the
t 2 u 2 u v v 2

2
equation becomes = 0 ; the most general solution of this is = f(u) + g(v),
uv
f and g being arbitrary functions. In the original variables this is
= f(xct) + g(x + ct). (7.3)
In two dimensions the equation of wave motion is
2 2 1 2
+ = . (7.4)
x 2 y 2 2
c t 2

By a method similar to D'Alembert's, it can be shown that the most general solution
involving only plane waves is
= f( x + my ct) + g( x + my + ct) (7.5)
2 2
where, as before f and g are arbitrary functions and + m = 1.
In three dimensions the differential equation is
2 2 2 1 2
+ + = , (7.6)
x 2 y 2 z 2 2
c t 2

and the must general solution involving only plane waves is


= f( x + m y + n z ct) + g( x + m y + n z + ct), (7.7)
2 2 2
in which + m + n = 1.
7.4:3 Separation of variables:
In this method it is assumed that a solution can be expressed as a product of
unknown functions each of which depends on only one of the independent variables.
The success of the method hinges on being able to write the resulting equation so that
one side depends only on one variable while the other side depends on the remaining
variables so that each side must be a constant. By repetition of this tile unknown can
then be determined. Superposition of these solutions can then be used to find the
actual solution.
Let it be required to find the solution of the equation

186
Advanced Engineering Mathematics Chapter 7- Partial Differential Equations

2u 2
2 u
=a , (7.8)
t 2 x 2

this satisfies the boundary-value conditions


u(0, t) = 0 , u( , t) = 0, (7.9)
u
u(x, 0) = f(x), | = g(x). (7.10)
t t=0
We shall seek a particular solution ( not identically equal to zero) of equation (7.8)
that satisfies the boundary conditions (7.9), in the form of the product of two
functions X(x) and T(t), of which the function X depend only on x, and the function
T depend only on t. i.e. u(x, t) = X(x) T(t) (7.11)
Substituting into equation (7.8), we get
2
X(x) T(t) = a X(x) T(t),
2
and dividing both sides by a X(x) T(t) we get:
T" X"
= . (7.12)
2
a T X
The left hand side of this equation is a function that does not depend on x, the right
hand side is a function that does not depend on t. Equation (7.12) is possible only
when the left and right hand sides are not dependent either on x or on t, that is are
equal to a constant number. We denote it by , where > 0 (later on we will
consider the case < 0).
T" X"
Thus = = .
2
a T X

From these equations we get two equations:


X + X = 0, (7.13)
2
T + a T = 0. (7.14)
The general solutions of these equations are
X(x) = A cos x + B sin x, (7.15)
T(t) = C cos a t + D sin a t, (7.16)
where A, B, C and D are arbitrary constants. Substituting into equation (7.11), we get
u(x, t) = (A cos x+ B sin x) (C cos a t+ D sin a t). (7.17)
Now choose the constants A and B so that the conditions (7.9) are satisfied.

187
Advanced Engineering Mathematics Chapter 7- Partial Differential Equations

Since T (t) 0 (otherwise we would have u(x, t) 0, which contradicts the


hypothesis), the function X(x) must satisfy the conditions (7.9); that is, we must have
X (0) = 0, X () = 0. Putting the value x = 0 and x = into (7.15), we obtain
0 = A.1 + B.0, 0 = A cos + B sin .
It follows that A = 0, B sin = 0, B 0 (otherwise we would have u 0).
Consequently, we must have sin = 0 , whence
= n/, (n = 1, 2, 3, ) (7.18)
(we do not take the value n = 0, since then we would have X 0 and u 0), and so
we have
xn = B sin nx , n = 1, 2, 3, (7.19)

These values of X are called eigenvalues of the given boundary-value problem. The
functions Xn(x) corresponding to them are called eigenfunctions.
2
Now, if in place of , we took ( =) k , then equation (7.13), would take
2
the form: X k X = 0.
kx kx
The general solution is X(x) = A e +Be .
A non-zero solution in this form cannot satisfy the boundary conditions (7.9).
Knowing that = n ; we can write

T (t) = C cos an t + D sin an t, (n = 1, 2, 3, ) (7.20)

For each value of n, hence for every value of , we put the expressions (7.19) and
(7.20) into (7.11) and obtain a solution of equation (7.8) that satisfies the boundary
conditions (7.9). We denote this solution by un(x, t).
un(x, t) = sin nx [C cos ant + D sin ant ]. (7.21)

For each value of n we can take the constants C and D and thus write Cn and Dn (the
constant B is included in Cn and Dn). Since equation (7.8) is linear and homogeneous,
the sum of the solutions is also a solution, and therefore the function represented by

the series u(x, t) = u n (x, t) =
n 1

= [Cn cos ant + Dn sin ant ] sin nx .

n 1
188
Advanced Engineering Mathematics Chapter 7- Partial Differential Equations

Solution (7.22) should satisfy the initial condition (7.10). Put t = 0 in (7.22), we get

f(x) = u(x, 0) = Cn sin nx . (7.23)

n 1
If the function f(x) is such that in the interval (0, ) it may be expanded in a Fourier
series, the condition (7.23), will be fulfilled if we put

Cn = 2 f (x) sin nx dx. (7.24)

0
Series (7.22) will be a solution of equation (7.8) only if the coefficients Cn and Dn are
such that this series converges and that the series resulting from a double term-by-
term differentiation with respect to x and to t converge as well. Differentiate the
terms of (7.22) with respect to t and let t = 0. From the second condition of (7.10), we
get the equality

g(x) = Dn an sin nx .

n 1
We define the Fourier coefficients of this series:

Dn = 2 g(x) sin nx dx. (7.25)
an
0
Thus, we have proved that the series (7.22), where the coefficients Cn and Dn are
defined by formulas (7.24) and (7.25), is a function u(x , t), which is the solution of
equation (7.8) and satisfies the boundary and initial conditions (7.9) and (7.10).
Example 7.l0:
a) Solve the boundary-value problem
u u 3y
=4 , u (0, y) = 8 e , by the method of separation of variables.
x y
3y 5y
b) If u(0, y) = 8 e +4e .
Solution:
Let u = X Y, then X Y = 4 X Y or X / 4 X = Y / Y.
Since X depends only on x and Y depends only on y, and since x and y are
independent variables, each side must be a constant, say C. Then X 4 C x = 0,

189
Advanced Engineering Mathematics Chapter 7- Partial Differential Equations

Y C y = 0,
4Cx Cy
whose solutions are X = A e ,Y=Be .
C(4x+y) C(4x+y)
Therefore u(x, y) = X Y = A B e =De .
a) From the boundary condition,
Cy 3y
u(0, y) = D e =8e ,
which is possible if and only if D = 8 and C = 3.
3(4x+y) 12x3y
Then u(x, y) = 8 e =8e , is the required solution.
, is a solution, then D1 eC1 (4x y) and D2 eC2 (4x y) , are
C(4x+y)
b) Since D e
solutions and by the principle of superposition so also is their sum. i.e. a solution is
u(x, y) = D eC1 (4x y) + D eC2 (4x y) .
1 2

From the boundary condition,


u(0, y) = D1 eC1 y + D2 eC2 y = 8 e + 4 e ,
3y 5y

which is possible if and only if D1= 8, D2 = 4 ; C1=3, C2= 5 or D1= 4, D2 = 8 ;


C1= 5, C2= 3.
3(4x+y) 5(4x+y) 12x3y 20x5y
Then u(x, y) = 8 e +4e =8e +4e ,
is the required solution.

7.4:4 Laplace transform methods:


The Laplace transform of the partial differential equation and associated
boundary conditions are first obtained with respect to one of the independent
variables. We then solve the resulting equation for the Laplace transform of the
required solution which is then found by talking the inverse Laplace transform.
Given the function u(x, t) defined for a x b, t > 0, and satisfies the
conditions for which L {u(x, t)} exists. Using Leibnizs rule for differentiating under

the integral sign, we get:


u d st d dU
L{ }= e st u L {u} =
x x
dt =
dx
e u dt =
dx dx
,
0 0
2u d2U
and L { }= .
x 2 dx 2

190
Advanced Engineering Mathematics Chapter 7- Partial Differential Equations

u u
But L { } = e st dt = s est u(x, t) dt u(x , 0) =
t t
0 0
= s U(x, s) u(x, 0) = s U u(x, 0), (1)
where U = U(x, s) = L{u(x, t)}.
u
Put v = in (1) we get
t
2u v u
L{ } = L { } = s V v(x, 0) = s L {v} |t=0=
t 2 t t
u
= s [s L {u} u(x, 0)] | =
t t=0
2 u
= s U s u(x, 0) |t=0.
t

Example 7.11:
u u 3x
Find the solution of =2 + u, u(x, 0) = 6 e , which is bounded for x > 0, t > 0.
x t
Solution:
Taking the Laplace transform of the given differential equation with respect to t, we
find
u u
L{ } = 2L{ } + L{u}.
x t
dU 3x
Hence = (2s+1) U 2 u(x, 0) = (2s+1) U 12 e . (1)
dx
from the given boundary condition. Note that the Laplace transformation has
transformed the partial differential equation into an ordinary differential equation (1).
To solve equation (1) multiply both sides by the integrating factor e (2s 1)dx =
= e(2s 1)x . Then equation (1) can be written in the form
d [U e(2s 1)x ] = 12 e(2s 4)x
dx
Integrating both sides we get
e(2s 4)x
U e(2s 1)x = 6 + C.
s2

191
Advanced Engineering Mathematics Chapter 7- Partial Differential Equations

e3x
or U=6 + C e(2s 1)x .
s2
Now since u(x, t) must be bounded as x, we must have U(x, s) also
e3x
bounded as x and it follows that we must choose C = 0. Then U = 6 , and
s2
2t3x
so, on taking the inverse, we find u(x, t) = 6 e .
This is easily checked as the required solution.
Example 7.12:
Solve by Laplace transforms the boundary-value problem
u 2 u
= , u (x, 0) = 3 sin 2x,
t x 2
u (0, t) = u (1, t) = 0, where 0 < x < 1, t > 0.
Solution:
Taking the Laplace transform of the partial differential equation, we find
d2 U d2U
s U u (x, 0) = or s U = 3 sin 2x.
dx 2 dx 2

The general solution of this equation is


U = A e s x + B e s x + 3 sin 2x.
s 4 2
Taking the Laplace transform of those boundary conditions, we have
L{u (0 , t)} = U(0 , s) = 0 and L{u(1, t)} U(1 , s) = 0.

Using the first condition U (0, s) = 0, we have A + B = 0.


Using the second condition U (l, s) = 0, we have
A e s + B e s = 0. Thus A = 0, B = 0 and U = 3 sin 2x,
s 4 2
from which we obtain on inversion
2
u(x, t) = 3 e4 t sin 2x. (1)
This problem has an interesting physical interpretation. If we consider a solid
bounded by the infinite plane face x = 0 and x = 1, the equation

192
Advanced Engineering Mathematics Chapter 7- Partial Differential Equations

u 2u
=k ,
t x 2

is the equation for heat conduction in this solid where u(x , t) is the temperature at
any plane face x at any time t and k is a constant called the diffusivity, which depends
on the material of the solid. The boundary conditions u(0 , t) = 0 and u(1 , t) = 0,
indicate that the temperatures at x = 0 and x = 1 are kept at temperature zero, while
u(x , 0) = 3 sin 2x,
represents the initial temperature everywhere in 0 < x < 1. The result (1) then is the
temperature everywhere in the solid at time t > 0.

193
Advanced Engineering Mathematics Chapter 7- Partial Differential Equations

EXERCISES 7
1) Eliminate the arbitrary functions from the following equations:
2
a) z = f(x + ay), b) z = f(x + iy) + g(x iy), where i = 1,
c) z = f(x cos + y sin at) + g(x cos + y sin + at),
2 2 ax+by n y
d) z = f(x y ), e) z = e f(ax + by), f) z = x f( ).
x
z z 2z 2z
[Answers: a) =a , b) + = 0,
y x 2 2
x y
2z 2z 1 2z z z
c) + = 2 , d) y + x = 0,
2 2 2 x y
x y a t
z z z z
e) b + a = 2 a b z, f) x + y = n z.]
x y x y
2) Eliminate the arbitrary constants from the following equations:
b) z = A ept cos qx sin ry, (where p = q + r )
2 2 2 2
a) z = A ep t cos ps ,
2 2
c) z = ax + (1 a)y + b, d) z = ax + by + a + b ,
2 2 2
e) a z + b = a x + y, f) z = (x a) + (y b) .
z 2 z 2z 2z 2z
[Answers: a) = , b) + + = 0,
t x 2 x 2 y 2 t 2
z z 2
c) + = 1, z z z 2 z
x y d) z = x + y + + ,
x y x y
z z 2 z 2
e) = 1, z
x y f) 4z = + .]
x y
3) Show that u = f (y 3x), where f is an arbitrary differentiable function, is a general
u u
solution of the equation +3 = 0. Find the particular solution which satisfies the
x y
condition u(0, y) = 4 sin y. [Answer: u(x, y) = 4 sin (y 3x).]
4) Show that u(x , y) = f(2x + 5y) + g(2x 5y) is a general solution of
2u 2u
4 = 25 . Find a particular solution satisfying the conditions
y 2 x 2
u
u(0 , y) = u( , y) = 0, u(x , 0) = sin 2x, | = 0. [Answer: u(x, y) = sin 2x cos 5y.]
t t=0

194
Advanced Engineering Mathematics Chapter 7- Partial Differential Equations

5) Determine whether each of the following partial differential equation is linear or


nonlinear, state the order of each equation
2u 2u 2u 2
4
2 u 2u 2u
a) +4 + = 0, b) (x + y ) = + ,
x 2 x y y 2 t 4 x 2 y 2

u 3u 2u 2u 2
c) u = , d) 4 =x ,
x y3 t 2 x 2
u u
e) + = 12 .
x y u
[Answers: a) Linear of order 2, b) Linear of order 4, c) Nonlinear of order 3,
d) Linear of order 2, e) Nonlinear of order one.]
6) Classify each of the following equations as elliptic, hyperbolic or parabolic:
u 2u 2u 2u 2u
a) + = 5, b) 2 +2 = x + 3 y,
x xy x 2 x y y 2

2u 2u 2
2 u 2u 2
2 u
c) = , d) x +2xy +y = 0,
x 2 y 2 x 2 x y y 2

2 2u 2u 2 2u
e) (x 1) +2xy + (y 1) = 0.
x 2 x y y 2

[Answers: a) Hyperbolic, b) Elliptic, c) Hyperbolic, d) Parabolic,


2 2 2 2 2 2
e) Elliptic if x + y < l, Hyperbolic if x + y > l, Parabolic if x + y = l.]
7) Show that u(x, y) = x f (2x+y) is a general solution of
u u 2
x 2x = u. Find the particular solution satisfying u (1, y) = y .
x y
2
[Answer: u (x, y) = x (2x + y 2) .]
3x
8) Show that u(x, y)= 4 e cos 3y is a solution to the boundary value problem
2u 2u 3x
+ = 0, u(x, /2) = 0, u(x, 0) = 4 e .
x 2 y 2
9) Find a partial differential equation having general solution
2u 2u 2u
u = f (x 3y) + g (2x + y). [Answer: 3 5 2 = 0.]
x 2 x y y 2

10) Find a partial differential equation having general solution

195
Advanced Engineering Mathematics Chapter 7- Partial Differential Equations
y
a) u = e f(2x 3y), b) u = f(x 2y) + g(2x + y).
u u 2u 2u 2u
[Answers: a) 3 + 2 = 2u, b) 2 3 2 = 0.]
x y x 2 x y y 2

2 u u
11) a) Solve x + = 0.
xy y
5 4
b) Find the particular solution for which u(x, 0) = x + x 34, u (2, y) = 3y .
6 2 4
[Answers: a) x u = f (x) + g (y), b) x u = x + x + 6y 34 x.]
12) Find general solutions of each of the following:
u u 2u 2u
a) + 2 = 3u, b) + = 0,
x y 2 2
x y
2u 2u 2u 2u 2u
c) = , d) 2 + = 0,
x 2 y 2 x 2 x y y 2

2u 2u 2u
e) 2 3 = 0.
x 2 x y y 2
3x
[Answers: a) u = e f (y 2x), b) u = f (x + iy) + g(x iy),
c) u = f(x + y) + g(x y), d) u = f(x + y) + x g(x + y)
e) u = f(3x + y) + g(x y).]
13) Find general solutions of each of the following:
2u 2u 2 2u 2u 2u
a) = + 12 t , b) 3 +2 = x sin y,
x 2 t 2 x 2 x y y 2
u u 4u 4u
c) + 2 = x, d) +2 = 4,
x y 4 3
x x y
4u 4u 4u
e) +2 + = 16.
x 4 2
x y 2 y 4
4
[Answers: a) u = f (x t) + g(x + t) t ,
b) u = f (x + y) + g(2x + t) 1 x sin y + 3 cos y,
2 4
2
c) u = f(y 2x) + 1 x ,
2
2 3
d) u = f(y) + x g(y) + x h(y) + T(y2x) + 1 x y,
3
2 2 2
e) u = f(x + iy) + g(x iy) + x h(x + iy) + x T(x iy) + 4(x +y ) .]
196
Advanced Engineering Mathematics Chapter 7- Partial Differential Equations

14) Solve each of the following boundary-value problems by the method of


separation of variables.
u u x
a) 3 + 2 = 0, u (x, 0) = 4 e ,
x y
u u 5x 3x
b) =2 + u, u (x, 0) = 3 e + 2 e ,
x y
u 2u
c) =4 , u (0, t) = 0, u (, t) = 0, u (x, 0) = 2 sin 3x 4sin 5x.
t x 2
2
u u u
d) = , |x=0 = u (2, t) = 0, u (x, 0) = 8 cos 3x 6 cos 9x .
t x 2 x 4 4
(3y 2x)/2 (5x3y) (3x 2y)
[Answers: a) u = 4 e , b) u = 3 e +2e ,
36t 100t
c) u = 2 e sin 3x 4 e sin 5x,
2 2
cos 3x 6 e cos 9x .]
9 t/16 81 t/16
d) u = 8 e
4 4
15) Solve the boundary-value problem by the method of separation of variable
2u 2u u
=4 , u (0, t) = u (5, t) = 0, u (x, 0) = 0, |t=0= f(x) if
t 2 x 2 t
a) f (x) = 5 sin x, b) f (x) = 3 sin 2x 2 sin 5x.
[Answers: a) u = 5 sin x sin 4t,
2
b) u = 3 sin 2x sin 4t 1 sin 5x sin 10t.]
4 5
16) Solve the boundary-value problem by the Fourier series
u 2u
=2 , u (0, t) = u (4, t) = 0, u (x, 0) = 25x, where 0 < x < 4, t > 0.
t x 2
2 2
em t /8
[Answer: u (x, t) = 200 cos m sin 1 mx.]
m 4
m 1
17) Show that the solution of the boundary-value problem
u 2 u u u
= , |x=0 = | , u(x, 0) = f (x), where 0 < x < , is given by
t x 2 x x x=
2
u (x, t) = 1 a0 + a m em t cos mx ,
2
m 1

197
Advanced Engineering Mathematics Chapter 7- Partial Differential Equations

2
am = f (x) cos m x dx, m = 0, 1, 2, 3, .

0
18) Solve each of the following boundary value problems by using Laplace
transforms.
u u 2x
a) = 3 , u (x, 0) = 4 e ,
t x
u u 2x 4x
b) = 2u, u (x, 0) = 10 e 6 e ,
t x
u 2 u
c) = , u (0, t) = u (4, t) = 0, u (x, 0) = 6 sin 1 x + 3 sinx,
t x 2 2

u 2 u u u
d) = , |x=0 = | , u (x, 0) = 4 sin x 2 sin 3x.
t x 2 x x x=2
2x6t x3t 4x6t
[Answers: a) u = 4 e , b) u = 10 e +6e ,
2 2 2 2
t/4 t t 9 t
c) u = 6 e sin 1 x + 3 e sin x, d) u = 4 e cos x 2 e cos 3x.]
2
19) Solve each of the following boundary value problems by using Laplace
transforms
u u u
a) = , |x=0 = u (2, t) = 0, u (x, 0) = 8 cos 3x 6 cos 9x ,
t x x 4 4
2u 2u u
b) =4 , u (0, t) = u (5, t) = 0, u (x, 0) = 0, |t=0= 5 sin x,
t 2 x 2 t
u 2
u
c) =4 , u (0, t) = u (3, t) = 0, u (x, 0) = 10 sin 2x 6 sin4x.
t x 2
2 2
cos 3x 6 e cos 9x ,
9 t/16 81 t/16
[Answers: a) u = 8 e
4 4
2 2
16 t 64 t
b) u = 5 sin x sin 2t, c) u = 10 e sin 2x 6 e sin 4x.]
2
20) Solve each of the following boundary value problems by using Laplace
transforms
u 2u
a) =2 , u (0, t) = u (5, t) = 0, u (x, 0) = 10 sin 4x 5 sin 6x,
t x 2

2u 2u u
b) =9 , u (0, t) = u (2, t) = 0, | = 0, u (x, 0) = 20 sin 2x 10 sin 5x,
t 2 x 2 t t=0

198
Advanced Engineering Mathematics Chapter 7- Partial Differential Equations

u 2 u
c) = 4u, u (0, t) = u (, t) = 0, u (x, 0) = 6 sin x 4 sin 2x,
t x 2
2u 2 u u
d) = 16 , |x=0= 0, u (3, t) = u (x, 0) = 0,
t 2 x 2 x
u
| = 12 cos x + 16 cos 3x 8 cos 5x,
t t=0
u 2u u
e) =3 , u (/2, t) = 0, |x=0= 0, u(x, 0) = 30 cos 5x,
t x 2 x
u 2u u
f) =3 , u (/2, t) = 0, |x=0= 0,
t x 2 x
u(x, 0) = 20 cos 3x 5 cos 9x.
2 2
32 t 72 t
[Answers: a) u = 10 e sin 4x 5 e sin 6x,
5t 8t
b) u = 20 sin 2x cos 6t 10 sin 5x cos 15t, c) u = 6 e sin x 4 e sin 2x,
d) u = 3 cos x sin 4t + 4 cos 3x sin 12t 2 cos 5x sin 20t,
3 5
75t 27t 243t
e) u = 30 e cos 5x, f) u = 20 e cos 3x 5 e cos 9x.]
u 2u
21) Solve =2 , 0 < x < 3, t > 0, given that u (x, 0) = u (3, t) = 0,
t x 2

u (x, 0) = 54 sin 3x + 2 sin 5x 2 sin 7x, |u (x, t)| < M, where the last condition
states that u is bounded for 0 < x < 3, t > 0.

199
Advanced Engineering Mathematics Chapter 7- Partial Differential Equations

Some Laplace Transforms


N f(t) F(s) = L {f(t)}

1
1 1
s
1
2 t
s2

n n!
3 t , (n = 0,1, 2, 3, )
s n 1

1
4
t s


5 t
s3

p (p 1)
6 t (p > 1)
s p 1

t 1
7 e
s

t
1
8 te
(s )2

2 t
2
9 t e
(s )3

n t
n!
10 t e (n = 1, 2, 3, )
(s )n 1

k t
(k 1)
11 t e , (k > 0)
(s )k 1

at bt ab
12 e e
(s a)(s b)

200
Advanced Engineering Mathematics Chapter 7- Partial Differential Equations

N f(t) F(s) = L{f(t)}

13 aeat bebt s
ab (s a)(s b)

b
14 sin b t
s2 b2
s
15 cos b t
s2 b2

16 sinh t
s2 2
s
17 cosh t
s2 2

at
18 e sin t
(s a)2 2

at sa
19 e cos t
(s a)2 2

201
Advanced Engineering Mathematics Chapter 7- Partial Differential Equations

Some Inverse Laplace Transforms


N F(s) = L {f(t)} f(t)

1
1
1 s
1
t
2 s2
1 t (n 1)
, (n = 1, 2, 3, )
3 s n (n 1)!

1 1
4 s t

1
t
5 s3
1 t (k 1)
, (k > 0)
6 s k (k)

1 t
7 e
s
1 t
te
8 (s )2

1 1 t2 et
9 (s )3 2

1 t (n 1) et
, (n = 1, 2, 3, )
10 (s )n (n 1)!

1 t (k 1) et
, (k > 0)
11 (s )k (k)

1 eat ebt
12 (s a)(s b) ab

202
Advanced Engineering Mathematics Chapter 7- Partial Differential Equations

N F(s) = L{f(t)} f(t)

13
s aeat bebt
(s a)(s b) ab
1 sin bt
14
s2 b2 b

s
15 cos bt
s2 b2
1 sinh t
16
s2 2

s
17 cosh t
s2 2
1 1 at
18 e sin t
(s a)2 2

sa at
19 e cos t
(s a)2 2

203
Advanced Engineering Mathematics Chapter 8 - Solution Of Differential Equations In Series

CHAPTER 8
SOLUTION OF DIFFERENTIAL EQUATIONS IN SERIES
2
Equation as simple as y' = 1 + x y and y" = x y
can not be solved by finite combinations of algebraic, exponential and trigonometric
functions, and many of the equations which occur in the mathematical expression of
natural phenomena can not be reduced to any of the soluble forms. We are thus led to
investigate solutions, which are expressible, by infinite processes, for example, as the
sum of an infinite series of elementary functions. A type of infinite series, which
suggests itself, is a power series in x,

y = an xn .
n 0
8.1 Series solutions near an ordinary point:
Let us consider a linear differential equation of the second order
(x) y" + (x) y' + (x) y = 0, (8.1)
where (x), (x) and (x) are polynomials in x. The point xo is said to be ordinary
point of the differential equation (8.1) if (xo) 0, otherwise a singular point. The
differential equation (8.1) can be written in the form
y"(x) = p(x) y' + q(x) y. (8.2)
Evidently p(x), q(x) can be expanded in power series of (xxo); which converge for
all x in a certain range ( p(x), q(x) analytic at x = xo ).
It can be shown that if xo is an ordinary point there exist a solution of equation (8.1)
of the form y = A(x) S(x) + B(x) T(x), where both S(x) and T(x) can be expressed in
power series in ( x xo). The radius of convergence of these power series is at least
as large as the minimum of the radii of convergence of the series for (x) and (x).
This type of equations (8.1) includes many equations of very great
importance, which cannot be solved in terms of simple combinations of elementary
functions. The usual procedure is to express the solution in the form of an infinite
series from which tables of the value of the solution may, if desired, be computed.
Thus the convergence of the series is important, not only as a basis of the validity of
the process, but also as an indication of the practical value of the result, for a slowly

204
Advanced Engineering Mathematics Chapter 8 - Solution Of Differential Equations In Series

converging series is of little use to a computer.


8.2 Solution developed as a Taylor series:
Let us consider the differential equation given in (8.2) and assume that the
coefficients p(x), q(x) are one-valued, and have derivatives of all order except
possibly for certain isolated values of x. Let x = a be a value for which p(x), q(x) and
all derivatives are finite: we shall obtain a solution such that y and y' have assigned
finite values yo , y'o when x = a.
By subsisting in (8.2) we obtain the corresponding value of the second derivative,
namely y "o = p(a) y 'o + q(a) yo . Differentiating (8.2), we obtain
y'" = p(x) y" + [p'(x) + q(x)]y' + q'(x) y,
in addition, as yo, y 'o , y "o are known, y "'o can be obtained immediately. Continuing
the process, we obtain the values of successive derivatives for x = 0, and thus we
have the coefficients in the Taylor series
(x a) 2 n
) (x a) +
y = yo + y 'o (x a) + y "o + + y (n
o
2! n!

Example 8.1:
To find the solution of y" x y = 0 such that y = A, y' = B when x = 0.
We have
iv
y" = xy, y"' = x y' + y, y = xy"+ 2y', and in general
(n) (n2) (n3)
y = xy + (n2) y .
The origin is an ordinary point; putting x = 0 in the above, we have
y "o = 0 , y "'o = yo , y iv
o = 2y o ,
' y ov = 3y "o ,
(n 3)
and in general y(n)
o = (n 2) yo , we thus find that
1) 2)
y(3n)
o = (1)(4)(7) (3n 2)A, y(3n
o = (2)(5)(8)(3nl)B, y(3n
o = 0.
Thus the required solution is
x 3 4x 6 28x 9 2x 4 10x 7 80x10
y = A[1 ] + B[x ]
3! 6! 9! 4! 7! 10!
The ratio test proves the convergence of each series separately satisfies the
differential equation. Since A and B may be regarded as ordinary constants, the above
is an expression of the general solution.

205
Advanced Engineering Mathematics Chapter 8 - Solution Of Differential Equations In Series

When p(x), q(x) are polynomials of low degree, a more practical way of
obtaining the solution is to assume a series of ascending powers of x

y = ao + a1 x + a2 x + = a n x n ,
2

n 0
to substitute this series in the differential equation and to determine the coefficients
so that the equation is identically satisfied; that is to say, so that the coefficient of
each individual power of x cancels out.

y' = a1 + 2a2 x + 3a3 x + 4a4 x + = na n x n 1 ,
2 3
i.e.
n 1

y" = 2a2 + (3)(2) a3 x + (4)(3) a4 x + = n(n 1)a n x n 2 ,
2

n 2
Substituting in the given differential equation, we get

2a2 + (n 2)(n 1)a n 2 a n 1 ]x n = 0.
n 1
n
By equating to zero the coefficient of x . Therefore a2 = 0 and an+2 =
a n 1
. This equation, known as a recurrence equation.
(n 1)(n 2)
1 1
Since yo = A , y 'o = B , then a1= B, ao = A and a3 = A , a4 = B,
(2)(3) (3)(4)
1 1 4A 1 (2)(5)B
a5 = a 2 = 0 , a6 = a3 = , a7 = a4 = , a8 = 0 and so on.
(4)(5) (5)(6) 6! (6)(7) 7!
1 4 28 1 2 10 80 10
Then y = A[1 x 3 x 6 x 9 ] + B[ x x 4 x 7 x ] .
3! 6! 9! 1! 4! 7! 10!

Example 8.2:
Find the solution in power series of the following differential equation
y" x y' + m y = 0.
Solution:
It is clear that; x = 0 is an ordinary point of this differential equation.

Let y = a n x n , y' = na n x n 1 , y" = n(n 1)a n x n 2
n 0 n 1 n 2
Substituting we get

206
Advanced Engineering Mathematics Chapter 8 - Solution Of Differential Equations In Series

n(n 1)a n x n 2 na n x n + m a n x n = 0,
n 2 n 1 n 0

hence (n 2)(n 1)a n 2 x n + (m n)a n x n = 0.
n 0 n 0

i.e. (n 2)(n 1)a n 2 (m n)a n x n = 0.
n 0
Equating to zero the coefficients of successive power of x, we obtain the following
recurrence relation:
(n+2)(n+1) an+2 + (m n) an = 0, n = 0, l, 2, 3, .
(n m)
i.e. an+2 = an .
(n 2)(n 1)
We see that the coefficients of even suffix depend on ao but not on a1; those of odd
suffix on a1 but not on ao, and that ao, a1, are arbitrary. Denoting by S(x) the series
obtained by taking ao = 0, a1 = 1, we have
m(m 2) 4 m(m 2)(m 4) 6
S(x) = 1 m x x ,
2! 4! 6!
(m 1)(m 3) 4 (m 1)(m 3)(m 5) 7
T(x) = x m 1 x x
3! 5! 7!
a nm
Since n 2 0 as n.
an (n 2)(n 1)
For any finite value of m, both series converges for all values of x. The
general solution is y = A S(x) + B T(x), where A, B are arbitrary constants. It will be
noted that when m is an even positive integer the series, S(x) terminates and reduces
to a polynomial of degree m; where m is odd, T(x) is a polynomial of degree m.
Note: It is sometimes possible, by a change of dependent variable, to simplify a
linear equation, or to identify it with one previously studied. The transformation most
frequently employed is of the form y = u v, where y is the original, v the new
dependent variable, and u a chosen function of x.
1 1
For example, if the equation y" + ( m + x 2 )y = 0, is satisfied by a series
2 4
of ascending integral powers of x as in Example 8.2, the recurrence relation between
the coefficients is (n+1)(n+2) an+2 + (m + ) an an2 = 0. This is a three-term
recurrence relation, which is much more difficult to manipulation than one that
207
Advanced Engineering Mathematics Chapter 8 - Solution Of Differential Equations In Series

1 x2
involves only two terms. If, however, we make the substitution y = e 4 v,
1 x2 2
so that y" = e 4 [ v" x v' + ( x ) v],
we find that the equation in v is v" x v' + m v = 0, which is precisely the equation
considered above.
8.3 Regular Singularity:
Let us consider an equation of the form
(x) y" + (x) y' + (x) y = 0 (8.1)
If (x a) is a factor of the coefficient (x), but not of both (x) and (x), the equation
fails to determine a value of y" for x = a, and the procedure of (8.2) is inapplicable.
We then say that x = a is a singular point or singularity of the equation.
The form of a solution valid in the neighbourhood of a singularity may
therefore differ from that of a solution appropriate to an ordinary point. It is important
to investigate this difference.
If x = a is a singular point, we can rewrite equation (8.1) in the form
2
(x a) y" + (x a) p(x) y' + q(x) y = 0 (8.3)
The point x = a is called a regular singular point of the differential equation (8.3), if
p(x) and q(x) can be expressed in convergent power series of (x a) (by means of
Taylor series of (x a) )
If x = a is a regular singularity of the differential equation (8.3), then there
always exist at least a solution of the form

Cr (x a)r , Co 0.
m
y = ( x a)
r 1
This solution converges in the common range of convergence of p(x) and q(x).
The substitution x a = Z shift the singularity to the origin and so there is no loss in
generality if we merely consider the equation
2
x y" + x p(x) y' + q(x) y = 0 , (8.4)

where p(x) = pr x r , q(x) = q r x r ,
r 0 r 0
and there is a regular singular point at the origin.

208
Advanced Engineering Mathematics Chapter 8 - Solution Of Differential Equations In Series

8.4 The method of Frobenius:


F. G. Frobenius, of Berlin (1873) assume as a trial solution

ar xr ,
C
y=x (8.5)
r 1
where the a's are constants. The index C will be determined by a quadratic equation
called the Indicial Equation. The roots of this equation may be equal, different and
differing by an integer, or different and differing by a quantity not an integer. These
cases will have to be discussed separately.
It is clear that,

y' = (C r)a r x (C r 1) ,
r 0

y" = (C r)(C r 1)a r x (C r 2) . (8.6)
r 0
Substituting in (8.4) we get

(C r)(C r 1)a r x (C r) + p r x r (C r)a r x (C r) +
r 0 r 0 r 0

+ q r x r a r x (C r) =0
r 0 r 1
C
By equating to zero the coefficients of x we have,
[C(C l) + po C + qo] ao = 0 , as ao 0
i.e. C(C l) + Po C + qo = 0 (8.7)
Equation (8.7) is called the Indicial Equation. It is quadratic in C. It has two roots C1,
C2. We have four cases depending on C1, C2. Equating to zero the coefficients of
C+1 C+2
x ,x , , we have
[C(C + l) + po(C + l) + qo] a1 + (p1 C + q1) ao = 0
[(C + S)(C + S 1) + po(C + S) + qo ] aS +
+ terms which are linear in aS1, aS2, ao = 0 (8.8)
Corresponding to a root C of the Indicial Equation, we can obtain the successive
coefficients a1, a2, , etc. in terms of ao. We always obtain two solutions of (8.4) in
the form (8.5) when the roots of I.E. are complex or are real but do not differ by an
integer. There is clearly only one solution of the type (8.5) when the roots of the I.E.
209
Advanced Engineering Mathematics Chapter 8 - Solution Of Differential Equations In Series

coincide. When the roots of the I.E. differ by the integer m the larger root yield the
desired solution. It may also happen that for the smaller root, the left-hand side of
equation (8.8) vanishes identically. In this case we shall see that a second solution of
the type (8.5) exists.
Case I. Roots of I.E. unequal and differing by a quantity not an integer:
Example 8.3:
Consider the equation 2 x y" + (x + l) y' + 3y = 0

a r x = a r x (C r) , ao 0,
r
C
put y=x giving
r 1 r 1

y' = (C r)a r x (C r 1) , y" = (C r)(C r 1)a r x (C r 2) .
r 0 r 0
Substituting we get

2 (C r)(C r 1)a r x (C r 1) + (C r)a r x (C r) + (C r)a r x (C r 1) +
r 0 r 0 r 0

+ 3 a r x (C r) = 0.
r 1

(C r 1)
2(C r)(C r 1) (C r) a r x + (C r 3)a r x (C r) = 0.
r 0 r 0

i.e. (C r)(2C 2r 1) a r x (C r 1) + (C r 3)a r x (C r) = 0.
r 0 r 0
put r l = m in the first summation we get

+ (C m 1)(2C 2m 1) a m 1x (C m) +
(C 1)
C(2C 1) ao x
m 0

+ (C r 3)a r x (C r) = 0.
r 0
(C 1)
i.e. C(2C 1) ao x +

(C m 1)(2C 2m 1)a m 1 (C m 3)a m x (C m) = 0.
m 0
C1 C C+1
By equating to zero the coefficients of x ,x ,x , , we have
C(2C 1) ao = 0 I.E. (Indicial Equation)
(C + m l)(2C + 2m + l) am+1 + (C + m +3) am = 0

210
Advanced Engineering Mathematics Chapter 8 - Solution Of Differential Equations In Series

1
C = 0 or C = .
2
Thus, if C = 0,
(m + l)(2m + 1) am+1 + (m + 3) am = 0, m = 0, l, 2, 3,
(m 3)
i.e. am+1 = am
(m 1)(2m 1)
Hence a1 = 3ao , a2 = 2ao, a3 = 2 ao , a4 = 1 ao , a5 = 1 ao and so on, replacing ao
3 7 45
by A we get:
2 3 4 5
y1 = A(l 3x + 2x 2 x + 1 x 1 x + ).
3 7 45
1
If C = , (m + 3 )(2m + 2) am+1 + (m + 7 )am = 0.
2 2 2
(2m 7)
i.e. am+1 = am .
(2m 3)(2m 2)
Hence a1 = 7 ao , a2 = 21 ao, a3 = 11 ao , a4 = 1113 ao = 143 ao,
6 40 80 8980 5760
a5 = 13 a = 13 a and so on, replacing ao by B we get:
6880 o 3840 o
2 3 4 5
y2 = B x [1 7 x + 21 x 11 x + 143 x 13 x +].
6 40 80 5760 3840
Thus y = A y1 + B y2 is a solution which contains two arbitrary constants, and so may
be considered the complete primitive.
m+C
In the series we obtained Um = am x
U m 1 a m 1 (C m 3)
Then = x= x.
Um am (C m 1)(2C 2m 1)
and the limit when m is zero, independent of the value of x. Hence both series
obtained are convergent for all values of x.
Case II. Roots of Indicial Equation equal:
Consider the equation (8.4) and its trial solution (8.5). As before equating to
C+1 C+2
zero all coefficients of x , x , , but do not yet put the coefficient of x equal to
zero. Insert in the series (8.5) the values of the coefficients ar in terms of C, we have

y = ao x C f r (C)x r = ao Y , fo = 1, (8.9)
r 0
Substituting in equation (8.4), we get
2 2 C
[x D + x p(x) D + q(x)] Y = ao x [C(C l) + po C + qo] (8.10)

211
Advanced Engineering Mathematics Chapter 8 - Solution Of Differential Equations In Series

Differentiate each side with respect to C. The order of differentiation with respect to
x and C may be interchanged and so
2 2
[x D + x p(x) D + q(x)]
Y
C
=

C

a o x C [C(C 1) po C q o ] =

= a o x C [C(C 1) po C q o ] n x + a o x C [2C po 1] . (8.11)

Now choose C(C l) + po C + qo = 0 ,


if the roots of the indicial equation are equal, then the sum of the two roots
C1+ C2 = 2C = 1 po . It is clear that the right-hand side of equation (8.11) is zero,
Y
for C1 = C2 = C, and so Y and are solutions of our differential equation
C C C 1

Since Y = x C f r (C)x r , fo = 1,
r 0
Y f (C)
C1 C1
then = x ( nx) r 1
f (C )x r
+ x r xr . (8.12)
C C C1 r 0 r 0 C C1
f (C)
Note that fr(C) are rational functions, and so r is best calculated by
C C1

logarithmic differentiation.
In general, if the Indicial Equation has two equal roots, we get two
Y
independent solutions by substituting this value of C in Y and . The second
C
solution will always consist of the product of the first solution (or a numerical
multiple of it) and n x, added to another series.

Example 8.4:
Integrate in series the equation x y" + y' + x y = 0.
Solution:

a r x = a r x (C r) .
r
C
Let y = x
r 1 r 1

Then y' = (C r)a r x (C r 1) , y" = (C r)(C r 1)a r x (C r 2) .
r 0 r 0
Substitute in the differential equation we obtain

212
Advanced Engineering Mathematics Chapter 8 - Solution Of Differential Equations In Series

(C r)(C r 1) (C r) a r x (C r 1) + a r x (C r 2) = 0.
r 0 r 0

(C r)2 a r x (C r 1) + a r x (C r 2) = 0.
r 0 r 0

+ (C r) 2 a r x (C r 1) +
2 C1 2 C 2 C+1
i.e. C ao x + (C + 1) a1 x + (C + 2) a2 x
r 3

+ a r x (C r 2) = 0.
r 0
2 C1 2 C 2 C+1
Thus C ao x + (C + 1) a1 x + (C + 2) a2 x +

+ (C r 3)2 a r 3 a r x (C r 2) = 0.
r 0

Equate coefficients of successive powers of x to zero, we get:


2
1) The Indicial Equations C = 0.
2
2) (C + l) a1 = 0, i.e. a1 = 0
2
3) (C + 2) a2 = 0, i.e. a2 = 0
2 1
4) (C + r + 3) ar+3 + ar = 0 i.e. ar+3 = ar
(C r 3)2
1
Therefore a3 = a , a4 = 0 , a5 = 0,
2 o
(C 3)
1 1
a6 = a3 = a , a7 = 0 , a8 = 0,
2 2 2 o
(C 6) (C 6) (C 3)
1 1
a9 = a6 = ao , a10 = 0 , a11 = 0.
(C 9) 2 (C 9) 2 (C 6)2 (C 3)2
Hence
x3 x6 x9
y = xC 1 , (1)
2
(C 3) (C 3)2 (C 6)2 (C 3)2 (C 6)2 (C 9)2
is a solution if C = 0
x3 x6 x9
i.e. y = [1 ] , is a solution.
2 2 2 2 2 2
3 3 .6 3 .6 .9
Differentiating (1) with respect to C we get
C 2x
3
Y 2x 6 2x 6
= Y n x + x +
C (C 3)
2
(C 3)3 (C 6) 2 (C 3) 2 (C 6)3

213
Advanced Engineering Mathematics Chapter 8 - Solution Of Differential Equations In Series

2x9 2x9 2x9


+
3 2 2 2 3 2 2 2 3
(C 3) (C 6) (C 9) (C 3) (C 6) (C 9) (C 3) (C 6) (C 9)
Y 2x 3 18x 6 198x 9
and = y n x + .
C C 0 3
3 3 3
3 .6 3 3 3
3 .6 .9
Y
Thus the complete solution a y + b =
C C 0
x3 x6 x9 2x 3 18x 6 198x 9
[a + b n x] 1 2 2 2 2 2 2 + b 3 3 3 3 3 3 .
3 3 .6 3 .6 .9 3 3 .6 3 .6 .9
Y x3 x6 x9
i.e. a y + b = [a + b n x] 1
2

4 2

6 2
+
C C 0 3 3 .(2!) 3 (3!)
x3 x6 x9
+ 2b 1
(1 ) (1 1 1 ) .
2
3 34 (2!) 2 2
37 (3!)2 2 3

The series are easily proved to be convergent for all values of x.
Case III: Roots of Indicial Equation differing by an integer,
making a coefficient of Y infinite:
Let the roots of differential equation are C1 and C1 + n (where n is positive integer),
and let some of the coefficients of Y become infinite when C = C1 (the smaller root).
(Y)C1 n is a solution. In general fr(C) have a factor (C C1) in the denominator.

Write
Y1 = ( C C1 ) Y. (8.13)
Then
2 2 C
[x D + x p(x) D + q(x)] Y1 = ao x (C C1) [C(C 1) po C q o ] (8.14)
Differentiate each side with respect to C. The order of differentiation with respect to
x and C may be interchanged and so
2 2 Y
[x D + x p(x) D + q(x)] 1 =
C

C

a o x C (C C1 )[C(C 1) po C q o ] =

= a o x C (C C1 )[C(C C1 ) po C q o ] n x +

+ a o x C [C(C 1) po C q o ] + a o x C (C C1 )[2C po 1] . (8.15)

Now, since C1(C11) + po C1 + qo = 0, then if we put C = C1, in Equation (8.15) we


get

214
Advanced Engineering Mathematics Chapter 8 - Solution Of Differential Equations In Series

2 2 Y
[x D + x p(x) D + q(x)] 1 =0
C C C1
Y1
i.e. C is a solution of the differential equation (8.4).
C C1

Thus we have found three solution to the differential equation (8.4) which of the
Y
second order. Possible solutions are (Y)C1 n , (Y1 )C1 , 1 .
C C C1

The second of these is a multiple of the first. In general, if the Indicial Equation has
two roots C1 and C1 + n differing by an integer n, and if some of the coefficients of Y
become infinite when C = C1, we modify the form of Y by multiplying it by (C C1).
We then get two independent solutions by putting C = C1, in the modified form of Y
Y1
(which is Y1) and . The result of putting C = C1 + n in Y merely gives a
C
numerical multiple of that obtained by putting C = C1.
Example 8.5:
Solve in series the following differential equation x y" 3 y' + x y = 0.
Solution:

a r x r = a r x (C r) .
C
Let y = x
r 1 r 1

(C r 1)
Then y' = (C r)a r x , y" = (C r)(C r 1)a r x (C r 2) .
r 0 r 0
Substituting we get

(C r)(C r 1) 3(C r) a r x (C r 1) + a r x (C r 1) = 0.
r 0 r 0

(C r)(C r 4)a r x (C r 1) + a r x (C r 1) = 0.
r 0 r 0
C1 C
i.e. C(C 4) ao x + (C + 1)(C3) a1 x +

+ (C r)(C r 4)a r x (C r 1) + a r x (C r 1) = 0.
r 3 r 0
C1 C
Thus C(C 4) ao x + (C + 1)(C3) a1 x +

+ (C r 2)(C r 2)a r 2 a r x (C r 1) = 0.
r 0

215
Advanced Engineering Mathematics Chapter 8 - Solution Of Differential Equations In Series

Equate coefficients of successive powers of x to zero we get :


1) The Indicial Equation C(C 4)ao = 0 i.e. C = 0 or C = 4.
2) (C + l)(C 3) a1 = 0 i.e. a1 = 0.
3) (C + r + 2)(C + r 2) ar+2 + ar = 0
i.e. ar+2 = 1 ar .
(C r 2)(C r 2)

Therefore 0 = a1 = a3 = a5 = a7 = a9 = = a2ml.
1 1
a2 = ao , a4 = ao ,
(C 2)(C 2) (C 4)C(C 2)(C 2)
1
a6 = ao ,
2
(C 6)(C 4)C(C 2) (C 2)
1
a8 = ao ,
2 2
(C 8)(C 6)(C 4) C(C 2) (C 2)
The roots of the Indicial equation are C = 0 or C = 4. But if we put C = 0, the
coefficients a4 , a6 , a8 , become infinite.
Now
Y1 = (C C1) Y = (C 0) Y = C Y =
C C 2 1 4
=x [C x + x
(C 2)(C 2) (C 4)(C 2)(C 2)
1 6 1 8
x + x ]
(C 6)(C 4)(C 2) 2 (C 2) (C 8)(C 6)(C 4) 2 (C 2)2 (C 2)
Putting C = 0 we get a solution
1 1 1 1
u= x4 + x6 x8 + x10 =
4.2.2 6.4.2.2.2 8.6.4.4.2.2.2 10.8.6.6.4.4.2.2.2
4 6 8 10
2x 2 x 2 x 2 x
= .
2! 2 3!.1! 2 4!.2! 2 5!.3! 2

But
Y1 C (C2 4)
= Y1 n x + x 1 x2
C [(C 2)(C 2)]
2

1 1 1 1 4
x +
(C 4)(C 2)(C 2) C 4 C 2 C 2

216
Advanced Engineering Mathematics Chapter 8 - Solution Of Differential Equations In Series

1 1 1 2 1 6
+
2 C 6 C 4 C 2 C 2 x

(C 6)(C 4)(C 2) (C 2)
1 1 1 2 2 1 8
x + .
(C 8)(C 6)(C 4) 2 (C 2)2 (C 2) C 8 C 6 C 4 C 2 C 2
Y
Putting C = 0 we get another solution 1 =v.
C C 0
1 1 1 1 1 6 1 1 1 2 2 1
v = u n x + 1 + x 2 [ ]x + [ ]x8
4 4.2.2 4 2 2 8.6.4.4.2.2.2 8 6 4 2 2
2 4 6
x 1 x 1 1 1 x
i.e. v = u n x + 1 + 1 +
2 2.2! 2 3!.1! 2 3 2
8 10
1 1 1 1 1 x 1 1 1 1 1 1 1 x
+ 1 1 +

4!.2! 2 3 4 2 2 5!.3! 2 3 4 5 2 3 2

4
Then the general solution is given by A u + B v. Putting C = 4 we get YC=4 = 2 u.
The series are easily proved to be convergent for all values of x, except at x = 0.
Case IV. Roots of Indicial Equation differing by an integer,
making a coefficient of Y indeterminate:
If the Indicial Equation has two roots C1, and C1 + n differing by an integer n, and if
one of the coefficients of Y becomes indeterminate when C = C1, the complete
primitive is given by putting C = C1 + n in Y, which then contains two arbitrary
constants. The result of putting C = C1 in Y merely gives a numerical multiple of one
of the series contained in the first solution.
Example 8.6:
With the help of power series, find the solutions of the equation
x y" + (x 1) y' y = 0 .
Solution:

a r x r = a r x (C r) .
C
Let y = x
r 1 r 1

Then y' = (C r)a r x (C r 1) , y" = (C r)(C r 1)a r x (C r 2) ,
r 0 r 0
and proceeding as usual, we get

(C r)(C r 2)a r x (C r 1) + (C r 1)a r x (C r) = 0.
r 0 r 0

217
Advanced Engineering Mathematics Chapter 8 - Solution Of Differential Equations In Series

C(C2) ao x
C1
+ a r (C r 1)a r 1 (C r 1)x (C r) = 0.
r 0
Then C(C2) ao = 0 i.e. C = 0 or C = 2 (1)
[ar + (C + r + l) ar+1 ] (C + r1) = 0
(2)
1
r = 0 : (ao + (C + 1)a1) (C l) = 0 i.e. a1 = ao (3)
C 1
r = l: [a1 + (C + 2) a2] C = 0, (4)
1
r = 2: [a2 + (C + 3) a3] (C + 1) = 0, i.e. a3 = a2 (5)
C3
1
r = 3: [a3 + (C + 4) a4] (C + 2) = 0, i.e. a4 = a3 (6)
C4
In general ar+1 = ar/ (C + r + 1) for all values of r, except at C + r +1 = 0.
Equation (1) gives C = 0 or C =2. The coefficient of [a1 + (C + 2) a2 ] in (4) vanishes
when C = 0, but as there is no other term in the equation this makes a2 indeterminate
instead of infinite.
If C = 2 , a2 = a1 = ao .
Thus, if C = 0, from equations (3), (5), (6) we get
1 2 1 1 2
a1 = ao , a3 = a2 = a2 , a4 = a3 = a2 = a2 ,
3 3! 4 3.4 4!
1 1 2 1 1 2 2
a5 = a4 = a2 = a2, a6 = a5 = a2 = a2 , a7 = a2 ,
5 3.4.5 5! 6 3.4.5.6 6! 7!
This gives us
2 2 2 2 2
(Y)C=0 = ao(1 x) + a2 x 2 x 3 x 4 x 5 x 6 x 7 =
3! 4! 5! 6! 7!
2 2 2 2 2 2
= ao(1 x) + a2 x 1 x x 2 x 3 x 4 x 5 .
3! 4! 5! 6! 7!
This contains two arbitrary constants, so it may be taken as the complete primitive. It
x
is clear that (Y)C=0 = (ao 2a2)(l x) + a2 e . But we have the other solution given
by C = 2.
1 1 2 1 2 2
a1 = ao , a2 = a1 = ao , a3 = a2 = ao , a2 ,
3 4 4! 5 5! 3!
1 2 1 2
a4 = a3 = ao , a5 = a4 = ao
6 6! 7 7!

218
Advanced Engineering Mathematics Chapter 8 - Solution Of Differential Equations In Series

and so on.
2 2 2 2 2 2
(Y)C=0 = ao x 1 x x 2 x 3 x 4 x 5
3! 4! 5! 6! 7!
x
= 2 ao e 2 ao (1 x) = 2 ao ex +2 ao (x 1) .
That is a constant multiple of the second series in the first solution.
Example 8.7:
2
Obtain the solutions of (1 x ) y" + 2 x y' + y = 0.
Solution:
Proceeding as usual, we get
C2 C1
C(C 1)ao x + (C + 1) C a1 x +

+ (C r 2)(C r 1)a r 2 2(C r) (C r)(C r 1) 1 a r x (C r) = 0.
r 0
Hence
C(C 1) ao = 0, (1)
(C + 1) C a1 = 0 , (2)
a2(C + 2)(C + l) + ao[2C C(C l) + l] = 0 , (3)
a3(C + 3)(C + 2) + a1[2(C + 1) (C + 1) + 1] = 0, (4)
and in general
(C + r + 2)(C + r l) ar+2 +[2(C + r) (C + r)(C + r l) + l] ar = 0 (5)
Equation (1) gives C = 0 or C = 1. The coefficient of a1 in (2) vanishes when C = 0,
but as there is no other term in the equation this makes a1 indeterminate instead of
infinite. If C = 1, a1 = 0.
Thus, if C = 0, from equations (3), (4), (5) we get
1 1 1 1 1
a2 = ao , a3 = a1 , a4 = a2 = ao , a5 = a1,
2 2 4 8 40
1 3
a6 = ao , a7 = a1 . This gives us
80 560
1 2 1 4 1 6 1 3 1 5 3 7
(Y)C=0 = ao [1 x + x + x + ] + a1 [ x x + x + x + ].
2 8 80 2 40 560
This contains two arbitrary constants, so it may be taken as the complete primitive.
The series may be proved convergent for |x| < l.

219
Advanced Engineering Mathematics Chapter 8 - Solution Of Differential Equations In Series

But we have the other solutions given by C = 1. Working out the coefficients,
1 2 1 4 3 6
(Y)C=1 = ao x[1 x + x + x + ],
2 40 560
that is, a constant multiple of the second series in the first solutions.
Some cases where the method fails:
If the differential equations is not in the form (8.4), where p(x) and q(x) are
finite when x = 0, then the equation is said to have no regular primitives (integrals) in
ascending powers of x.
Let us consider the following differential equation
4 3
x y" + 2 x y' y = 0.
If we try to apply the usual method, we get for the indicial equation, ao = 0, which has
no roots, as by hypothesis ao 0.
Such a differential equation is said to have no regular integrals in ascending powers
1/x 1/x
of x. It is easy to verify that e and e are solutions to the given differential
1/x 1/x
equation. Of course e and e can be expanded in powers of (1/x) .
2 1
It is easy to see that the equation x y" + 2 x y' y=0,
2
x
gives us ao = 0 , a1 = 0 and all other coefficients are also zero.
If we try to apply the usual method on the differential equation
2
x y" (l 3x) y' + y = 0.
We can see that no integrals that is regular in ascending powers of x, as the one series
obtainable diverges for all values of x.
4 2 3 2 3
The following equation x (1 x ) y" + 2x y' (l x ) y = 0 ,
has no integrals that are regular in either ascending or descending powers of x. This
(x + 1/x) (x + 1/x)
equation has the primitive : a e +be .

220
Advanced Engineering Mathematics Chapter 8 - Solution Of Differential Equations In Series

Exercises 8
Solve the following differential equations in series of powers of x :
2
1. y" + y' x y = 0 , y(0) = 2 , y'(0) = l.
1 2 5 3 1 4
[Answer: y = 2 + x x + x + x + ]
2 6 8
2
2. y' = y x ; y(0) = 1.
1 2 2 3 7 4
[Answer: y = 1 + x + x x x ].
2 3 12
3. y' = x + 1 ; y(0) = 1.
y
1 1
[Answer: y = 1 + x + x 3 x 4 ].
3 3
y
4. y' = y + x e ; y(0) = 0.
1 2 1 3 1 4
[Answer: y = x x x ].
2 6 6
8. y' = 2x + cos y ; y(0) = 0.
1 1
[Answer: y = x + x 2 x 3 x 4 ].
6 4
2 3
6. y' = x + y ; y(1) = 1.
2 3 4
[Answer: y = 1 + 2(x1) + 4(x1) + 25 (x1) + 81 (x1) +].
3 4
2
7. y" = x y y ; y(0) = 1, y'(0) = 2.
1 2 1 3 1 4
[Answer: y = 1 + 2x x x x ].
2 3 3
2 2
8. y" = y' + x y y ; y(0) = 4, y'(0) = 2.
2 3 4
[Answer: y = 4 2x + 2x 2x + 19 x + ].
6
2 2
9. y' = y + x ; y(0) = 2. [Answer: y = 2 2x x ].
2(x1)
10. y' = 2y + x 1 ; y(1) = A. [Answer: y = ( A + 1 ) e 1 x + 1 ].
4 2 4
2 3
11. y' = y + x ; y(0) = 1 .
2
2 3 4 5
[Answer: y = 1 + 1 x + 1 x + 1 x + 9 x + 21 x +].
2 4 8 16 32 320
2 2
12. y' = x y ; y(0) = 0.
[Answer: y = 1 x 1 x 7 2 x11 ].
3
3 7.9 7.11.27
13. (1 x) y' = 1 + x y ; y(0) = 0.

221
Advanced Engineering Mathematics Chapter 8 - Solution Of Differential Equations In Series

1
[Answer: y = x + 1 x 2 1 x 3 1 x 4 + x n +,
1.2 2.3 3.4 (n 1)n
the series converges for 1 x 1 ].
14. x y" + y = 0 ; y(0) = 0 , y'(0) = 1.
[Answer: y = x 1 x2 + 1 x3 + 1 x4+ ].
2
2.(1!) 3.(2!)2 4.(3!) 2

18. y" + x y = 0 ; y(0) = 1 , y'(0) = 0.


3 6 9
[Answer: y = 1 1 x + 1.4 x 1.4.7 x + ].
3! 6! 9!
16. y" + 2 y' + y = 0 ; y(0) = 1 , y'(0) = 0.
x
[Answer: y = sin x / x].
17. y" + 1 y' + y = 0 ; y(0) = 1 , y'(0) = 0.
x
[Answer: y = 1 12 x 2 1 x4
2 2
1
2 2 2
x 6 ].
2 2 .4 2 .4 .6
18. y" + y cos x = 0 ; y(0) = a , y'(0) = 0.
[Answer: y = a(1 1 x 2 2 x 4 9 x 6 55 x8 ].
2! 4! 6! 8!
2
19. (1 + x )y" + x y' y = 0 ; y(0) = a , y'(0) = 0.
[Answer: Recurrence relation (n+2) an+2 + (n 1) an = 0,
1.3.5. (2k 3) 2k
y = Ao x + A1[1 + 1 x + (1)k 1
2
x ].
2 k
k 2 2 k!
2
20. y" x y' y = 0 .
[Answer: Recurrence relation (n+2)(n+1) an+2 (n 1) an1 an = 0,
2 1 4 1 5 1 6 13 7
y = A(1 + 1 x + x x x x ) +
2 24 20 720 2520
1 1 1 5 7 6 41 7
+ B( x + x 3 x 4 x x x )].
6 12 120 360 5040
2 2
21. y" 2 x y' + 4x y = x + 2 x + 2.
[Answer: Recurrence relation (n+3)(n+2) an+3 2(n 2) an = 0, n 2,
2 2 2 9 1 1 1 10
y = A(1 x 3 x 6 x ) + B( x x 4 x 7 x ) +
3 45 405 6 63 567
1 1 1 1 7 1 9 1 10
+ ( x 2 x3 x 4 x 6 x x x )].
3 12 45 126 405 1134
2 2
22. 2 x y" x y' + ( x + 1 ) y = 0.
[Answer: Recurrence relation (C + n + 1)(2C + 2n + 3) an+2 + an = 0,

222
Advanced Engineering Mathematics Chapter 8 - Solution Of Differential Equations In Series

1 1 4 1
y = A x (1 x 2 x x 6 ) +
6 168 11088
1 2 1 4
+ B x( 1 x x 1 x 6 ) ].
10 360 28080
2
23. 3 x y" + 2 y' + x y = 0.
[Answer: Recurrence relation (C + n + 3)(3C + 3n +8) an+3 + an = 0,
1 1 1 3 1 6
y = A (1 x 3 x 6 ) + B x ( 1
1/3
x x ) ]
24 2448 30 3420
24. x y" + y' y = 0.
2
[Answer: Recurrence relation (C + n + 1) an+1 an = 0, n 0, C = 0 , 0 ;
1 1 3
y = (A + B n x) (1 + x x2 x )
2 2
(2!) (3!)
1 1 1 1 1
2 B( x (1 )x 2 (1 )x 3 ) ].
(2!)2 2 (3!)2 2 3
2
28. (x x ) y" 3y' + 2y = 0.
[Answer: Recurrence relation (C + n 3)an+1 (C + n 2) an = 0, C = 4 , 0 ;
2 4
y = A (x + 2 x + 3) + B x .]
(1 x)2
2
26. (x x) y" + 3y' 2y = x + 32 .
x
[Answer: see 25, Recurrence relation (C + n 3)an+1 (C + n 2) an = 0, C = 4 , 0;
2 4 2
y=A(x +2x+3)+B x + 1 x 3 .]
(1 x) 2 4 5x
2 3 2
27. 2(x + x ) y" (x 3x ) y' + y = 0
[Answer: Recurrence relation an = anl , C = 1 ,
2 3 4 5
y = (A x + B x)( l x + x x + x x +) , | x | < l )].
28. 4 x y" + 2(l x) y' y = 0
[Answer: Recurrence relation 2(c + n + 1) an+1 = an,
1 1 2 1 3
y = A (l x x x ) +
2.1! 2 3
2 .2! 2 .3!
1 1 2 1
+ B x (1 x x x 3 )
1.3 1.3.5 1.3.5.7
2 2
29. 2 x y" x y' + (1 x ) y = 0
[Answer: Recurrence relation (C + n + l)(2C + 2n + 3) an+2 = an , a2n+1 = 0,

223
Advanced Engineering Mathematics Chapter 8 - Solution Of Differential Equations In Series

1 2 1 1
C = 1, , y = Ax (1 x x4 x 6 ) + B].
2.5 2.4.5.9 2.4.6.5.9.13
30. x y" + y' + x y =0 Bessel's equation of order zero.
2
[Answer: Recurrence relation (C + n + 2) an+2 + an = 0 , a2n+l = 0 , C = 0 , 0
1 2 1 1
y = (A + B n x) (1 x x4 x 6 ) +
2 2 2
2 (2.4) (2.4.6)
1 1 1 1 1 1
+ B ( x2 (1 )x 4 (1 )x 6 )
22 (2.4)2 2 (2.4.6)2 2 3
2 2
31. x y" x y' + (x + 1) y = 0.
2
[Answer: Recurrence relation (C + n + 1) an+2 + an = 0 , a2n+l = 0, C = 1 , 1,
1 2 1 1
y = x(A + B n x) (1 x x4 x 6 ) +
22 24 (2!)2 26 (3!)2
1 1 1 1 1 1
+ B x ( x2 (1 )x 4 (1 )x 6 ) ].
22 (2.4)2 2 (2.4.6)2 2 3

32. x y" 2 y' + y = 0


[Answer: Recurrence relation (C + n 2) (C + n + 1)an+1 + an = 0,
1 1 1 5
y = (A + B n x) ( x 3 x 4 x ) +
12 48 480
1 1 1 19 3 137 3
+ B (1 x x 2 x 3 x x ) ].
2 4 36 576 28800
33. x y" + 2 y' + x y = 0
[Answer: Recurrence relation
(C + n + 2)(C + n + 3) an+2 + an = 0, a2n+1 = 0 , C = 0 , 1
1 1 1 1 1
y = A [1 x 2 x 4 ] + B [1 x 2 x 4 ] ].
x 2! 4! 3! 5!
2
34. x (x + l) y" + x(x + l)y' y = 0.
[Answer: Recurrence relation (C + n + 2) an+1 + (C + n )an = 0 , C = 1, 1
1 1 1 1
y = A x [1 x x 2 x 3 ] + B x (1+x)].
3 6 10
38. 2x y" + y' y = x + 1.
[Answer: Recurrence relation
(C + n + 1) (2C + 2n + 1) an+1 an = 0 , C = 0,
1 1 1 1 1 3
y = A [1 x x 2 x 3 ] + B x [1 x x 2 x ] 2x].
6 90 3 30 360

224
Advanced Engineering Mathematics Chapter 8 - Solution Of Differential Equations In Series

3
36. (1 x ) y" + 6 x y = 0.
[Answer: Recurrence relation (n + 3) an+3 (n 3)an = 0 , C = 0,
1 1 1
y = A(1 x ) + B [x x 4 x 7 x10 ] ].
3
2 14 35
2 2 2
37. 2 x (1 + x ) y" + xy' 12 x y = 0.
[Answer: Recurrence relations (2C + 2n + 3) an+2 = (2C + 2n 6)an , C = 0 ,
4 8 16 8
y = A [1 2x 2 x 4 x 6 x ] +
7 77 385
5 5 5 6
+ B x [1 x 2 x 4 x ] .
4 32 128
38. x y" + y' 4 x y = 0
2
[Answer: (C + n + 2) an+2 = 4 an , C = 0 , 0 ;
1 1
y = (A + B n x)( 1 x 2 x4 x6 )
2 2
(2!) (3!)
1 1 1 1 1
B (x 2 (1 )x 4 (1 )x 6 ) .]
(2!) 2 2 (3!) 2 2 3

39. x y" + 2y' + x y = 0. ( See problem 33 )


[Answer: (C + n + 2)(C + n + 3) an+2 + an = 0, a2n+1 = 0 , C = 0 , 1
1 1 1 1 1 1
y = A ( x x 3 ) + B ( 1 x 2 x 4 x 6 ) =
x 2! 4! 3! 5! 7!
A B
= cos x sin x .]
x x
2 2
40. 2 x y" + ( 3x 2x ) y' (x + 1) y = 0
[Answer: (C + n + 2) an+1 an = 0, , C = , 1
1 1 1 1 1 1
y = A ( 1 x x 3 ) + B x [1 (2x) (2x)2 (2x)3 ] .
x 2! 3! 5 5.7 5.7.9
2 2
41. 9 x y" (x 2) y = 0
[Answer: (3C + 3n + 5)(3C + 3n + 4) an+2 an = 0, C = 1/3 , 2/3
1 2 1 1
x4 x 6 ) +
1/3
y = A x (1 x
5.6 5.6.11.12 5.6.11.12.17.18
1 2 1 1
x4 x 6 ) ].
2/3
+ B x (1 x
6.7 6.7.12.13 6.7.12.13.18.19
2
42. x y" x2 y' + ( x 2 ) y = 0
[Answer: (C + n + 2) an+1 an = 0, C = 1 , 2,

225
Advanced Engineering Mathematics Chapter 8 - Solution Of Differential Equations In Series

1 x 1 1 4 1 5
y = A ( 1 ) + B (x 2 x 3 x x ) ].
x 2 4 4.5 4.5.6
2 2
43. x y" + 2 x y' ( x + 2 x + 2) y = 0
[Answer: (C + n + 1)(C + n + 4) an+2 2 an+1 an = 0, C = 1 , 2
1 1 1 3
y = (A + B n x) x 1 x x 2 x +
2 5 20
1 1 2 1 4 1 5 7 6
+A 1 x x x x x ].
2 2 8 40 720
x
44. x y" x y' y = 0
[Answer: (C + n) an+1 an = 0, C = 0 , 1;
1 3 11
y = (A + B n x) x e + B 1 x 2 x3 x 4 ].
x

12 12.22 12.22.32
48. x y" + y' x y = 0
2
[Answer: (C + n + 2) an+2 an = 0, C = 0 , 0;
1 2 3 1 3 4
y = (A + B n x) 1 x x 4 +B x 2 x ].
22 22.42 4 128
3
46. (2x + x ) y" y' 6 x y = 0
[Answer: (2C + 2n + 1) an+2 + (C + n 3)an = 0, C = 0 , 3/2 ;
2 3 4 1 6 1 8
y = A( 1 + 3x + x x + x + ) +
5 15 65
3/2 3 2 1.3 4 1.3.5 6 1.3.5.9 8
+B x ( 1 + x x + x x + )].
8 8.16 8.16.24 8.16.24.32
47. 4 x y" + 2 y' + y = 0
[Answer: 2(C + n + 1)(2C + 2n + 1) an+1 + an = 0, C = 0 , ;
2 2
y = A ( 1 x/2! + x /4! ) + B x ( 1 x/3! + x /5! ) =
= A cos x + B sin x )].
48. 2x(l x) y" + (l x)y' + 3y = 0
[Answer: (2C + 2n + 1) an+1 (2C + 2n 3)an = 0, C = 0 , ;
3 2 3 3 3 4 3 5
y= A x (l x) + B( 1 3x + x x x x )]
1.3 3.5 5.7 7.9
2
49. ( x x ) y" + (1 x) y' y = 0
2 2
[Answer: (C + n + 1) an+1 (C + n + 2) an = 0, C = 0 , 0;
2 2 2 2 2 3 2 4
y = (A + B n x)(1 + 2 x + 3 x + 4 x + 5 x + )

226
Advanced Engineering Mathematics Chapter 8 - Solution Of Differential Equations In Series

2 3
2B(1.2 x + 2.3 x + 3.4 x + )].
2
50. ( x x ) y" + (1 x) y' y = 0
2 2
[Answer: (C + n + 1) an+1 [(C + n) + 1]an = 0, C = 0 , 0;
2 3 4 2 3
y =(A + B n x)(1+x+ 2 x + 2.5 x + 2.5.10 x + ) + B(2x + x + 14 x + )].
4 4.9 4.9.16 27
51. x y" + (1 + x) y' + 2y = 0
2
[Answer: (C + n + 1) an+1 + (C + n + 2) an = 0, C = 0 , 0;
2 3
y =(A + B n x)(12x+ 3 x 4 x +) +
2! 3!
1 1 1 2 1 1 1 3
+ B(2 (2 ) x + 3 (2 ) x + 4 (2 ) x )].
2 2! 2 3 3! 2 3 4
2 2
52. x y" + x y' + (x 1) y = 0
[Answer: (C + n + 1) (C + n + 3) an+2 + an = 0, C = 1 , 1;
3 5
y =(A + B n x)( 1 x+ 21 x + 2 12 x + ) +
2 .4 2
2 .4 .6
3 5
+B(1/x + 12 x (1 1 ) 21 x + (1 1 1 ) 2 12 x + )].
2 4 2 .4 4 6 2 .4 .6
2 2
53. x y" + x y' + ( x 4 ) y = 0 Bessel's equation of order 2.
[Answer: (C + n ) (C + n + 4) an+2 + an = 0, C = 2 , 2;
2 4 6 8
y =(A + B n x)( 21 x + 31 x 3 12 x + 3 2 12 x ) +
2 .4 2 .4.6 2 .4 .6.8 2 .4 .6 .8.10
2 1 1 2 11 4 31 4
+ B (x + 2+ 2
x 2
x + 2
x )].
2 (2.4) (2.4.6) (2.4.6.8)

54. x ( 1 x ) y" 3 x y' y = 0


[Answer: (C + n ) an+1 (C + n + 1) an = 0, C = 0 , 1;
2 3 4
y =(A + B n x) ( x + 2x + 3x + 4x + ) +
2 3 2 1
+ B(1 + x + x + x + x4 + ) = ( A + B n x)( x(l x) ) + B(l x) ].
58. x ( 1 x ) y" ( 1 + 3x ) y' y =0
[Answer: (C + n 1) an+1 (C + n + 1) an = 0, C = 0 , 2;
2 3 4
y =(A + B n x) ( 1.2 x + 2.3x + 3.4x + ... ) +
2 3 4
+ B( 1 + x + 5x + 11x + 19x + )].
2 3
56. ( x + x + x ) y" + 3 x2 y' 2 y = 0
[Answer: (C + n +1) an+2 +(C + n 1) an+1 +(C + n ) an = 0, C = 0 , 1;
2 3 4 5
y =(A + B n x) ( 2x + 2x x x + 5 x )+
4

227
Advanced Engineering Mathematics Chapter 8 - Solution Of Differential Equations In Series

2 3 4
+ B( 1 x 5x x + 11x /3 )].
2
57. ( 1 x ) y" 2 x y' + 2 y = 0 Legendre's equation of order unity.
2 4 6
[Answers: ( n + 1) an+2 (n 1) an = 0; y = A x + B(l x x /3 x /5 + ].
2
58. ( 1 x ) y" 2 x y' + m(m + 1) y = 0 Legendre's equation of order n.
[Answers: (n + 2) (n + 1) an+2 ( n m) ( n + m + 1) an = 0;
2 4
y = A(1 m(m + l)x /2! + m(m + l)(m + 3)(m 2)x /4! ) +
3 5
+ B( x (m l)(m + 2)x /3! + (m l)(m 3)(m + 2)(m + 4) x /5! )].
2
59. y" + x y = 0
[Answers: (n + 4) (n + 3) an+4 + an = 0;
4 8 12
y = A (1 x /(3.4) + x /(3.4.7.8) x /(3.4.7.8.11.12)+ ) +
5 9 13
+ B(x x /(4.5) + x /(4.8.8.9) x /(4.8.8.9.12.13) + ) ].
2
60. (2 + x ) y" + x y' + (1 + x) y = 0
2
[Answers: 2(n + 3) (n + 2) an+3 + [(n + 1) + 1] an+1 + an = 0;
2 3 4 5
y = A( 1 x /4 x /12 + 5x /96 + 13x /480 + ) +
3 4 5
+ B( x x /6 x /24 + x /24 + ) ].

228
Advanced Engineering Mathematics Chapter 8 - Solution Of Differential Equations In Series

REFERENCES
(1) Advanced Mathematics for Engineers and Scientists By Murray R. Spiegel.
(2) Advanced Calculus, By Murray R. Spiegel.
(3) Laplace Transforms, By Murray R. Spiegel.
(4) Vector Analysis, By Murray R. Spiegel.
(5) Differential Equations, By Jr. Frank Ayres.
(Schaum's outline series in Mathematics-McGraw-Hill book company)
(6) A course in Mathematics Parts I, II, III. By Dr. Naguib G. Bakhoom.
(7) Mathematics for Engineers, By Dr. Abdel-Latif El-Sedeek.
(8) Mathematics for Engineering Students (1984)
By Dr. A. A. Azim, Dr. M. Shams, and Dr. Labib Iskandar
(9) Solved Examples for Engineering Students (1983)
By Dr. A. A. Azim, Dr. M. Shams, and Dr. Labib Iskandar.
(10) Mathematics for Engineering Students Part II
By Dr. Abdel-Lateef El Sedeek and Dr. S. Elnahwy
(Faculty of Engineering Cairo University)
(11) Special functions By Earl D. Rainville.
(12) Special functions of Mathematical Physics and Chemistry By Ian N. Sneddon
(Oliver & Boyd LTD.)
(13) The theory of ordinary differential equations By J. C. Burkill
(Oliver and Boyd LTD.)
(14) Integration of ordinary differential equations By E. L. Ince
(Oliver and Boyd LTD.)
(15) An elementary treatise on differential equations and their applications.
By H.T.H. Piaggio.
(16) Volume and integral By W. W. Rogosinski (0 & B)
(17) Vector Methods By D.E. Rutherford (0 & B)
(18) Multiple Integrals, Field theory and Series By B. M. Budak and S. V. Fomin.
(19) University Mathematics By J. Blakey
(A textbook for students of Science and Engineering)
(20) Laplace Transform theory By M. G. Smith.
(21) Problems in Mathematical Analysis - Under the editorship of B. Demidovich.
(22) Differential and Integral Calculus By N. Piskunov.

229

Potrebbero piacerti anche